Sei sulla pagina 1di 202

Basic Mathematics

& Measurements

Section I
Single Correct Option

1, K =

1
mv2
2

p2 =
m
v
K
=
+2

K max
m
v

Maximum error = 2% + 2 (3%)


In the estimate of,
kinetic energy ( K ) = 8%
Option (c) is correct.
m m
2. d =
= 3
V
l

m
l
d 100
=
100 + 3 100

d
max m
l

= 4% + 3(3%)
= 13%
Option (d) is correct.
F
F
3. p =
=
A L2
Permissible error in pressure ( p)
= 4% + 2 (2%)
= 8%
Option (a) is correct.
4.

p1V1 = p2 V2
V
p2 = p1 1
V2
= p1
=

V1
V1 10% of V1

p1
90%

p1
9

Percentage increase in pressure =

100
9

= 111
.
Option (a) is correct.
p2
5. K =
2m
Error in the measurement of kinetic
energy (K)
= 2 100%
= 200%
Option (d) is correct.
6. 3400 = 3.400 103
Number of significant figures = 2
Option (d) is correct.
7. A = 3.124 m 3.002 m
= 9.378 / 248 m 2
= 9.378 m 2
Option (a) is correct.
GM
8.
g= 2
R
constant
=
R2
1 2
K = I
2
1 2
= MR2

2 5
= constant R2

Basic Mathematics & Measurements


Decrease in R (radius) by 2% world
increase g by 4% and decrease K
(rotational kinetic energy) by 4%.

16. Q = I e

= 12 6 2.45
= 176.4
= 1764
.
102 cm 3
2
= 2 10 cm 2
Option (b) is correct.
P
11. I =
4 r2
i.e., Ir2 = constant
i.e., if r is increased by 2% the intensity
will decrease by 4%.
Option (d) is correct.
12. Option (b) is correct.
4
13.
V = r 3
3
V
r
=3
V
r
= 3(1%)
= 3%
Option (c) is correct.
14. a 3 = 6a2 (given)
a=6
V = 63 = 216 m 3
Option (b) is correct.
l
15. g = 4 2 2
T
l
T
g

100
=
100 + 2
100
T

l
g
max
1 mm

015
.
=
100 + 2
10
2003

100 m

= 01
. % + 01
. % = 0.2 %
Option (a) is correct.

(given)

Q = It

9. Heat (H) = 22 Rt

10. V = lbt

| 3

We know that

Option (b) is correct.


Maximum error in measuring heat (H)
= 2 (2%) + 1% + 1%
= 6%
Option (b) is correct.

tI

( V ) 0

t= e

tI
(V ) 0

[ ] = [ t ]
tI
and [ ] =

( V ) 0

or

= tI = I 1
( V ) 0 t ( V ) 0
1
1
=
[Resistance] 0
1
1
=
[ML2 T 3A 2 ] [M1L3T 4 A 2 ]
1
= 1 = [velocity]
[L T]
1
=
[ 0 0 ]1/ 2

Option (a) is correct.


pq
17. a = 2 3
r s
1 p
1 q

a 100
=
100 +
100
a
max 2 p
2 q

r
s

+2
100 + 3
100
r

1
1
= (1%) + (3%) +2 (0.5%) + 3(0.33%)
2
2
= 0.5% + 1.5% +1% + 1%
= 4%
Option (c) is correct.
18. Least count of main scale
2 mm
=
= 0.5 mm
4
least count of main scale
Least count =
50
= 0.1 mm
Zero error = 30 0.01 mm
= 0.3 mm
(ive sign, zero of circular scale is lying
above observed reading of plate thick)
= 2 MSR + 20 CSR

4 | Mechanics-1
= (2 05
. mm) + (20 0.01 mm)
= 1 mm + 0.2 mm
= 1.2 mm.
Plate thickness (corrected reading)

= observed reading zero error


= 1.2 mm + 0.3 mm
= 1.5 mm
Option (d) is correct.

More than One Correct Options

1. Maximum percentage error in x :


= (% error in a) + 2 (% error in b) +3 (% error in c)
= 15%
Assertion and Reason

1. Least count of screw gauge


Pitch
=
Number of divisions of circular scale
Less the value of pitch, less will be least
count of screw gauge leading to len
uncertainty that is more accuracy in the
measurement.

Thus, assertion is true.


From the above relation we conclude that
least count of screw gauge is inversely
proportional to the number of divisions of
circular scale.
Thus reason is false.
Option (c) is correct.

Match the Columns

1. (a) F =

GM1M2
r2

GM1M2 = Fr2
i. e., [GM1M2 ] = [MLT 2 ][L2 ]
= [ML3T 2 ]
(a) (q)
3 RT 3 pV 3 work
(b)
=
=
M
n
nM
3 RT ML2 T 2

=
= [L2 T 2 ]
M M

qvB sin
=

qB

= v2 sin2

F2
1 2
2 2
q2 B2 = [LT ] = [L T ]

(c) ( r)
(d)

g=

GM e
R2e

GM e
= gRe
Re

(b) (r)
F2
F
(c) 2 2 =
qB
qB

GM e
2
R = [LT ][L]
e
= [L2 T 2 ]

(d) ( r)

Units & Dimensions


Vectors
Section I

Single Correct Option

Force
Area
[MLT 2 ]
[ p] =
[L2 ]

1. Pressure ( p) =

= [ML1T 2 ]
Option (d) is correct.
2.

W = I 2 Rt
[ML2 T 2 ] [ML2 T 2 A 2 ]
[R] =
(i)
=
[T ]
[A 2 T]
dI
and W = Vq
V =L
dt
W dt
L=
q dI
[L] =

[ML2 T 2 ][T]
= [ML2 T 2 A 2 ]
2
[A T]

Using Eq. (i)


[ L]
[T ]
L
[T ] =
R
[R] =

i.e.,

Option (c) is correct.


3.

F = 6 av
[F ]
[ ] =
[ av ]
[ML T 2 ]
=
[L L T 1 ]

= [ML1 T 1 ]
Option (d) is correct.

4.

= Li
[ ] = [ L ][ i]

= [ML2 T 2 A 2 ] [ A ]
= [ML2 T 2 A 1 ]
Option (a) is correct.

5. Linear impulse ( I ) = F t

[I] = [ML T 2 ] [T]


= [ML T 1 ]
Option (c) is correct.
mm
6. F = G 1 2 2
d
[ F ][ d2 ] [ML T 2 ] [L2 ]
[G] =
=
[ m1 m2 ]
[M2 ]
= [M1 L3 T 2 ]
Option (c) is correct.
i i
7.
F= 0 1 2
4 d
[ML T 2 ][L]
[ 0] =
= [ML2 T 2 A 2 ]
[A 2 ]
Option (c) is correct.
[L]
8. [ k] =
= [T]
[L T 1 ]
Option (c) is correct.
[ML T 2 ]
9. [ a ] =
= [ML T 3 ]
[T]
[ b] =

[ML T 2 ]
= [ML T 4 ]
[ T2]

Option (c) is correct.

6 | Mechanics-1
10. E = h

11. [Energy] = [ML2 T 2 ]

q = CV
and
V = iR

q = iCR
i t = iCR
[CR] = [ t ] = [M0 L0 T A 0 ]
Option (a) is correct.
1 q1 q2
18. F =
4 0 r2

= [M][L T 1 ] 2
[Mass] = [ Ev 2 ]
Option (c) is correct.
1
Energy
12. 0 E2 = Energy density =
2
Volume
2
[ML
T 2 ]
1
2
E =

0
3
2

[L ]

Unit of 0 = Newton-metre2 /coulomb2 .


Option (b) is correct.
nh
19. Angular momentum ( J ) =
2
I = mr2
h 2 J / n
mvr

=
=
2
I
mr
mr2

17.

[ML2 T 2 ]

[ h] =
= [ML2 T 1 ]
[T 1 ]
nh
Angular momentum ( J ) =
2
[ J ] = [ h] = [ML2 T 1 ]
Option (b) is correct.

1
h = [L T ] = [T 1 ]
I L

= [ML1 T 2 ]
Option (b) is correct.
[ a ] = [T 2 ]

13.

[T 2 ]
[ b] =
[L ][ML1 T 2 ]

a = [M T 2 ]
b

Option (b) is correct.


dv
14. Velocity gradient =
dx
[L T 1 ]
[Velocity gradient] =
[L]
= [T 1 ]
= [M0 L0 T 1 ]
Option (a) is correct.
[Force] = [ML T 2 ]
[F]
[Mass] =
[L T 2 ]

15.

= [F L1 T 2 ]
Option (a) is correct.
16. Coefficient of friction ()

Limitting frictional force


Normal force

[] = [M0 L0 T 0 ]

Option (b) is correct.

= Frequency
Option (a) is correct.
b
20.
v = at +
t+c
b
t +

[ c] = [ T ]

= [v ]
c

[ b] = [L T 1 ] [T] = [L]
[ at ] = [v ] = [L T 1 ]
[a] = [L T 2 ]

Option (a) is correct.


2
21.
y = A sin ( c t x)

x
2
= A sin c t


2x
= (angle)

[ x ] = [ ] = [L ]
Further, y = A sin

[ A ] = [ y ] = [L ]
Option (a) is correct.
or

22.

[ X ] = [M1 L3 T 3 A 2 ]
[T A 2 ]
=
[ML2 T 2 ]

Units & Dimensions Vectors


2

[ t] [ i ]
[ Work ]

[QW = i2 Rt]

X is resistance.

F = 2 ^i 3 ^j + 4 k

23.

or

24.

0.25 + 0.64 + c2 = 1
c2 = 1 0.89
c = 0.11
Option (b) is correct.
or
or

A2 + 2 A B + B2 = C2

A B = 0

or

cos = 0

=
2

122 + 52 + 2 P Q = 132

P Q = 0

33. Option (b) is correct.

27. Work ( = F s ) is a scalar quantity.

or

Option (d) is correct.

or

28. Speed = |v|

or

Option (d) is correct.

29. |A |= 3, |B|= 5 and angle between A and B

A B = |A ||B|cos 60

34. P + Q + R = 0

Option (d) is correct.

is 60.

Option (b) is correct.

|A |= |B|

Angle between P and Q =

A2 B2 = 0
A=B

A A + B A B B A B = 0

(P + Q) (P + Q) = R R

26. ( A + B) ( A B) = 0

P2 + Q2 + 2 P Q = R2

|A ||B|cos = 0

Angle between A and B = 90

32. P + Q = R

A B = 0

A A + 2 A B + B B = C C

or

Option (d) is correct.

A2 + B2 + 2 A B = A2 + B2 2 A B

31. Magnetic field intensity.

( A + B) ( A + B) = ( A B) ( A B)

i.e.,

Option (d) is correct.

or

25. |A + B|= |A B|

or

or

(0.5)2 + (0.8)2 + ( c)2 = 1

( A + B) ( A + B) = C C

or

= 17 ^i 6 ^j 13 k

^ ^ ^
i
j k

= r F = 3 2 3
2 3 4

= 7.5
Option (b) is correct.
30. A + B = C

r =3i+2j+3k

1
= (3) (5)
2

P+Q= R

(P + Q) (P + Q) = ( R) ( R)

P P + Q Q + 2 P Q = R R

P2 + Q2 + 2 P Q = R2

Let Q2 = P2 and R = P 2
Thus, Eq. (i) takes the form
P2 + P2 + 2 PQ cos = 2 P2
or
2 PQcos = 0

(i)

8 | Mechanics-1
or
or

( Q sin )
Q sin

P + Q cos P Q cos
=
( Q sin )
Q sin
1

( P + Q cos ) ( P Q cos )
2 PQ sin
= 2
P + Q2 cos 2

cos = 0
= 90

Angle between P and Q is 90

P+Q+R=0

P+R= Q

(P + R) (P + R ) = ( Q) ( Q)

or
or
or
or
or

P2 + R2 + 2 PR cos = Q2
2 PR cos = Q2 P2 R2
2 PR cos = R2
2P cos = R
2 P cos = P 2
1
cos =
2

or

or

P Q will vary from 0 to .


Option (b) is correct.
36. R2 = P2 + Q2 + 2 PQ cos

for R = P = Q
P2 = P2 + P2 + 2 PP cos
1
or
cos =
2
or
= 120
Option (b) is correct.

= 135

Angle between P and R is 135.

W = F s

37.
135

= (3 ^i + 4 ^j) (3 ^i + 4 ^j)

P
90

= 25 J
Option (b) is correct.

135

+Q
P

'
P
Q

tan =

Q sin
P + Q cos

Angle between P Q and P


Q sin ( + )
tan =
P + Q cos ( + )
Q sin
=
P Q cos
tan tan
tan[ + ( )] =
1 + tan tan

= a2 2a 3

35. Angle ( ) between P + Q and P Q

38. P Q = ( a i + a j + 3 k) ( a i 2 j k)

Option (a) is correct.

This implies that angle between P + Q and

For P Q, P Q = 0
i.e., a2 2a 3 = 0
or ( a 3)( a + 1) = 0

a=3
Other value is ive.
Option (d) is correct.
39. If a vector makes angles , and with

the co-ordinate axes, then


cos2 + cos2 + cos2 = 1
2

3
9 6
Now, =
, =
7
49 7
9
and
+
49
Option (a) is correct.

40. A = 4 i 3 j

B = 8 ^i + 8 $j

and

36 2
4
, =
49 7
49
36
4
+
=1
49 49

A + B = C = 12 ^i + 5 ^j

Units & Dimensions Vectors

C=

B=2N
Option (c) is correct.

12 ^i + 5 ^j
C
=
|C|
122 + 52

C = i + 2$j + 3 k

Vectors A , B, and C will be coplanar if


their scalar triple product is zero i.e.,

(A C ) B = 0
$
i$ $j k
^

2 3 2 (5 ^i + n ^j + k) = 0

1 2 3

or

=a a +ba a bbb

=0a ba b0

= 2(a b) = 2( b a )
Option (a) is correct.
44.

A = 3 ^i + 4 ^j + 5 k

2 + 32 2

47. R2 = (3 P)2 + (2 P)2 + 2 3 P 2 P cos

43. (a + b) (a b)

2+3
2

Option (a) is correct.

42. Option (a) is correct.

(13 ^i 4 ^j + 7 k) (5 ^i + n ^j + k) = 0

Component of A along ^i + ^j

5
C=
(2 ^i + 3 ^j )
26

5
|C|=
2

or
65 4 n + 7 = 0
or
n = 18
Option (a) is correct.

A B

|A ||B|
5
=
13 2
5
=
26

B = ^i + ^j

and

^
^
^
41. A = 2 i + 3 $j 2 k, B = 5 i + n$j + k,

46. Angle between A = 2 i + 3 j

12 ^ 5 ^
=
i+
j
13
13
Option (b) is correct.

or
(i)
R2 = 13 P2 + 12 P2 cos
Further
(2R)2 = (6 P)2 + (2 P)2 + 2 6 P 2 P cos
or 4 R2 = 40 P2 + 24 P2 cos
(ii)
Dividing Eq. (ii) by Eq. (i),
10 P2 + 6 P2 cos = 13 P2 + 12 P2 cos
or
6 cos = P
or
= 120
Option (b) is correct.
Q sin
48.
tan =
P + Q cos

B = 3 ^i + 4 ^j 5 k

cos =

( A B)

=0

= 90
Option (c) is correct.
45.

Q = 2P

|A ||B|
9 + 16 25
= 2
3 + 42 + 52

A + B=7
A B=3

R
= 90
P

As

i.e.,

= 90, tan =
P + Q cos = 0
P
Q
P
=
2P

cos =

10 | Mechanics-1
1
2
= 120

or

P 16 = 96

Option (a) is correct.

Option (a) is correct.


49. A B = 0

51. |A B|= 3 ( A B)

A

B C

tan = 3
= 60

|A B|= [ A2 + B2 + AB] 1/ 2

A C = 0

(ii)

AC

= A2 + B2 + 2 AB cos 60
= A2 + B2 + AB

(i)

AB

|A B|2 = |A |2 |B|2 + 2|A ||B|cos

|A ||B|sin = 3 |A | |B|cos

P = + 6N
Q = 10 N

52. C is perpendicular to both A and B



C=AB

From Eq. (i) and Eq. (ii), we conclude that

A is perpendicular to the plane containing

B and C.

This implies that A is perpendicular to


B C.
Option (c) is correct.
or

or
or

P + Q + 2 PQ cos = 8

( P + Q)2 + 2 PQ (cos 1) = 64

2 PQ(cos 1) = 192

Using Eq. (ii) and Eq. (i),


P ( P) PQ = 96
or

P ( P + Q) = 96

8 + 12 + 8 = 0

Option (c) is correct.

P+Q+R= 0

54.

P + Q cos = 0
Q cos = P

(A B ) A = 0

53. (2 i + 3 j + 8 k) ( 4 i + 4 j + k) = 0

(16)2 + 2 PQ (cos 1) = 64

PQ cos PQ = 96 (i)
Q sin
(as
tan =
=
P + Q cos
= 90)

Option (d) is correct.

or

C A = 0

or

or P2 + Q2 + 2 PQ + 2 PQ cos 2 PQ = 64
or

P2 + Q2 + 2 PQ cos = R2

50.

(ii)

If,

|P|
|Q|
|R|
=
=
sin sin sin

1
2

Units & Dimensions Vectors

56. Using answer to questions no. 35, as angle

OA + AB = OB

55.

between A + B and A B is 90

B (2,6,4)

A2 + B2 cos 2 = 0

or

A2 = B2 cos 2

or

A2 = B2 cos 2
2

or

A2 = B2 cos

or

A2 = B2

A=B

Y
A (0, 3, 1)

AB = OB OA

11

= ( 2 i + 6 ^j + 4 k) (0 ^i + 3 ^j k)

Q =

Option (a) is correct.

= 2 ^i + 3 ^j + 5 k
Option (c) is correct.
Match the Columns

1. (a) |A B|= |A B|

or

tan = 1
3

= ,
2 4
Thus, (a) (r) (s).

(b) A B = B A

or

4 A B = 0

AB

or
or
or

2 A B = 0 (Q A2 + B2 = C2 )

AB

Thus, (d) (q).

Section II
Subjective Questions

1. 2 1011 N/m 2 =

(2 1011)(105 dyne)
(104 cm 2 )

= 2 1012 dyne/cm 2
(72)(105 N)
2. 72 dyne/cm =
= 0072
.
N/m
(102 m )
3.

[ a ] = [ y] = [ L]

A2 + 2 A B + B2 = C2

A A + B A + B A + B B = C C

or

|A + B|2 = |A B|2

( A + B) ( A + B) = C C

(d) A + B = C

(c) |A B|= |A B|
or

Thus, (c) (q).

or
sin = sin
or
2 sin = 0

= 0 rad
Thus, (b) (p).

|A ||B|sin = |A ||B|sin

or

A B = (B A )

or

= A A B A A B + B B

(given)

or

or A A + B A + B A + B B

|A ||B|sin = |A ||B|cos

( A + B) ( A + B) = ( A B) ( A B)

[t ] = [M0L0 T 0 ] [ ] = [ T 1 ]
[ ] = [M0L0 T 0 ]
E
J
4. h =
=
= J-s
per sec
[ h] = [ML2 T 2 ][T] = [ML2 T 1 ]
5. [ b] = [ x2 ] = [ L2 ]

12 | Mechanics-1
b = [ p]
at

14. A + B = R (say) = 3 i$ + $j

L2
b
[a] = =
= [M1T 2 ]
2 3
tp
TML
T

1 2
1

6. St = ut + at u( t 1) + a( t 1)2


2
2

1
1
2
= u.1 a(1) + at(1) = u + at a
2
2
a
= u + (2t 1)
2
Here t in second. Hence the given
equation seems to be dimensionally
incorrect. But it is correct because 1 is
hidden.
7. LHS is dimensionless. While RHS has the
dimensions [L1 ].
8. LHS is dimensionless. Hence n = 0.
9. Just write the dimension of different
physical quantities.
10. E = km x n y a z .
Here k = a dimensionless constant

[ E ] = [ m ]x [ n] y [ a ] z
2 2

[ML T ] = [M]x [T 1 ] y [L] z

x = 1, y = 2 and z = 2
x y z

11. F = km v r

(k = a dimensionless constant)
x
[ F ] = [ m ] [v ] y [ r] z
[MLT 2 ] = [M]x [LT 1 ] y [L] z
Solving we get,
x = 1, y = 2 and z = 1
kmv2

F=
r

A B = S (say) = i$ + 5$j

RS
3+5
8
4
cos =
=
=
=
RS
9 + 1 1 + 25
260
65
4
= cos 1

65
15. Their dot product should be zero.
$ should be
16. Ratio of coefficients of $i, $j and k
same.
17. No solution is required.

2 x

$ =R
R
R

$
20. A B = C (say) = 3 i$ + 8 $j + 2k

Now A C = 6 + 8 2 = 0

AC

BC = 0+8 8 = 0

BC

A B
62+8
13. cos =
=
AB
9 + 1 + 4 4 + 4 + 16
12
3
=
=
336
21

21. Area of parallelogram = |A B|

22.
B

B sin

A
B cos

[ML ] = [MLT ] [L] [T]


Equating the powers we get,
x = 1, y = 4, z = 2

[ d] = [FL4 T 2 ]
Similarly other parts can be solved.

A B
B

$
19. A + B = R = 5$i $j + k

18. Component of A along B = A cos =

12. (a) [ d] = [ F ]x [ L ] y [ T ] z
3

Angle between R and S

B sin = R =

B
2

= 30

Angle between A and B = 180 = 150

23. A + B = ( 4 $i + 6$j ) + B = 10 $i + 9 $j

B = (6$i + 3 $j)

Units & Dimensions Vectors


24.

3P

25.

4P

Applying sine law, we have


a
b
=
sin (180 A) sin (180 B)
c
=
sin (180 C)
a
b
c
or
=
=
sin A sin B sin C

2P

R = P $i + 2 P $j 3 P $i 4 P $j
= ( 2 P i$ 2 P $j )
26.

R2 = P2 + Q2 + 2 PQ cos
S2 = P2 + Q2 2 PQ cos
R2 + S2 = 2 ( P2 + Q2 )

13

Motion in One Dimension


Introductory Exercise 3.1

1. Suppose

a particle is moving with


constant velocity v (along the axis of x).
Displacement of particle in time t1 = vt1
Displacement of particle in time t2 = vt2
Displacement of the particle in the time
interval t ( = t2 t1)
= vt2 vt1
= v ( t2 t1)
Average velocity in the time interval
v ( t2 t1)
t =
( t2 t1)
=v
Now, as the particle is moving with
constant velocity (i.e., with constant speed
in a given direction) its velocity and speed
at any instant will obviously be v.
Ans. True.

2. As the stone would be free to acceleration

under earths gravity it acceleration will


be g.
3. A second hand takes 1 min i.e., 60s to

complete one rotation (i.e., rotation by an


angle of 2 rad).
Angular speed of second hand
2 rad
=
60 s

=
rad s 1
30
Linear speed of its tip = radius angular
speed

= 2.0 cm

rad s 1
30

cms 1
15
As the tip would be moving with constant
speed.

Average speed =
cms 1
15
In 15 s the second hand would rotate
through 90 i.e., the displacement of its tip
will be r 2.
Modulus of average velocity of the tip of
second hand in 15 s.
r 2
=
15
2 2
=
cms 1
15
=

4. (a) Yes. By changing direction of motion,

there will be change in velocity and so


acceleration.
(b) (i) No. In curved path there will always
be acceleration. (As explained in the
previous answer no. 3)
(ii) Yes. In projectile motion the path of
the particle is a curved one while
acceleration of the particle remains
constant.
(iii) Yes. In curved path the acceleration
will always be there. Even if the path
is circular with constant speed the
direction of the acceleration of the
particle would every time be changing.

Motion in One Dimension


Circumference
Speed
2 4 cm
=
= 8 = 25.13 s
1 cm / s

4v 2
2r
v
2 2 v2
=
r
2 2 (1)2
=
4

5. (a) Time speed =

(b) As particle is moving with constant


speed of 1 cm/s, its average speed in
any time interval will be 1 cm/s.
r 2
|Average velocity|=
T/4
=

4r 2
2 2
speed
=

2r

speed

| 15

= 0.23 cm s 2
6. Distance = Speed time

D1 = v1t1
D2 = v2 t2

2 2
cms 1

= 0.9 cms 1
v 2
|Average acceleration|=
T/4

Average speed =

D1 + D2 v1t1 + v2 t2
=
t1 + t2
t1 + t2
( 4 2) + (6 3)
2+3

= 5.2 ms 1

(where v = speed)

Introductory Exercise 3.2


1. Acceleration (due to gravity).

1
1
= ut + at2 u ( t 1) + a( t 1)2

2
2

st = u + at

1
a
2

Displacement
Velocity

Therefore, the given


dimensionally incorrect.

Acceleration

equation

v = t 3/ 4
ds
= t 3/ 4
dt

4.

1
2. st = u + at a is physically correct as it
2
gives the displacement of the particle in tth
second (or any time unit).
st = Displacement in t seconds
displacement in ( t 1) seconds

s=

(given)
(i)
3/ 4

dt =

3
+1
t4

3
+1
4

+c

4 7 /4
t
+c
7
i.e.,
s t7 /4
Differentiating Eq. (i) w.r.t. time t,
3
d2 s 3 4 1
=
t
dt2 4

a t 1/ 4
or

s=

5. Displacement (s) of the particle

is

3. Yes. When a particle executing simple

harmonic motion returns from maximum


amplitude position to its mean position
the value of its acceleration decreases
while speed increases.

s = ( 40 6) +

1
( 10) 62
2

= 240 180
= 60 m (in the upward direction)
Distance covered ( D) by the particle
Time to attain maximum height

16 | Mechanics-1
40
= 4s < 6 s
10
It implies that particle has come back
after attaining maximum height (h) given
by
u2
h=
2g
=

( 40)2
= 80 m
2 10

D = 80 + (80 60)
= 100 m

v = 40 10t
dx

= 40 10t
dt
or
dx = ( 40 10t) dt
or
x = ( 40 10t) dt
or
x = 40t 5t2 + c
As at t = 0 the value of x is zero.
c=0

x = 40t 5t2
For x to be 60 m.
60 = 40t 5t2
or
t2 8 t + 12 = 0

t = 2 s or 6 s
Displacement in time t
7. Average velocity =
t
1 2
ut + at
2
=
t
1
= u + at
2
6.

8.

v2 = v1 + at
at = v2 v1
Displacement in time t
t
1 2
v1t + at
2
=
t
1
= v1 + at
2
v2 v1
= v1 +
2
v1 + v2
=
2

Ans. True.

125 = 0 t +

9.

1 2
gt
2

t = 25 s
125 m
Average velocity =
5s

(downwards)

= 25 m/s

(downwards)

(i

v = 10 + 5t t

10.

)
dv
= 5 2t
dt
At
t =2s
a=522
= 1 m/s2
From Eq. (i),
dx
= 10 + 5t t2
dt

x = (10 + 5t t2 ) dt
5t2 t 3
or
x = 10t +

+c
2
3
As, at
t = 0 the value of x is zero
c=0
5
t3

x = 10t + t2
2
3
Thus, at t = 3 s
5
33
x = (10 3) + (3)2
2
3
= 30 + 22.5 9
= 43.5 m
a=

11. u = 2 i m/s

a = (2 cos 60 ^i + 2 sin 60 ^j ) m/s2


= (1 ^i + 3 ^j) m/s2
y

Average velocity =

a = 2 m/s2
60
u = 2 m/s

v=u+at
= 2 ^i + (1 ^i + 3 ^j) 2
= 4 ^i + 2 3 ^j

Motion in One Dimension

Part II Yes. As explained below.

|v|= 42 + 12 = 2 7 m/s

1
s = u t + a t2
2
1
^
= (2 i) 2 + (1 ^i + 3 ^j) 22
2

v = 2 ^i + 2t ^j implies that initial velocity of

the particle is 2 ^i m/s2 and the acceleration


is 2$j m/s2

s (at t = 1 s) = (2 ^i 1) + (2 ^j) 12
2

= 4 ^i + 2 ^i + 2 3 ^j

= (2 ^i + ^j) m

= 6 ^i + 2 3 ^j

13. x = 2t and y = t2

|s |= 36 + 12

v = (2 ^i + 2t ^j)m/s

(i)

dv
= 2 ^j
dt

a = 2 ^j m/s2

From Eq. (i),

ds
= (2 ^i + 2t ^j )
dt
s = (2 ^i + 2t ^j ) dt

or,
x2 = 4 y
(The above is the equation to trajectory)
x = 2t

dx

= 2 i.e., v x = 2 ^i
dt
y = t2

dy

= 2t i.e., v y + 2t ^j
dt

Thus,

x
y =
2

=4 3m
12. Part I

| 17

v = vx + v y
= (2 ^i + 2t ^j) m/s

s = 2t ^i + t2 ^j + c

Taking initial displacement to be zero.

s (at t = 1 s) = (2 ^i + ^j) m

a=

dv
= 2 ^j m/s2
dt

Introductory Exercise 3.3


1. At t = t1
v

t2

t1
0

t1

t2

v = tan
As < 90, vt1 is + ive.
At t = t2
vt 2 = tan
As > 90, vt 2 is ive.
Corresponding v-t graph will be

Acceleration at t = t1 :
at1 = tan
As < 90, a t1 is + ive constant.
Acceleration at t = t2
at 2 = tan
As < 90, at 2 is + ive constant.
2. Let the particle strike ground at time t

velocity of particle when it touches ground

18 | Mechanics-1
1
mg2 t2
2
i.e., KE t2 . While going up the velocity
will get ive but the KE will remain. KE
will reduce to zero at time 2 t when the
particle reaches its initial position.

Corresponding velocity-time will be

would be gt. KE of particle will be

Velocity (m/s)

4
KE

4.

t =

KE =

2t

2h
g

time

t (s)

h
2
= tan =
( t 2)
(2 1)
h = 2 ( t 2)

a (m/s )

1
1
2h
mg2 t2 = mg2
2
2
g

= mgh

(t2)

3. Speed of ball (just before making first

collision with floor)


= 2gh = 2 10 80
= 40 m/s
Time taken to reach ground
2 80
2h
=
=
=4s
g
10
Speed of ball (just after first collision with
floor)
40
=
= 20 m/s
2
Time to attain maximum height
20
t=
= 2s
10
Time for the return journey to floor = 2 s.
Speed (m/s)

t
h

Particle will attain its initial velocity i.e.,


net increase in velocity of the particle will
be zero when,
area under a-t graph = 0
(1 + 2) 2 ( h) ( t 2)
+
=0
2
2
or
3 ( t 2)2 = 0
or
( t 2)2 = 3
or
t2= 3
or
t=2 3
Ans . At time t = 2 + 3 s
(t = 2 3 not possible).

t (s)

Introductory Exercise 3.4


1. Relative acceleration of A w.r.t. B

a AB = ( + g) ( + g) = 0
2. Velocity of A w.r.t. B = v A vB

Relative displacement (i.e., distance


between A and B) would be
1
s = (v A vB ) t + a AB t2
2

Motion in One Dimension


or

sin 30 sin sin (180 30 + )


=
=
150 t
20t
500 103
1
sin
=
300
20
1
= sin 1
15
15
sin (30 + )
1
Now
=
3
300 t
500 10
224
5000
or
= sin 30 cos + cos 30 sin
3t

s = (v A vB ) t
tan = (vAvB)

3. In figure, u = speed of boat

v = speed of river flow


B

C
Actual
path of boat

Boat sailing
direction

or

u
d = 400 m

A v

Time to cross river


AB BC
AC
=
=
=

2
u
v
u + v2

400 m
=
10 m/s
= 40 s
v
BC = AB
u
2 m/s
=
( 400 m )
10 m/s
= 80 m
4. Let C be the point along which pilot

should head the plane.


B

30

5 00

km

North

Drifting due
to wind

Speed = 150 m/s

A
Wind/speed = 20 m/s

Apply sine formula in ABC

| 19

East

5000 1 224
3 1
=
+

3t
2 15
2 15
5000
= 0.5577
3t
5000
t=
3 0.5577
= 2989 s
= 50 min

5.

10 m
A

a A = 1 m/s2 ,
aB = 2 m/s2
v A = 3 m/s,
vB = 1 m/s
Acceleration of A w.r.t. B = 1 2 = 1 m/s2
Velocity of A w.r.t. B = 3 1 = 2 m/s
Initial displacement of A w.r.t. B = 10 m
At time relative displacement of A w.r.t. B
1
s = 10 + 2 t + ( 1) t2
2
or
s = 10 + 2t 0.5t2
For s to be minimum
ds
=0
dt
or
2 (0.5 2t) = 0
i.e.,
t = 2s

smin = 10 + (2 2) 0.5 (2)2


= 10 + 4 2
= 8m
Minimum distance between A and
B = 8 m.

20 | Mechanics-1
AIEEE Corner
Subjective Questions (Level 1)
1. (a) D = v1t1 + v2 t2

km
km 1
= 60
1 h + 80
h


h
h
2
= 100 km
(b) Average speed
100 km
=
= 66.67 km/h.
1.5 h
2. (a) Displacement in first two seconds

= ( 4) (2) +

1
(6) 22
2

= 20 m

20 m
Average velocity =
= 10 m/s
2s
(b) Displacement in first four seconds
1
= ( 4) ( 4) + (6) 42 = 64 m
2
Displacement in the time interval
t = 2 s to t = 4 s
= 64 20
= 44 m
44 m
Average velocity =
= 22 m/s.
2s

(b) Average velocity =


=

Net displacement
Time taken

60 m
6.1 s

= 9.8 m/s
(downwards)
= 9.8 m/s
vt
4. Average velocity =
t
vt0 + 2 vt0 + 3vT
2.5v =
t0 + t0 + T

or

5t0 + 2.5T = 3 t0 + 3 T
T = 4 t0

5. (a) Average acceleration

Final velocity initial velocity


( 4 + 8) s
00
=
12
= 0 m/s2
v
(c) a = tan = max
4
=

v
vmax

3. Let the particle takes t time to reach

ground.
+20 m/s

4s

60 m

64 = 20 t +

5t2 20t 64 = 0
t = 6.1 s
If the particle goes h meter above tower
before coming down
0 = (20)2 + 2 ( 10) h

h = 20 m
Total distance moved
(a) Average speed =
Time taken
(20 + 20 + 60)
=
= 16.4 m/s
6.1
i.e.,

12s time

vmax
(Q a = 4 m/s2 )
4
i.e.,
vmax = 16 m/s
Displacement of particle in 12 seconds
= Area under v-t graph
v
= 12 max
2
12 16
=
2
= 96 m
Average velocity
Displacement (96 m) at 12 s
=
Time (12 s)

1
( 10) t2
2

8s

4=

= + 8 m/s

Motion in One Dimension


(b) As the particle did not return back
distance travelled in 12 s
= Displacement at 12 s
Average speed = 8 m/s.
21
6. (a) Radius ( R) of circle =
m
22
Circumference of circle = 2R
22 21
m
=2

7
22
= 6m
Speed (v) of particle = 1 m/s
Distance moved by particle in 2 s = 2 m
Thus, angle through which the particle
moved
2
2
= 2 =
= 120
6
3
Magnitude of Average velocity
Magnitude of displacement
=
Time ( = 2 s)

7. Position vector at t = 0 s

r1 = (1 ^i + 2 ^j) m
Position vector at t = 4 s

r2 = (6 ^i + 4 ^j) m
(a) Displacement from t = 0 s to t = 4 s

= (6 ^i + 4 ^j) (1 ^i + 2 ^j)
= (5 ^i + 2 ^j) m
Average velocity =

(b) Average acceleration


Final velocity Initial velocity
=
4s

120
O

AB 2R sin 60
=
2
2
3
=R
2
21 3 21 3
m/s
=
=
22 2
44
(b) Magnitude of average acceleration

vB v A
=

2s

120
2 v sin
2
=
2
=

[Q

|v A |= |v B |= v = 1 m/s)
= v sin 60
3
m/s2
=
2

(5 ^i + 2 ^j) m
4s

= (125
. ^i + 05
. ^j) m/s

(2 ^i + 10 ^j) ( 4 ^i + 6 ^j)
4

2 ^i + 4 ^j
4

A(t = 0 s)

(t = 2s)

vB

= ( r 2 r1 )

vA

| 21

= ( 05
. ^i + ^j) m/s2
(c) We cannot find the average speed as
the actual path followed by the particle
is not known.
Uniform acceleration
(a) One dimensional motion
8. If at time t the vertical displacement

between A and B is 10 m
1 2 1
gt g( t 1)2 = 10
2
2
or
or
or

t2 ( t 1)2 = 2
t2 ( t2 2t + 1) = 2
2t = 3
t = 1.5 s

9. The two bodies will meet if

Displacement of
Displacement of
first after attaining = second before
attaining highest
highest point
point

22 | Mechanics-1
v0 t
or
or

or

10.

T
O

1 2
1
gt = v0 ( t t0 ) g( t t0 )2
2
2
1
2
0 = v0 t0 g( t0 2t0 t)
2
1 2
gt0 t = gt0 + v0 t0
2
1
gt0 + v0
t= 2
g
t0 v0
=
+
2
g
1
5 = gt2
2
t = 1s
5m
A
25 m

H W

or
or
t=

2t0 ( 2t0 )2 41( t20 )

2
2t0 2t0 2
=
2
= t0 + t0 2 (
absurd)
= (1 + 2) t0
= 2141
.
t0

Time interval
1s

ive

t second for A

12.
(t1) second for B

v=0
A

5 m/s
H

For A
H = 0. t +
H=

1 2
gt
2

1 2
gt
2

15m

(i)

1
(ii)
g( t 1)2
2
1 2 1
or
gt g( t 1)2 = 25
2
2
[Substituting value of H from Eq. (i)]
1
g[ t2 ( t 1)2 ] = 25
2
t2 ( t2 2t + 1) = 5
2t 1 = 5

t=3s
Substituting t = 3 s in Eq. (i)
1
H = 10 32 = 45 m
2
1 2
11. s = at0 Forward motion
2
H 25 =

Backward motion

(a) For H
02 = u2 + 2( 10) H
i.e.,
20H = 325
or
H = 1625
. m

For B

v = at0

being

52 = u2 + 2 ( 10)15
u2 = 325

or

sign

From the begining of the motion the point


mass will return to the initial position
after time 3.141t0 .

B u=0

1
( a) t2
2
1
1
at20 = ( at0 ) t at2
2
2
2
2
t0 = 2t0 t t
2
t 2t0 t t20 = 0
s = ( at0 ) t +

(b) For t
0 = 325 + ( 10) t
325
t=
10
= 1. 8 s

13.

At rest

a
x

15 m/s

60 m
6.0 s

(a)
(i)
152 = u2 + 2a 60
and
(ii)
15 = u + a 6
Substituting the value of 6a from Eq. (ii)
in Eq. (i)
225 = u2 + 20 (15 u)

Motion in One Dimension


2

i.e.,

u 20 u + 75 = 0
(u 15)(u 5) = 0

u = 5 m/s
(15 m/s being not possible)
(b) Using Eq. (ii)
5
a = m / s2
3
(c)
u2 = 02 + 2ax
u2
(5)2
i.e.,
x=
=
2a 2 5
3
= 7.5 m
1
(d) s = at2
2
1 5
= t2
2 3
5
= t2
6
t(s )

v (m/s)

5/6

7.5

Starts
A

(iii)
9

12

30
15
t (s)

12

Differentiating Eq. (iii) w.r.t. time t


5
v= t
3
t(s )

12

v (m/s)

10

15

20

v (m/s)
20
15
10
5
6

12

Stops
B
B

s2

Journey A to P
vmax = 0 + xt1
and
v2max = 2xs1
v

t1 = max
x

60

t2

4 min

30 67.5 120

t1
4 km

90

a = y

s1

120 s (m)

a=+x

14.

| 23

t (s)

(i)
(ii)

Journey P to B
0 = vmax + ( y) t2
2
and
vmax = 2 ys2
v

t2 = max
y
vmax vmax

+
= t1 + t2 = 4
x
y
1 1
vmax + = 4
x y
From Eq. (ii) and Eq. (iv)
v2
v2
s1 + s2 = max + max
2x
2y
v2max 1 1
or
4=
+
2 x y
or

(iii)
(iv)

(v)

(vi)

Dividing Eq. (vi) by Eq. (v)


vmax = 2
Substituting the value of vmax in Eq. (v)
1 1
(Proved)
+ =2
x y
15. Let acceleration of the particle be a using
2m
4
0
t = 10 s t = 6 s
v=0

t=0s

+ive
x-axis

v = u + at
0 = u + a6
u

a=
6
(a) At t = 10 s, s = 2 m
1
2 = u 10 + a 102
2
or
2 = ( 6a) 10 + 50a
or
10a = 2
or
a = 0.2 m/s 2

24 | Mechanics-1

(b) v(at t = 10 s) = u + a 10
= 6a + 10a
= 4a
= 08
. m/s

u = 0 m/s

a 2 = 4 ^j m/s2 (t = 2 s to t = 4 s) t2 = 2 s
(a) Velocity

a=

16.

= 0 + (2 ^i) 2

F 10 N north
=
m
2 kg

= 4 ^i

= 5 m/s , north
^

= 5 j m/s

v2 = (8 ^i 8 ^j ) m/s

or

= 10 i m/s

(b) Co-ordinate of particle

1
s 1 = s 0 + u t1 +
a 1 t12
2

using v = u + a t

v = 10 ^i + (5 ^j 2)
^

= 4 ^i + (2 ^i 4 ^j ) 2

u = 10 m/s, east

v2 = v1 + (a 1 + a 2 )t2

v1 = u + a 1 t1

(c) Two or three dimensional motion

= (2 ^i + 4 ^j) + (0)(2) +

= 10 i + 10 j

1 ^ 2
(2 i) 2
2

= 2 ^i + 4 ^j + 4 ^i

|v|= 10 2 m/s

= 6 ^i + 4 ^j

v = 10 2, north-east

1
using s = u t + a t2
2
1
^
= (10 i 2) + (5 ^j)22
2

1
( a1 + a2 ) t22
2
1
= 6 ^i + 4 ^j + ( 4 ^i) 2 + (2 ^i 4 ^j) 22
2

s 2 = s 1 + v1 t2 +

= 6 ^i + 4 ^j + 8 ^i + 4 ^i 8 ^j

= (20 ^i + 10 ^j) m

= 18 ^i 4 ^j

|s|= 202 + 102

Co-ordinate of the particle


[18 m , 4 m ]

= 10 5 m
20
cot =
=2
10
= cot1 2

18. u = (2 i 4 j) m/s, s 0 = 0 m

a = ( 4 ^i + ^j) m/s2

North

(a) Velocity

East

s = 10 5 m at cot1(2) from east to north.

17. s 0 = (2 i + 4 j) m

= (2 ^i 4 ^j) + ( 4 ^i + ^j)2

^j
O ^
i

v=u+at

a 1 = 2 ^i m/s2 (t = 0 s to t = 2 s) t1 = 2 s

= (10 ^i 2 ^j ) m/s
(b) Co-ordinates of the particle

1
s = s 0 + u t + a t2
2

1
^
= 0 + (2 i 4 ^j) + ( 4 ^i + $j )22
2

Motion in One Dimension


= 10 ^i 2 ^j

Co-ordinates of particle would be


[10 m , 2 m ]
u = 8 ^j m/s

|a av|= 52 + 202

a = ( 4 ^i + 2 ^j) m/s2

= 206
. m/s
20
tan =
5
i.e., = tan1( 4)
Non uniform acceleration.

s0 = 0

s = 29 ^i + n ^j

(a) s = s 0 + u t +

1 2
at
2

21. x = 2 + t2 + 2t 3

1 ^
( 4 i + 2 ^j) t2
2

29 ^i + n ^j = 0 + (8 ^j) t +

(a) At t = 0, x = 2 m
(b)

29 ^i + n ^j = 8 ^j t + 2 ^i t2 + ^j t2
Comparing the coefficients of ^i and ^j
29 = 2t2
n = 8 t + t2
29
t=
= 3.807 s
2

and

Substituting value of t in Eq. (ii)


n = 8 3.807 + (3.807)2
= 44.95
29
(b) Speed at t =
s
2

(i)
(ii)

d2 x
= 2 + 12t
dt2
( a) t = 2 s = 2 + (12 2) = 26 m/s2
dv
22.
a= v
dx
= (10) 3
= 30 m/s2
23. s = t 3 9 t2 15t

ds
= 3 t2 18 t 15
dt
dv
ie,
a=
= 6t 18
dt
Acceleration (a) in the interval 0 t 10 s
will be maximum at t = 10 s
a(at t = 10 s) = (6 10) 18
= 42 m/s2

= 8 ^j + ( 4 ^i + 2 ^j)(3.807)
= (2 3.807) ^i + {( 4 3.807) + 8 } ^j
= (7.614 ^i + 23.228 ^j) m/s

speed = |v|= (7.614)2 + (23.228)2


= 24.44 m / s
20. s 0 = 5 i m

Average velocity v av
^

s s0
=
002
.
^

(51
. i + 0.4 j) (5 i)
002
.

v=

a = 3 2t
dv
= 3 2t
dt

24.

at t = 002
. s, s = (51
. ^i + 0.4 ^j) m

dx
= 2t + 6t2
dt
dx
= 0 m/s

dt t = 0 s

(c)

v=u+at

01
. ^i + 0.4 ^j
002
.

= 5 ^i + 20 ^j m/s

19.

| 25

or

dv = (3 2t) dt

or

v = 3 t t2 + c

or

v = 3 t t2 + v0 [as at t = 0, v = v0 ]

or
or

(3 t t2 + v0 ) dt
3 t2 t 3
s=

+ v0 t
2
3

ds =

26 | Mechanics-1
(a) Displacement at = Displacement at
t = 0s
t = 5s
3.52 53
0=

+ v0 5
2
3
s

v0 = 5/6 = 0833
.
(b) v = 3 t t2 + v0
Velocity at t ( = 500
. s)
= 3.5 52 + 0833
.
= 15 25 + 833
= 9.167 m/s
25. v = 3 t2 6t
2

ds = 3(3 t

6t) dt
s = t 3 t2 + c
s = t 3 3 t2 = t2 ( t 3)
(3.5)2 (3.5 3)
(a) Average velocity =
3.5
= 3.5 05
.
= 175
. m/s
(b) Distance covered

or
or

Now, at t = 2 s, v = 6 m/s
62

= ( 4 2) + C
2
Thus, C = 10.
v2
i.e.,
(ii)
= 4 t + 10
2
at
t = 3 s,
v2
= ( 4 3) + 10
2
v2 = 44
v = 44 = 2 11 m/s
Substituting above found value of v in Eq.
(i),
2 11 a = 4
2
i.e.,
a=
11
= 0.603 m/s 2
27. According to question, the velocity of the

particle varies as shown in figure.


v
20 m/s

1
2

3 3.5

3.5

0 (6t 3 t ) dt + 2

(3 t2 6r) dt

= [3 t2 t 3 ]20 + [ t 3 3 t2 ] 23.5
= 3(2)2 (2) 3 + (3.5) 3 3(3.5)2 (2) 3 + 3(2)2
= 12 8 + 42875
.
3675
. 8 + 12
= 14.125 m
14.125
(c) Average speed =
3.5
= 4.036 m/s
4
26.
v=
a
i.e.,
(i)
va = 4
or
v dv2 = 4 dt
v
or
= 4t + C
2

30 m/s s

v
s
+
=1
20 30
or
3v + 2s = 60
or
(i)
3v = 60 2s
Differentiating above equation w.r.t. time t
dv
ds
3
= 2
dt
dt
or
3a = 2v
60 2 s
or
3 a = 2
[using Eq. (i)]
3
2
or
a = (60 2 s)
9
2
[at s = 15 m]
= (60 2 15)
9
20
m/s2
=
3

Motion in One Dimension


1
S1 = gt 2
2
1 h
= g
2 g
h
=
2
Assertion is true.
Reason is also true as assertion is based on
this.

| 39

12. Please see answer to question no. 2 of


objective questions (level 1).
um
T2 =
( mg + f )( mg f )
Now, if masses of the bodies are different,
the value of T2 (time to reach earth) will be
different if f (air resistance) is same.
Thus, Assertion is false further Reason is
correct as explained above.

Objective Questions (Level 2)


Single Correct Option
1. Applying sine formula in AOB

B AO = 180 (90 + ) ( 37 )

= 90 + 37
= 127 ( + Q )

v
Nm

vr

Comparing Eqs. (i) and (ii)


sin ( 37 ) sin [127 ( + )]
=
cos 37
sin ( 37 )
sin cos 37 cos sin 37
cos 37
sin 127 cos ( + ) + cos 127 sin ( + )
=
sin cos 37 cos sin 37

37

B
D

vm

37

AO
OB
=
sin (96 + 37 ) sin ( 37 )
vr
vm
=
cos 37 sin ( 37 )

(i)

Applying sine formula in


A'

vr

vm

37

B'
37

AOB
AO
OB
=
sin ( 37 ) sin [( + )]
vr
vm
(ii)
=
sin ( 37 ) sin [127 ( + )]

sin cos tan 37


4
3
cos ( + ) + sin ( + )
5
5
=
4
3
sin cos
5
5
4 cos ( + ) + 3 sin ( + )
3
or sin cos =
4 sin 3 cos
4
3
or sin cos
4
4 [cos cos sin sin ] + 3 [sin cos + cos sin ]
=
4 sin 3 cos
3
or sin cos
4
4 [cos sin tan ] + 3 [sin + cos tan ]
=
4 tan 3
3
or sin cos
4
7
7
4 [cos sin ] + 3 [sin + cos ]
8
8
=
7
4 3
8
3
or sin cos
4
4 [ 8 cos 7 sin ] + 3 [ 8 sin + 7 cos ]
=
4

40 | Mechanics-1
or 4 sin 3 cos
= 32 cos 28 sin + 24 sin + 21 cos
50
7

or 32 cos + 21 cos + 3 cos


= 28 sin 24 sin + 4 sin
or
56 cos = 8 sin
7
tan =
1
From Eq. (i)
vr
vm
=
cos 37 sin ( 37 )
vm cos 37
vr =

sin cos 37 cos sin 37


vm cos 37
=
cos cos 37 [tan tan 37 ]
vm
=
3
cos 7

4
4 vm
=

25 cos
4 5
=
50
25 1
= 32
Option (b) is correct.
dv
2.
= 4v + 8
dt
a = 4v + 8
At the time the body acquires terminal speed
its acceleration (a) must be zero.
Thus
4v + 8 = 0
v = 2 m/s
Terminal speed = 2 m/s.
Option (b) is correct.
3. Particles will collide if

v1

|v1

v2
r
= 2

v 2| |r2
^

r1

r1|
^

v1 v 2

|v1

v 2|

^
r2 r1 = r2 = 30 i

|r2 r1| = 30

r2 r1
^
=i

|r2 r1|

|v1 v 2| = 5 2

v1 v 2

|v1 v 2|

Option (c) is correct.


4. Displacement along y-axis at time t
Y
v0

P
a = 2t
u=0

a =2t
dvy
=2t
dt
or
dvy =2 2t dt
or
vy = t + C1
At t = 0, vy = 0 (given)
C1 = 0
vy = t 2

dy
= t2

dt
2
or
dy = t + C2

t3
+ C2
2
At t = 0, Y = 0 (given)
t3
Y=

3
Displacement along x-axis at time t :
or

or

y=

x = v0t
x
t=
v0

Substituting above value of t in Eq. (i)


^

Y=

v1 v 2 = (5 i + 10 j + 5 k ) (10 i + 5 j + 5 k )
= 5i + 5 j

1 ^
1 ^
i+
j
2
2

1 x

3 v0

Option (a) is correct.


5. t = x 2 + x
Differentiating w.r.t. time t

(i)

Motion in One Dimension

or
or

dx
dx
1 = 2x
+
dt
dt
v ( + 2 x ) = 1
v = ( + 2 x )1
dv
1
dx
=
2
dt
dt
( + 2 x )2
2
a=
v
( + 2 x )2
2
=
v
( + 2 x )2

7. Force = kx

Acceleration =

or

Thus

6. f = a bx
dv
= a bx
dx
or
v dv2 = (a bx )2dx
v
bx
or
= ax
+C
2
2
As at x = 0 car is at rest C = 0
v2
bx 2
= ax

2
2
For v to be maximum.
d
bx 2
ax
=0
dx
2

(i)

a
in Eq. (i),
b
2
vmax
a b a 2
= a
2
b 2 b
2
a
=
2b
a
vmax =
b

Car will come to rest when


bx 2
ax
=0
2
2a
x=
b
Distance between two stations =
Option (a) is correct.

k a3
m 3

v2
kx 2
ka 3
=
+
2
m 3 m 3
2 ka 3
3m

Option (d) is correct.


8. vx = 4 + 4t

Substituting x =

C=

v=

a bx = 0
a
x=
b

or

v2
k x3
=
+C
2
m 3

Velocity at x = 0

i.e.,

kx 2
dx
m

Now at x = a, v = 0

= (2 v2 ) v
= 2 v3
Retardation = 2 v3
Option (a) is correct.

k2 2
x
2m

dv
kx 2
=
dx
m

v dv =

or

| 41

2a
b

and

vy = 4 t

dx
dy
= 4 + 4t and
= 4t
dt
dt
or
dx = (4 + 4t ) dt

and
dy = 4t dt 2
or
x = 4t + 2t + C1
C1 = 1

Thus
(i)
x = 4t + 2t 2 + 1
and
y = 2t 2 + C2
At
t =0x =1
and at t = 0, y = 2
C2 = 0

Thus
(ii)
y = 2t 2 + 2
Substituting value of t in Eq. (i) in terms of y
using Eq. (ii) we wont get relationship as
mentioned in option (a), (b) or (c).
Option (d) is correct.
1
9.
a = gt 2
2
= 4 tan 120
= 4 [ tan 60 ]
= 4 3 m/s 2
Magnitude of acceleration = 4 3 ms 2.
Option (a) is correct.

42 | Mechanics-1
10.

1 2
gt
2
1
x1 + x2 = g (2t )2
2
1
1
x2 = g 4t 2 gt 2

2
2
3 2
x2 = gt
2
x2 x1 = gt 2
x2 x1

t=
g

or

14. Distance =

1 2

t
2 +

0=

1 1 4 2

t
2 1 + 4

t = 500 s

= 22.36 s
Option (a) is correct.
15. Let BC = t

y2 + x 2 = l2
dy
dx
2y
+ 2x
=0
dt
dt
dy
x dx
=
dt
y dt

v (m/s)
24

x
dx
x tan 30 dt
= 2 3 m/s
=

12. Maximum separation (xmax ) between the


police and the thief will be at time t (shown
in figure)
Police

108 km/h
90 km/h

v
Thief

t
7s

time (t)

Let OA = t
AB = 56 t

1
1032 = [56 + (56 t )] 24
2
t = 20 s

24 m / s
maximum acceleration =
20 s
= 1.2 m/s 2

16. From OAB


P

(4

)m
2t
0.

13. If meeting time is t


2v
sin = u sin 30
3
3
i.e.,
sin =
4

Option (b) is correct.

2t
0.

v 90 kmh
=
a
5 ms 2
90 1000
s
=

5 3600
i.e.,
t =7s
1
1
xmax = [90 kmh 7 s ] 5 s 90 kmh 1
2
90 1000 9
=
m
3600
2
= 112.5 m
Option (a) is correct.
t 2 =

C t (s)

24
=4
t
t = 6 s
OB = 50 s

Option (c) is correct.

2s

3
4

Option (c) is correct.

Option (a) is correct.


11.

= sin 1

or

x1 =

0.2t
Q

cos AOB =

60
B

(3 0.2t) m

OA 2 + OB2 AB2
2 OA OB

Motion in One Dimension


2

OA + OB AB = OA OB
AB2 = OA 2 + OB2 OA OB
= (4 0.2t )2 + (3 0.2t )2
+ (4 0.2t )(3 0.2t )
2
2
or
AB = 16 + 0.04t 1.6t + 9 + 0.04t 2
1.2t + 12 1.4t + 0.04t 2
2
2
or
AB = 0.12t 4.2t + 37
For AB to be minimum
0.24t 4.2 = 0
or
t = 17.5 s
2
( AB)min = 0.12(17.5)2 4.2 (17.5) + 37
= 36.75 73.5 + 37
( AB)2min = 0.75 m 2
= 7500 cm 2
= 50 3 cm
Option (d) is correct.

i.e.,

17. Velocity of ball w.r.t. elevator = 15 m/s


Acceleration of ball w.r.t. elevator
Max. height attained by ball

H
15 m/s = vBE

5 m/s2

2m

10 m/s

Now
v = u + 2as
02 = (25)2 + 2( 10) H

or
H = 31.25 m
Maximum height by ball as measured from
ground = 31.25 + 2 + 50
= 83.25 m
Option (c) is correct.
19. Displacement of ball w.r.t. ground during its
flight = H = 31.25 m

Option (d) is correct.


20. Displacement of ball w.r.t. floor of elevator
at time t
1
s = 15 t + ( 15) t 2 + 2
2
s will be maximum, when
ds
=0
dt
i.e.,
15 15t = 0
i.e.,
t = 1s
1
smax = (15 1) + ( 10)(1)2 + 2
2
= 12 m
Option (a) is correct.
21. Let the particles meet at time t
i.e., displacement of the particles are equal
and which is possible when
vB

B'

v
50 m

A uA
C
P

P'

B uB

= ( 10) ( + 5) = 15 m/s 2
Final displacement of ball w.r.t. elevator
= 2m
1
2 = 15 t + ( 15) t 2

2
i.e.,
15t 2 30t 4 = 0
30 + 900 + 4 4 15
t=

2 15
= 2.13 s
Option (a) is correct.
18. Velocity of ball w.r.t. ground (vBE )
= (15 + 10) m/s
= 25 m/s

| 43

vA
O

A'
R
Q
time (s)

Area of ACB = Area of A B C


[Area OBCA O being common]
or Area of APC = Area A P C
1
1
AP PC = A P P C
2
2
or
AP PC = A P P C
or
( PC tan ) PC = ( P C tan ) P C
or
PC = P C

44 | Mechanics-1
23. u A = 6 ms 1, u B = 12 ms 1
and at t = 4 s common velocity = 8 ms 1
To find velocity of A at t = 10 s
vA 8 u A 8
=
10 4
4
vA 8 6 8
or
=
6
4
vA = 5 ms 1

OR = PC + P C = 2 PC = 8 s.

Option (c) is correct.


22. Area of A B O = Area of ABO
1
1
(vB vA ) 4 = (u A u B ) 4

2
2
i.e.,
vB vA = u A u B
= 5 15 = 10
vA vB = 10 ms 1

Option (b) is correct.

Option (d) is correct.

More than One Correct Options


a = v1/ 2
dv
i.e.,
= v1/ 2
dt
or v1/ 2 dv = dt
2 v1/ 2 = t + C1
Now, at t = 0, v = v0
C1 = 2 v10/ 2

2 v1/ 2 = t + 2 v10/ 2

i.e., the particle will stop at


2 v1/ 2
t= 0

Option (a) is correct.


Option (b) is incorrect.
From Eq. (i),
dv
v
= v1/ 2
dt

1. Q

or

1/ 2

dv = dx

2 3/ 2
v = + C2
3
2
As at x = 0, v = v0, C2 = v03/ 2
3
2 3/ 2
2
v = x + v03/ 2

3
3
or

i.e., when the particle stops


2 3/ 2
x=
v0
3
Option (d) is correct.
Option (c) is incorrect.
2. a = 0.5 t (m/s 2)

or

dv
t
=
dt
2
1
dv
=

t dt

(i)

v=

t2
+ C1
4

At t = 0, v = 16 m/s
t2
+ 16
4
From above relation v is zero at
t = 8s
Options (a) is correct.
From relation (i),
ds
t2
=
+ 16
dt
4
t2

ds
=

4 + 16 dt
v=

i.e.,

s=
s=

(i)

t3
+ 16t + C2
12

t3
+ 16t
12

[As at t = 0, s = 0]

At t = 4 s
s = 58.67 m
Option (b) is correct.
The particle returns back at t = 8 s
From relation (ii)
83
s8 =
+ (16 8) = 85.33 m
12
103
s10 =
+ (16 10) = 76.66 m
12
Distance travelled in 10 s
= s8 + ( s8 s10 )
= (2 85.33 ) 76.66
= 94 m
Option (c) is correct.
Velocity of particle at t = 10 s

Motion in One Dimension


2

(10)
+ 16
4
= 25 + 16
= 9 m/s
Speed of particle at t = 10 s is 9 m/s.
Option (d) is correct.
v10 =

3. |v|is scalar.
Option (a) is incorrect.

Position of A relative to B :

S AB = S A S B
^

v = (Let)
Rest

t1
Rest

East
X (km)

South

vA

= 20 j kmh 1

vB

= (40 cos 37 ) i + (40 sin 37 ) j kmh 1

= (32 i + 24 j) kmh 1

vAB = vA vB
^

= ( 20 j) (32 i + 24 j)
^

= 32 i 44 j
Option (a) is correct.
Option (c) is incorrect.
At any time

SA = 3 i + 4 j + ( 20 j) t

v' = 2v

2a1

t3
a2

Average
velocity = v2

S2

20 km/h

Rest

Average
velocity = v1

Case II.

A (3, 4)

a2

v = a1t1
v = a 2t2
1
1
s1 = a1t12 + a 2t22
2
2

d v^ ^
=aa
dt
Option (d) is incorrect.

37

t2

a1
S1

t1

|v|

h
m/
0k

Option (b) is correct.


Option (d) is incorrect.

is v^ (unit vector)

4. North
Y (km)

= (3 32t ) i + (4 44t ) j

(by definition).

=a

dt
Option (b) is correct.
v2 is scalar.
Option (c) is incorrect.
v

= (3 i + 4 j 20t j) (32t i + 24t j)

5. Case I.

dv

| 45

S B = (32 i + 24 j) t

v = 2a1t1
= 2v
2v = a 2t3
2 ( a 2t2 ) = a 2t3
t3 = 2t2

1
1
s2 = (2a1 ) t12 + a 2t32
2
2
1
2
= a1t1 + a 2(2 t2 )2
2
= a1t12 + 2 a 2t22
2s2 = a1t12 + a 2t22
s1 > 2s1

4s1 = 2a1t12 + 2a 2t22


s2 < 4s1

Combining Eqs. (i) and (ii),


2s1 < s2 < 4s1
Option (d) is correct.
s1
In Case I. vav =
t1 + t2
1
1
a1t12 + a 2t22
2
2
i.e.,
v1 =
t1 + t2

(i)
(ii)

46 | Mechanics-1
( a1t1 ) t1 + ( a 2t2 ) t2
2 ( t1 + t2 )
vt1 + vt2 v
=
=
2 ( t1 + t2 ) 2
=

a=

8.

5a = s + 30
dv
5 v
= s + 30
ds

In Case II.
vav =
i.e.,

s2
t1 + t3

a1t12 + 2a 2t22
t1 + t3
( a1t1 ) t1 + a 2t2 (2 t2 )
=
t1 + 2 t2
vt + v (2 t2 )
= 1
t1 + 2t2

v2 =

= v = 2v1
Option (a) is correct.
Option (b) is incorrect.
6. If the particles initial velocity is + ive and
has some constant ive acceleration the
particle will stop somewhere and then
return back to have zero displacement at
same time t ( > 0).
Also if particles initial velocity is ive and
has some constant + ive acceleration the
particle will stop somewhere and then
return back to have zero displacement at
same time t ( > 0).
Options (b) and (c) are correct.
and options (a) and (d) are incorrect.
7. F = t

ma = t

or
(i)
a=
t
m
Graph between a (acceleration) and time ( t )
will be as curve 1.
Option (a) is correct.
From equation
dv
= t
dt m

dv = m t dt
t2
i.e.,
v= + c
m 2
Graph between velocity (v) and time ( t ) will
be as curve 2.
Option (b) is correct.

6
s+6
30

or

or
or

5v dv = ( s + 30) ds
5v2
s2
=
+ 30s + c1
2
2
5 2
S2
v =
+ 30s
2
2

(i)

[C1 = 0 as at s = 0 particle is at rest]


Substituting s = 10 m in Eq. (i),
5 2
(10)2
v =
+ 300
2
2
i.e.,

v = 10 m/s

Option (b) is correct.


From Eq. (i) v to be maximum
s + 30 = 0
s = 30
2

or

5 2
(30)
vmax =
+ (30 30)
2
2
5 2
vmax = 450
2

or
vmax = 180 m/s
Option (c) is correct.
9. If particles path is
(i) straight with backward motion
(ii) not straight somewhere.
Distance moved will be greater than the
modulus of displacement

|v av | < vav

Option (a) is correct.


If particle returns to its initial position, the

value of v av will be zero while its average


speed (vav ) will not be zero.
Option (c) is correct.
1
10. If u = 0, v = at and s = at 2
2
v-t graph will be as shown in (a).
s-t graph will be as shown in (d).

Motion in One Dimension


t = t1 + t2 + t3
2l
l
2l
= 1 + 2 + 3
v
v
v
l1 l2 l3 l1 l3
= +
+
+ +
v
v
v
v
v
l 1
= + [ l1 + l2 ]
v v
l 1 1
1
= + t12 + t22

v v 2
2

11. Swimmer will reach point B


B
b

v cos

v sin

if
v sin = u
i.e.,
v>u
Option (c) is correct.
Time to cross the river
b
b
t=
=
2
v cos
v u2

1 t12 1 t22
l
+ v
+

2
v
2 v2
2 v
1 t12 1 t22
l
= + v
+

2 2
v
2 2t22
2 t1
l v 1 1
= + +
v 2
=

[as v sin = u]

Option (b) is correct.


b
v (cos )max
b
=
v
Option (a) is correct.
tmin =

12. At time T, particles velocity change from


ive to + ive.
Option (a) is correct.
As slope v-t is same throughout, particles
acceleration is constant.
Option (b) is correct.
As net area under the curve is zero.

vT vT
Anet = 2 + 2 , displacement of

particle at time 2T is zero.


Option (c) is correct.

Initial speed = |v| = v

Final speed = |v| = v


Option (d) is correct.
13.
t1

Velocity O
Accel :

()
l1

t2

l2

v
( )

t3

Option (b) is correct.


For t to be minimum.
d l v 1 1
+ + =0
dv v 2

or
or

v = t3

1 2
1
t1 , l2 = vt2, l3 = t32
2
2
1
1
= vt1 = vt3
2
2

l1 =

l
1 1 1
+ + =0
v2 2
l +
=
2
v2
2 l
v=
+

Option (d) is correct.


14. x = t 2 and y = t 3 2 t
dx
dy
and
= 2t
= 3 t2 2
dt
dt
dx
dy
At t = 0,
= 0 but
= 2
dt
dt
at t = 0 particle is moving parallel to y-axis.
Option (a) is correct.
Option (d) is incorrect.
At t = 0,

l3

v = t1

| 47

and
At t = 0,
i.e.,

v = 2 j

d 2x
=2
dt 2
d2 y
= 6t
dt 2

a =2i

va

Option (b) is correct.

48 | Mechanics-1
At t =

2
:
3
dy
2
dx
2
but
= 3 2
=2
3
dt
dt
3

Speed (v) at

= a ( S1 + S1 )
= a (25a t12 )
v = 5a t1

or
v = 10 m/s
Option (a) is correct.

t2

Time t to reach P from T


10 = 2 + at
8
t=

Particle is moving parallel to x-axis.


Option (c) is correct.
15.
Speed 2 m/s
X
Starting
point

S1

14 m/s
P
(Mid point)
of X and Y

t1
S2

S1 + S2
2

S1 + S2
:
2
S + S2
v2 = 2a 1

(i)

Time t to reach Y from P


14 = 10 + at
4
t =

a
Comparing Eq. (i) and Eq. (ii),
t = 2t
Option (c) is correct.

at1 = 2 and at2 = 14


t2 = 7t1
1
1
S1 + S2 = at12 + at22
2
2
1
2
= a ( t1 + t22 )
2
1
= a [ t12 + ( 7t1 )2 ]
2
= 25a t12

(ii)

Match the Columns


1. (a)

(b)
a (acceleration)

a (acceleration)
+

v (velocity)
+

t
Speed increasing

(a) (a) (r) (s).

v (velocity)

v (velocity)
t

v (velocity)
+

Speed decreasing

t
Speed decreasing

Speed increasing

(b) (b) (r) (s).


(c) s = kt 2
v=

ds
= 2kt
dt

Motion in One Dimension


v must be increasing with time.
speed must also be increasing with time.
(c) (p).
(d) Slope is + ive and decreasing.
Velocity must be decreasing with time.
speed must also be decreasing with time.
(d) (q).

2. (a) v = 2 i

Initial speed = |v| = 2

a = 4 j

Velocity ( ) at time t

=v + at
^

= 2 i 4t j
Speed at time t = || = ( 2)2 + ( 4 t )2
At t > 0

|| > |v|
Speed increasing.
(a) (p).

(b) v = 2 i

|v| = 2

=2i + 2ti + 2t j

|| = (2 + 2 t ) + (2 t )

As t > 0,

|| > |v|
(a) (p).

(c) v = 2 i and a = + 2 i

Using = v + a t

= 2i + 2ti

or

= (2 t 2) i

||= |2 t 2|

At t = 0 s

Speed = || = 2 m/s

t =1s

|| = 0 m/s

t =2s

|| = 2 m/s

t =3 s

|| = 4 m/s

a = 2i + 2 j

=v + at
^

=2i 2ti + 2t j
= (2 2 t ) i + 2 t j

|| = (2 2 t )2 + (2 t )2

At t = 0
1
t=
2

|| = 2

t =1

|| = 2

t =2

|| = 20

|| = 2

3. (a) From A to B, v is increasing and area is


+ ive.
(a) (p).

(b) From B to C, v is increasing and area is


+ ive.
(b) (p).

(c) From C to D, v is decreasing while area is


+ ive.
(c) (q).

(d) From D to E, v is ive and |v|is


increasing, area is ive.
(d) (r).

Using = v + a t

(d) v = 2 i and

a =2i + 2 j

(c) (s).

(d) (s).

| 49

4. (a) Displacement = Area


10 4
=
= 20 unit
2
20
= + 5 unit
Average velocity = +
4
i.e.,
(a) (r).
1 5
(b) Displacement = + 20
= + 17.5 unit
2
+ 17.5 unit
Average velocity =
3
= 5.83 unit
(b) (s).

2 ( 10)
(c) Displacement = + 20 +

2
= + 10 unit
+ 10
Average velocity =
6
i.e.,
(c) (s).

50 | Mechanics-1
(d) Rate of change of velocity at t = 4 s
0 10

a=
= 5
4 2

i.e.,

Rate of change of speed at


t = 4 s would be 5.
i.e.,
(d) (r).
5. (a) x = 20 + 5t 2
0 = 20 + 5t 2
t =2s

i.e.,

(a) (r).

(b) x = 20 + 5t

dx
= 10 t
dt
d 2x
= 10
dt 2
Velocity will be numerically
acceleration at t = 1 s.
Thus

equal

to

(b) (q).

t =1s
Y = 4 4t + t 2
dy
= 4 + 2t
dt
= 4 + (2 1)
= 2 unit

(a) (q).
(b) Y = 4 4t + t 2
0 = 4 4t + t 2 [For particle to cross x-axis]
i.e.,
t =2s
x = 1 2t + t 2
dy
= 2 + 2t
dt
[at t = 2 s]
= 2 + (2 2)
= + 2 unit
(b) (p).
dx
(c)
= 2 unit
dt t = 0 s

(c) At t = 0 s

x = 20 m

dy
and
= 4 unit
dt t = 0 s

t =1s

x = 15 m

Initial velocity of particle

t =2s

x =0m

t =3 s

x = + 25 m

Particle is always moving along + ive x-axis.


(c) (s).
dx
(d) v =
= 10 t is zero at t = 0 s.
dt
(d) (p).

6. (a) x = 1 2t + t 2
0 = 1 2t + t 2 [for particle to cross y-axis]

[at t = 1 s]

= ( 2 )2 + ( 4 )2
i.e.,
d 2x
(d) 2 = 2
dt

= 2 5 unit
(c) (s).

d2 y
=2
dt 2
Initial acceleration of particle = 2 2 unit
i.e.,
(d) (s).
and

Motion in One Dimension

| 27

Graphy
28. OA : slope is + ive and increasing.

30. Case I

s
C
B

m = tan

s0
t

v0
a

velocity is + ive and acceleration is + ive.


AB : slope is + ive and constant
velocity is +ive and acceleration is zero.
BC : sope is + ive and decreases.
velocity is + ive and increasing.
CD : slope is ive and increasing
velocity is ive and acceleration is ive.
29.
v
v
M2

M1

v = mt v0 (st-line)

ds = (mt2 v0 ) dt

t
v0 t + s0
2
1
or
s = s0 v0 t + at2
2
dv
Further, a =
= tan
dt
i.e.,

(Parabola)

s= m

(Q m = a)

As for 0 < 90

tan is + ive, a is + ive.

v
M4

a = tan

Case II

M3
v
t

s0
t

In M1 and M 3 : 0 < 90
slope is + ive i.e., acceleration is + ive.
In M2 and M4 : 90 < 180
slope is ive i.e., acceleration is ive.
(a) M1 : Magnitude of velocity is increasing.
M2 : Magnitude of velocity is decreasing.
M 3 : Magnitude of velocity is increasing.
M4 : Magnitude of velocity is decreasing.
Ans : M1 and M 3.
(b) P M1
Q M2
R M3
S M4

v = k (constant) (st line


time-axis)

ds = k dt
s = kt + s0
dv
Further,
a=
=0
dt

parallel

to

28 | Mechanics-1
Case III.

Time-acceleration graph
s

v
v0

time

s0

0-2s
2-4s
4-6s
6-8s
8 - 10 s

slope

acceleration
5 m/s 2
0 m/s 2
5 m/s 2
5 m/s 2
+ 5 m/s 2

5
zero
5
5
+5

a
a

a = tan

v = v0 + mt

ds = (v0 + mt2) dt
t
i.e.,
s = v0 t + m + s0
2
1
or
s = s0 + v0 t at2
2
dv
Further, a =
= tan
dt
As for 90 < 180
tan is ive, a is ive.

(Q m = a)

Time-displacement graph
Time

at 0 s
2s
4s
6s
8s
10 s

Area
Initial
Net
under the Displacement displacement
graph
0m
10 m
30 m
40 m
30 m
20 m

10 m
10 m
10 m
10 m
10 m
10 m

10 m
0m
+ 20 m
+ 30 m
+ 20 m
+ 10 m

31. a = slope of v- t graph.


S = area under v t graph.
Corresponding graphs are drawn in the
answer sheet.
s Net area A1 A2
32. Average velocity = =
=
t
Time
t
d A1 + A2
Average speed = =
t
t
33. Average acceleration
v f vi 10 20
=
=
= 5 m / s2
time
6
v f vi = Area under a- t graph.
34. (a) Acceleration = slope of v- t graph.
35. (b) rf ri = s = area under v- t graph.
(c) Equations are written in answer sheet.

Relative Motion

+ 30 m

35. (a) Acceleration of 1 w.r.t. 2


+ 20 m
+ 10 m

2
10 m

10 t

10 t

= ( g) ( g)
= 0 m/s2
(b) Initial velocity of 2 w.r.t. 1
= ( +20) ( 5)
= 25 m/s
(c) Initial velocity of 1 w.r.t. 2
= ( 5) ( + 20)
= 25 m/s

Motion in One Dimension

= 25 m/s
(d) Initial relative displacement of 2 w.r.t. 1

S 0 = 20 m

1
Using S = S 0 + u rel t + a rel t2 ,
2
as at time t (= time of collision of the
particles) the relative displacement of 2

w.r.t. 1 will be zero (i.e., S = O)

1
O = ( 20) + ( 25)
2

t = 0.85 s

36. Let length of escalator L (= 15 m)

walking speed of man =

L
90

L
Speed of escalator =
60
Time taken by man walking on a moving
L
escalator =
L
L
+
90 60
= 36 s
Required time has been found without
using the actual length of the escalator.

37. S 0 = Initial displacement of elevator w.r.t.

ball = + 10 m.
Relative velocity of elevator w.r.t. ball
= (2) (18) = 16 m/s
Accelerator of elevator w.r.t. ball
= (0) ( 10)
= 10 m/s2

38.
3.5 m/s2

time = 0
2.2 m/s2

time =ts

A
A
x
60 m

1
(a) (2.2) t2 = 60
2

(i)

t = 7.39 s
1
2
(b) (3.5) t = 60 + x
2

x = 35.5 m
(c) At the time of overtaking
Speed of automobile = (3.5) (7.39)
= 25.85 m/s
Speed of truck = (2.2) (7.39)
= 16.25 m/s
39. Let, acceleration of lift = a (upward)

acceleration of thrown body w.r.t. lift


= ( g) ( + a)
If time of flight is t, using
vrel = urel + arel t

10 m

12 m

2 m/s
Elevator
5m

Using S = S 0 + urel t +

1
a t2
2 rel

(ii)

Dividing Eq. (ii) by Eq. (i),


60 + x 3.5
=
60
2.2
x 1.3
or
=
60 2.2

= ( g + a)

18 m/s

| 29

1
0 = ( + 10) + ( 16) t + ( +10) t2
2
or
5t2 16t 10 = 0
t = 3.65 s
Position of elevator when it meets ball
= 5 + (2 3.65)
= 12.30 m level

1
Velocity of 1 w.r.t. 2 at time t = s
2
1

= ( 25) + (0)
2

( v) = ( + u) + { ( a + g)} t
( a + g) t = 2 u
2u
or
a=
g
t

30 | Mechanics-1
A

40.
20 m

2
4
=
sin ( 45 ) sin 45

B
OAB = OA = 20 m

= sin 1
45

2 2

22 m/s
45
O

42. Let pilot heads point R to reach point Q


2

OB = (OA) + ( AB)

= (20)2 + (20)2
= 20 2 m
Speed along OB is 2 2 m/s
Time taken to reach B =

200 km/h

RQ
PR
200 t
=
500 t

sin =

41. In OPQ,
B
River flow

P
Q

45
O

or

OP = |v br |

East

P
South

= 10 s
A

1000 km

Wind

20 2 m
2 2 ms 1

North
Q

or

OP = |v r |
PQ
OP
=
sin POQ sin OPQ

= sin 1 (0.4)
( PR)2 = ( RQ)2 + ( PQ)2
(500 t)2 (200 t)2 = (1000)2
1000
t=
(500)2 (200)2
10
10
h
=
=
25 4
21

Objective Questions (Level 1)


Single Correct Option

mg + f
0=u
T1
m
um
T1 =
mg + f

1. As the packet is detached from rising

balloon its acceleration will be g in the


downward direction.
Option (b) is correct.
2. While going up :
Stops

Using v2 = u2 + 2as
mg + f
02 = u2 2
s
m
i.e.,

S=

While coming down

1 mg f 2

T2
2 m
mu2
1 mg f 2
=
T2
2 ( mg + f ) 2 m
s=

f (Air resistance)
mg

mu2
2( mg + f )

or

(i)

Motion in One Dimension

T2 =

um
( mg + f ) ( mg f )

(ii)

8. For first one second

2=

Using Eq. (i) and Eq. (ii)


T2
=
T1

mg + f
mg f

2
rad s 1
60
Speed of the tip of seconds hand

[Q v = r
and
v=
1 cm/s
30
r = 1 cm]
As in 15s the seconds hand rotates
through 90, the change in velocity of its
tip in 15 s will be
2
=v 2=
cm s 1
30
Option (d) is correct.
5+5
4. Average speed =
= 40 ms 1
5
5
+
30 60
=

Option (c) is correct.


5. Relative velocity of boat w.r.t. water

= (3 ^i + 4 ^j) ( 3 ^i 4 ^j) = 6 ^i + 8 ^j
Option (b) is correct.
(18 11) + ( 42 v)
6. 21 =
18 + 42
v = 25.29 m/s
8 t3
7.
x = 32t
3
dx
(i)
v=
= 32 8 t2
dt
Particle is at rest when
32 8 t2 = 0
i.e.,
t = 2s
Differentiating Eq. (i) w.r.t. time t
dv
a=
= 16 t
dt
a at time t = 2 s (when particle is at rest)
= (16) (2) = 32 m/s2
Option (b) is correct.

1
a 12
2

a = 4 m/s 2

Velocity at the end of next second


v = ( 4) (2)

T2 > T1
Option (c) is correct.
3. Angular speed () of seconds hand

| 31

= 8 m/s
Option (b) is correct.
9. x = 3 t + t 3

displacement at time t (= 1 s)
= 3(1) + (1) 3 = 2 m
and displacement at time t (= 3 s)
= 3(3) + (3) 3 = 18 m
And as such displacement in the time
interval (t = 1 s to t = 3 s)
= (18 m ) ( 2 m )
= + 20 m
Option (c) is correct.
10. Acceleration a = bt

dv
= bt
dt

or

dv = bt dt

1 2
bt + C
2
Now, at t = 0, v = v0

C = v0
1
i.e.,
v = bt2 + v0
2
ds
1
or
= v0 + bt2
dt
2
1 2

or
ds = v0 + 2 bt dt
1
or
s = v0 t + bt 3 + k
6
At t = 0, s = 0,

k=0
1

s = v0 t + bt 3
6
or

v=

Option (a) is correct.

32 | Mechanics-1
11.

1
g (2t2 + 6) m (given)
2

v2 = 2 ^i + 4 ^j
Option (b) is correct.

3rd drop
1 gt2
2

time = t

2nd drop

u2
=3
v20

or
u = v0 3
Option (a) is correct.

1 g (2t)2
2

time = t

13. v20 = 2 gh and u2 = 2 g (3 h)

14. vx = 8 t 2

dx
= 8t 2
dt
or
dx = (82 t 2) dt
or
x = 4 t 2t + k1
Now at t = 2, x = 14

14 = 4 22 (2) 2 + k1
i.e.,
k1 = 2
Thus,
x = 4 t2 2t + 2
Further,
vy = 2
dy
i.e.,
=2
dt
or
dy = 2 dt
or
y = 2t + k2
Now, at t = 2, y = 4

k2 = 0
Thus,
y = 2t
y
Substituting t = in Eq. (i),
2
2
y
y
x = 4 2 + 2
2
2

1st drop

Ground

1 2 5
gt = m
2
4
Height of 2nd drop from ground
5
= 5m m
4
= 3.75 m
Option (c) is correct.

12.
[at time = 1] 0 m/s

20 m/s

[At time = t 1]
1
s = 10t2
2
1
10 ( t 1)2
2
t2 = 4 ( t 1) + ( t2 2t + 1)
2t 3 = 0
3
t= s
2
2
1
3
s = 10
2
2
= 20 ( t 1) +

or
or
or

= 11.25 m
Option (c) is correct.
t3 ^

v = t ^i +
j
2
Thus, velocity of particle at time t (= 2 s)
will be

or
x = y2 y + 2
Option (a) is correct.
15. x = 5t and y = 2t2 + t

dx
dy
= 5 and
= 4t + 1
dt
dt
dy dy / dt
Now,
=
dx dx / dt
4t + 1
tan 45 =
5

t =1s
Option (b) is correct.

16. y = 8 t 5t2 and x = 6t

dy
dx
= 8 10t and
=6
dt
dt

(i)

(ii)

Motion in One Dimension


At t = 0

20.

dy
dx
= 8 and
=6
dt
dt
Velocity of projection
2

vmax

dx
dy
= + = 62 + 82
dt
dt

2
g

Acceleration =
In time interval t2

t =T
3

H W

H/9

vmax
= 2a
t2
v
= 2 max
t1

i.e.,
Now

R
Ground

t=T

vmax
t2

H H = 8H
9
9

vmax
=a
t1

Retardation =

H

9
Top of tower

t2

In time interval t1

= 10 ms 1
Option (c) is correct.
2H
17.
T=
g
T
=
3

t1

Option (c) is correct.


18. Distance of farthest corner from one

corner
= a + a + a = 3a
3a
Time taken =
u
Option (a) is correct.
19. Time-velocity

graph of the
time-acceleration graph will be

given

t1 = 2t2
t1 + t2 = t
2t2 + t2 = t
t
or
t2 =
3
Depth of shaft = Displacement of lift
1
= vmax ( t1 + t2 )
2
1
= vmax (2t2 + t2 )
2
3
= vmax t2
2
3
= (2at2 ) t2
2
= 3 at22
2
t
= 3 a
3

at2
3
Option (b) is correct.
1
1
21. s = 2ut + at2 = ut + 2at2
2
2
=

v 8
(m/s)

12 t (s)

Height of lift above the starting point


when it comes to rest
= Area under t-v graph
4 + 12
=
8 = 64 m
2
Option (b) is correct.

a
2u

time = 0
Q

time = t

u
2a

Q
s

ut

1 2
at = 0
2

| 33

34 | Mechanics-1
1
at = 0
2
2u
t=
a
2
2u 1 2u

s = 2u + a
a 2 a

or

or

4u2 2u2
+
a
a
6u2
=
a
Option (a) is correct.

v (m/s)
10
A1
O

A2 2

10

22. u = vertical speed (w.r.t. cart) of the

particle.
Actual path followed
by the particle

Displacement of particle at time (t = 2)


= ( + 5 m ) + ( 5 m )
= 0 m.
i.e., the particle crosses its initial position
at t = 2 s.
Option (b) is correct.
24. As downward direction is considered to be

t=T
u = Vertical speed
(w.r.t. cart of
the particle)

t=0
30 m/s
80 m

80 m
8
= s
30 m / s 3
1
For particle 0 = uT + ( g) T2
2
1
2
i.e.,
gT = uT
2
2u 8
T=
=
g 3
2u 8
i.e.,
=
10 3
40
or
u=
ms 1
3
Option (c) is correct.
1
23. A1 = ( +10) ( +1) = + 5 m
2
1
A2 = ( 10)( +1) = 5 m
2
For cart T =

f (Air resistance)

u=0

+ ive, velocity of ball at t = 0 will be v0 .


Thus, option (a) and (c) are incorrect.
Now, as the ball will have +ive
acceleration throughout its motion option
(b) is also incorrect.
Correct option is (d).
25. Let acceleration of lift = a (upwards)

Displacement of ball in time t


= Displacement of lift in time t
1
1
v0 t gt2 = at2
2
2
gt at
or
v0
=
2
2
2 v0 gt

a=
t
Option (a) is correct.
26. a = 0.2 v2

or

dv
= 0.2 v2
dt
2
v dv = 0.2 dt

v 2 + 1
= 0.2t + C
2+1
1
or
= 0.2t + C
v
Now, at t = 0, v = 10 m/s (given)
1

=0+C
10
1
i.e.,
C=
10
or

Motion in One Dimension


1
1
Thus, = 0.2t
v
10
Q For velocity v at time t (= 2 s)
1
1
= (0.2 2)
v
10
1
4
1
=

v
10 10
1 1
=
v 2

v = + 2 m/s.
Option (a) is correct.
27. For displacement ( S1) of train 1 before

coming to rest
02 = (10)2 + 2 ( 2) S1
i.e.,
S1 = 25 m
For displacement ( S2 ) of train 2 before
coming to rest
02 = (20)2 + 2 ( 1) S2
i.e.,
S2 = 200 m

Smin = S1 + S2
= 225 m
Option (b) is correct.
28. Let the balls collide after time t the first

ball is shot.
displacement (S) of ball 1 at time t
= displacement ( S) of ball 2 at time ( t 2)
1
1
40t + ( g) t2 = 40 ( t 2) + ( g) ( t 2)2
2
2
1
2
2
or
g[ t ( t 2) ] = 80
2
or
( t + t 2) ( t t + 2) = 16
or
2t 2 = 8
or
t = 5s
1
S = 40 5 + ( 10) 52
2
= 200 125
= 75 m
Option (b) is correct.
29.

i.e.,

and

0 = u gT
u = gT
1
H = uT gT2
2
1 2
h = ut gt
2

(i)
(ii)
(iii)

| 35

Substracting Eq. (ii) from Eq. (iii),


1
h H = u ( t T) + g ( T2 t2 )
2
1
= gT ( t T) g ( t T) ( t + T)
2
1

= ( t T) g T ( t + T)
2

2
T

= ( t T) g

1
= g ( t T)2
2
1
i.e., h = H g ( t T)2
2
Option (d) is correct.
t2
30.
x=
2
dx
vx =
=t
dt
x2 ( t2 / 2)2 t4
y=
=
=
2
2
8
dy t 3
vy =
=
dt
2
31.

x = t+3
i.e.,

x = t2 + 6t + 3
dx
= 2t + 6
dt
i.e.,
vt
Option (b) is correct.
32. Aeroplanes velocity at time t (= 20 s)

= Area under curve ( t-a graph)


1
= (20 5) 10 2
2

= 90 m/s
Option (c) is correct.
33. v = 5 1 + 5

At s = 0 (i.e., initially),
velocity of particle = 5 m/s
Option (b) is correct.
Differentiating equation (i) w.r.t. s
1

dv
1
= 5 (1 + 5) 2
ds
2
1
1

dv 5
v
= (1 + 5) 2 5 (1 + 5) 2
ds 2
= 12.5 m/s 2

(i)

36 | Mechanics-1
Option (b) is correct.

= 12.5 16.0 + 10
= 6.5 m/s
Option (b) is correct.

34. See answer to question no. 2.

mg f
mg + f

Time of ascent
=
Time of descent

37. u2 = 2 gH

mg ma
mg + ma

=
=

ga
g+a

10 2
10 + 2
=

H=

H
2
2
u = 100 + gH

102 = u2 2 g

Stops

2
3

Option (b) is correct.

H
2

10 m/s

35.

10 t

u2
2g

River flow

E
S

Thus,

5t

sin =

5t
10 t

1
2

H=

100 + gH
2g

or
gH = 100

H = 10 m
Option (b) is correct.

= 30
Option (b) is correct.
36. Fnet = 3 t2 32

manet = 3 t2 32
3 t2 32
or
anet =
10
2
or
anet = 0.3 t 3.2
dv
or
= 0.3 t2 3.2
dt
2
or
dv = (30.3 t 3.2) dt
0.3 t
or
v=
(3.2) t + k
3
or
v = 0.1t 3 (3.2) t + k
Now, at t = 0, v = 10 m/s

10 = k
Thus,
v = 0.1 t 3 (3.2) t + 10
at t = 5 s
v = 0.1 (5) 3 (3.2) (5) + 10
i.e.,

38.

10 m/s

15 m
u
t=0
2

10 = u2 2 g 15
100 = v2 2 10 15
i.e., u = 20 m/s
Now, v = u gt = 20 10 3
= 20 10 (3)
= 10 m/s, downward
Option (d) is correct.

Motion in One Dimension

| 37

JEE Corner
Assertion and Reason

Displacement
t0
1
v0 t0
v
=2
= 0
t0
2

1. As acceleration (a ) is just opposite to

velocity ( v), the particle will move along a


straight line.
a

Assertion is true.
Reason is also true as proved.
4. Acceleration (a) will be zero only when
v

Assertion is false (motion being one


dimensional).

Reason is true as v and a do not depend


upon time.
Option (d) is correct.
2. Displacement-time graph is parabolic only

when the slope of the straight line


velocity-time graph is not zero i.e.,
acceleration is not zero.
If acceleration is zero.
s = ut
i.e., the displacement-time graph will be a
straight line.
Assertion is wrong.
Reason : v = u + at
ds
i.e.,
= u + at
dt
or
ds = (u1 + at) dt
s = ut + at2 + k
2
If at t = 0, s = 0
The value of k will be zero.
1

s = ut + at2
2
Thus, reason is true.
3. Displacement in time t0

= Area under v-t graph


1
= v0 t0
2
average velocity in time interval t0

velocity does not change with time


dv
a =v
ds
ds dv
=

dt ds
dv
=
dt
dv
a = 0 if
=0
dt
or v is constant with time.
Thus, Assertion is wrong.
dv
Reason is correct as a is equal to
which
dt
is instantaneous acceleration.
5. If acceleration is in the opposite direction

to the velocity of the particle the speed


will decrease.
u = 10 m/s
a = 2t m/s 2
dv
= 2t
dt
dv = 22 t dt
t
v = 2 + k
2
At t = 0 s, velocity = + 10 m/s

k = + 10 m/s

v = t2 + 10
Time (t)

v m/s

speed (m/s)

15
25
35

9
6
1

9
6
1

Assertion is true.

38 | Mechanics-1
Reason is false as when acceleration is
positive the speed will increase.
da
6.
(given)
= 2 (ms 3)
dt
[This implies that reason is true]
da = 2 dt
a = 2t + C
If at t = 0, a = 0
we have
C=0

a = 2t
dv
or
= 2t
dt
Assertion is false.
Option (d) is correct.
7. If initially particle velocity is ive and

acceleration is uniform and + ive, the


particle will return to its initial position
after a certain time interval. In this time
interval the average velocity will be zero
as net displacement will be zero.
Assertion is false.
For average velocity to zero, the particle
must return to its initial position (as
discussed above) and for this velocity cant
remain constant.
Reason is true.
Option (d) is correct.
8. From O to A velocity of the particle is

increasing while from A to B it is


decreasing without change in direction as
for the velocity to change its direction the
slope of s-t graph must be negative.
Assertion is false.
If the slope of s-t graph is + ive the velocity
of the particle will be + ive while if it is
ive the velocity of the particle will also
be ive.
Reason is true.
2

9. S1 = 2t 4 t

and S2 = 2t + 4 t2
displacement of particle 2 w.r.t. 1.
S21 = S2 S1
= ( 2t + 4 t2 ) (2t 4 t2 )
= 4 t + 8 t2

Time t Relative Displacement


0s
1s
2s
3s

0m
4m
24 m
60 m

As relative displacement is increasing the


relative velocity would also be increasing.
Assertion is false.
Reason is true.
10. If v = u + ( a) t [acceleration being made

ive]
Velocity (v) of the particle will be zero
u
at
t= .
a
u
Thus, for t < , v is + ive
a
i.e., the acceleration can change its
direction without change in direction of
velocity.
Assertion is true.
If V changes sign say from + ive to ive,
V
the acceleration which equals
will also
t
change sign from + ive to ive.
Reason is true but it is not the correct
explanation of the assertion.
11. At time t when the two are at the same

height.
A
S1

u=0
a=+g

time t

S2
u = gh
B

S1 + S2 = h
1 gt2 + ut 1 gt2 = h
2

2
ut = h
gh t = h
t=

h
g

Projectile Motion
Introductory Exercise 4.1

1. A particle projected at any angle with

horizontal will always move in a plane


and thus projectile motion is a
2-dimensional motion. The statement is
thus false.

= 80

= 402 + 202

2. At high speed the projectile may go to a

place where acceleration due to gravity


has some different value and as such the
motion may not be uniform accelerated.
The statement is thus true.

i.e.,

3. See article 4.1.

= 802 + 602

As horizontal acceleration would be zero.


vy
u

sx

vx
sy

= 20 5 m/s
v y 20
tan =
=
vx 40
1
= tan 1
2
s = s2x + s2y

4. u = 40 2 m/s, = 45

uy

1
10 22 = 60 m
2
v = v2x + v2y

ux

vx = ux = u cos = 40 m/s
sx = ux t = (u cos ) t = 80 m
A : position of particle at time = 0.
B : position of particle at time = t.
As vertical acceleration would be g
v y = u y gt
= u sin = gt
= 40 20
= 20 m/s
1
s y = u y t gt2
2

i.e.,
5. s y = u y t +

= 100 m
sy
tan =
sx
60
=
80
3
= tan 1
4
1
( g) t2
2

or

s y = (u sin ) t

or

15 = 20t

or
i.e.,

1 2
gt
2

1 2
gt
2
t2 = 4 t + 3 = 0
t = 1 s and 3 s

6. See figure to the answer to question no. 4.

u = 40 m/s, = 60

ux = 40 cos 60 = 20 m/s

52 | Mechanics-1
Thus, vx = ux = 20 m/s
As = 45
tan =

vy
vx

v = v2x + v2y = vx 2

Thus,

= 20 2 m/s

r = 2.4 ^i m

i.e.,

x = 2.4 m

11. vx = ux = 10 m/s
v
u = 20 m/s

vx

60
ux = 20 cos 60

u2 sin 2 / g
2 u sin / g

2 u2 sin cos
g

g
2 u sin

8. Change in velocity
t=T
u sin

u cos

x = 0m

= ucos
+ u sin u

r = 0 ^i m

i.e.,
At t = 0.8 s,

After attaining highest point


20 = 40 3 10t
i.e.,
2=2 3 t
or
t = 2 ( 3 + 1) s
Range
7. Average velocity =
Time of flight

Before reaching highest point


v y = u y + ( g) t

20 = 40 sin 60 10 t
or
2=2 3 t

t = 2 ( 3 1) s

y-coordinate will be zero when


4 t 5t2 = 0
4
i.e.,
t = 0 s, s
5
t = 0 belongs to the initial point of
projection of the particle.

=1

y y = vx = 20 m/s

10. r = [3 t i + ( 4 t 5t2 ) j] m

u sin u

= ( u sin ) ( + u sin )
= 2usin
= 2usin (downward)
9. Formulae for R, T and H max will be same

if the projection point and the point where


the particle lands are same and lie on a
horizontal line.

= 10 m/s
vx = ux = 10 m/s
vx = v cos
v

cos = x
v
10
=
v
10
20
[as, v =
(given)]
=
10
2
=1
i.e.,
= 0
Speed will be half of its initial value at
the highest point where = 0.
Thus,

u sin
g
20 sin 60
=
= 3s
10

t=

Projectile Motion

53

Introductory Exercise 4.2


1. Time of flight

10
3

3
2

=
vx
vy

= 5 m/s

vnet

3. Let the particle collide at time t.


v

u
= 60
= 30
u = 10 m/s2
g = 10 m/s2

T=

10 m

2u sin ( )
g cos

x1

(2 10 sin(60 30 )
=
10 cos 30
2
s
=
3
Using,

x1 = (u cos ) t
x2 = v t
d = x2 x1
= (v + u cos ) t
= [10 + 10 2 cos 45 ] t = 20 t
1
Using equation, s = ut + at2
2
For vertical motion of particle 1 :
1
h 10 = (u sin ) t + ( g) t2
2
1
i.e.,
(i)
h = 10 + (u sin ) t gt2
2
1
or
h 10 + 10 t gt2
2
For the vertical motion of particle 2 :
1
20 h = gt2
2
1
i.e.,
(ii)
h = 20 gt2
2
Comparing Eqs. (i) and (ii),
1
1
10 + 10 t g t2 = 20 g t2
2
2

t = 1s

d = 20 m
and

v = u + at
vx = ux = u cos 60
1
= 10 = 5 m/s
2
v y = u y + ( g) T
= u sin 60 gT
3
2
= 10
10
2
3
3 4
= 10

2 3
5
m/s
=
3
vnet = v2x + v2y
5
= 52 +
3
=5 1+
=

1
3

10
m/s
3

2. Component of velocity perpendicular to

u = 10 m/s
v = 5 2 m/s

4.

plane
= vnet cos
=

10
cos 30
3

x2
d

v
u

B
d

54 | Mechanics-1
= 30
= 45
d = 15 m
Let the particles meet (or are in the same
vertical time t).

d = (u cos ) t + (v cos ) t

15 = (10 cos 30 + 5 2 cos 45 ) t


or
15 = (5 3 + 5) t
3
or
s
t=
3 +1

u = 40 m/s
= 60
= 30
g = 10 m/s 2
=
vx = ux = u cos
vx = v cos = v cos
u cos = v cos
u cos 40 cos 60
v=
=
cos
cos 30
1
40
2
=
3
2
40
m/s
=
3

or

= 1.009 s
Now, let us find time of flight of A and B
2 u sin
TA =
g
= 1s
As TA < t, particle A will touch ground
before the expected time t of collision.
Ans : NO.

7. (a) At time t, vertical displacement of A

= Vertical displacement of B

5. For range to be maximum

vA

20

/s
m

time t

a = 10 m/s2

= 30 =
6


= +
4 2
/6

=
+
4
2

=
3
= 60

1 2
1
gt = vB t gt2
2
2
v A sin = vB

(v A sin ) t

v
A
u

(b) x = (v A cos ) t

= (20 cos 30 )
=5 3 m

vB
vA
10 1
=
=
20 2
= 30

sin =

6. At point A velocity ( v) of the particle will

be parallel to the inclined plane.

i.e.,

vB = 10 m/s

1
2

Projectile Motion

55

AIEEE Corner
Subjective Questions (Level 1)
1. (a) R =

u2 sin 2 (20 2)2 sin 90


=
g
10

or

= 80 m
2

H=

u sin (20 2) sin 45


=
2g
10

= 40 m
2 u sin 2 (20 2) sin 45
T=
=
g
10
=4s

(b) u = (20 i + 20 j) m/s and a = 10 ^j m/s2

T=

T2 4 T 8 = 0
( 4) + ( 4)2 4(1)( 8)
2 (1)

Leaving ive sign which is not positive.


4 + 48 4 + 4 3
=
=
2
2
= (2 + 2 3 ) s
= 5.46 s
R = 20 5.46 = 109.2 m
1
(b) s = ut + at2
2

v=u+at

20 m/s

= (20 ^i + 20 ^j) + ( 10 ^j) 1(at t = 1 s)


40 m

= (20 ^i 10 ^j) m/s

a = 10 m/s2

(c) Time of flight

2 u sin
T=
g
1
2 20 2
2
=
10
=4s
(c) Velocity of particle at the time of
collision with ground.
= (20 ^i + 20 ^j) + ( 10 ^j ) 4

1
( 10) T2
2
T =2 2s
= 2.83 s
R = 2.83 20
= 56.6 m

( 40) = 0 T +

(c) s = ut +

1 2
at
2

= (20 ^i 20 ^j ) m/s
2. (a) s = ut +

1 2
at
2

20 2 m/s

40 m

20 m/s
202 m/s
+

40 m

45

1
( 10) T2
2
5T2 + 20T 40 = 0
T2 + 4 T 8 = 0

( 40) = ( 20) T +
or
or

a = 10 m/s2
T
R

or

1
( 10) T2
2
5T2 20T 40 = 0

( 40) = ( + 20) T +

T=
=

4 + ( 4)2 4 (1) ( 8)
2 (1)
4+ 4 3
= 2 + 2 3 = 1.46 s
2

56 | Mechanics-1
R = 20 1.46
= 29.2 m

100
1
40
2
5
m/s
=
2

v=

3. (a) Change in velocity (vc )

= Change in vertical velocity


(as horizontal velocity does not change).
= (u sin gt) (u sin )
= gt
= (10 3) m/s
= 30 m/s
= 30 m/s (downward)

5. Horizontal compnent of velocity at

P = Horizontal component of velocity at O


v sin
P
60 m/s

(b) u = (20 ^i + 20 ^j) m/s

a = 10 ^j m/s 2

= 60 ^i + 15 ^j
v av =

60 ^i + 15 ^j
3

= 20 ^i + 5 ^j

|v av |= (20)2 + (5)2
= 425
= 20.6 m/s
u
Player

t=0

R
t=T

50 m

60 cos 60

vT = 50

t=0

Differentiating above equation w.r.t. time t


dy
dx
dx
(i)

=b
2cx
dt
dt
dt
dy
dx
(at x = 0)

=b
dt
dt
2

60

6. Y = bx cx2

4. R + vT = 50

Coach

v cos = 60 cos 60
v cos 45 = 60 cos 60
60 cos 60

v=
cos 45
= 30 2 m/s
For point P :
v sin 45 = 60 sin 60 + ( 10) t1
3
or
30 = 60
10 t1
2
t1 = 3 ( 3 1)
= 2.20 s
For point Q :
v sin 45 = 60 sin 60 + ( 10) t2

t2 = 3 ( 3 + 1)
= 8.20 s

= 60 ^i + 60 ^j 45 ^j

t = t1 v cos
v
a = 10m/s2 v sin

Displacement at time t (= 3 s)

1
= u t + a t2
2
1
^
= (20 i + 20 ^j) 3 + ( 10 ^j) 32
2

u sin
g

2 u sin
(20)2
v
= 50
g
10
2

20

sin
45

v
= 10

10

dx
dy
v= +
dt
dt
2

dx
dx
= + b2
dt
dt
= 1 + b2

dx
dt

(ii)

Projectile Motion
Differentiating Eq. (i) w.r.t. time t
2
d2 y
d2 x
d2 x
dx
=
b

2
c
2cx 2 (iii)
2
2
dt
dt
dt
dt
Acceleration of particle
2

d2 x
d2 x
a = 2 + 2
dt
dt
d2 x
=0
dt2
d2 x
d2 y
Substituting
and
=
0
= a in
dt2
dt2
Eq. (iii)
2
dx
a = 2c
dt

(6.1) =
+ 2 ( 10) (9.1)

u y = 37.21 + 182
= 14.8
Angle of projection
uy
= tan 1
ux
uy
= tan 1
ux
14.8
= tan 1

7.6
= 62.82
u2 sin 2
Range =
g

dx
a
=
dt
2c

i.e.,

dx
Substituting above value of
in Eq. (ii)
dt
a
v=
(1 + b2 )
2c
^

uy

B
+h

(u2x + u2y ) sin 2


g
(v2x

u2y )

sin 2

[(7.6)2 + (14.8)2 ] sin 125.64


=
10
= 22.5 m
(c) Magnitude of velocity just before the
ball hits ground

7. (a) Velocity at point A = 7.6 i + 6.1 j


vy

57

u2y

= [( u y )2 + (ux )2 ]2
9.1 m

a = 10 m/s2

= [( 14.8)2 + (6.1)2 ]2

Initial vertical velocity at a = 6.1 ^j m/s.


Final vertical velocity at B (highest point)
= 0 m/s
2
2
Using v = u + 2as
(Between A and B)
02 = (6.1)2 + 2 ( 10) ( + h)
(6.1)2

h=
20
= 1.86 m
Maximum height attained by ball
= 9.1 + 1. 86
= 10.96 m
(b) Let magnitude of vertical velocity at O
(point of projection) = u y
Using v2 = u2 + 2as
(Between O and A)

= 16 ms

[ux = vx ]

1 u y

(d) = = tan
ux
14.8
= tan 1

7.6
= tan 1 (1.95)
8. As

initial vertical velocities of both


particles will be zero and both fall under
same acceleration (g), at anytime t, the
vertical displacement of both will be same

u1

time = t

12

u2

+h

a=+g
time = t

B
(90 )

58 | Mechanics-1
i.e., both will always remain in the same
horizontal line as shown in figure.
At time t :
Vertical velocity of A
= Vertical velocity of B
= 0 + ( + g) t
= gt
At A :
Vertical velocity of particle 1
tan =
Horizontal velocity of particle 1
gt
or
(i)
tan =
u1

Y
y
+
= tan
x Rx

or
or

tan + tan = tan


(Students to remember this formula)
3 3

tan = +
6 12
9
=
12
3

= tan 1
4
10. On the trajectory there be two points P

and Q at height h from ground.


P

At B :

Vertical velocity of particle 2


Horizontal velocity of particle 2

tan (90 ) =

or

cos =

gt
u2

(ii)

Multiplying Eq. (i) by Eq. (ii),


gt gt
1=

u1 u2
t=

u1u2
g

Distance between A and B


= R1 + R2
= u1t + u2 t
= (u1 + u2 ) t
uu
= (u1 + u2 ) 1 2
g

If particle takes t time to reach point A


(i.e., vertical displacement of + h)
1
( + h) = (u sin ) t + ( g) t2
2
2
or
(i)
gt 2 (u sin ) t + 2h = 0
The above equation is quadratic in t. Two
values of t will satisfy Eq. (i). One having
lower value will be time ( = t1) to reach
point P while the higher value will be the
time ( = t2 ) to reach point Q.
Time to reach point Q from point P
= t2 t1
= ( t2 + t1)2 4 t1t2
2

2u sin
2h
=
4
g

g
[Using Eq. (i)]
4u2 sin2 2 gh
=
g

x
= tan
R
(y, x)

or

u
Y = 3m


x
=6m

a=g

7 2
9.8
= 2.47 m

9. Y = x 1

(R x)

YR
= tan
x( R x)

( t2 t1) =

16 gh sin2 8 gh
g
(Qu = 2 gh)

Distance between P and Q :


2h = (u cos ) ( t2 t1)

4 h2 = u2 cos2 ( t2 t1)2
16 gh sin2 8 gh
or 4 h2 = ( 4 gh) cos2

g2

Projectile Motion
2

or
or
or
or

1 = cos (16 sin 8)


1 = cos2 [16 (1 cos2 ) 8 ]
1 = cos2 [8 16 cos2 ]
16 cos4 8 cos2 + 1 = 0
( 4 cos2 1)2 = 0
1

4 cos2 =
4
1
cos =
2
cos = 60
1
(cos = being not possible).
2
16 gh sin2 60 2 gh

( t2 t1) =
g2
=2

h
g

R+ b=
i.e.,

aR + ab =

13. R1 = v1

2h
g
v1

or u sin cos u cos sin = gt cos


gt cos
or
u=
sin ( )

R1 = 10 m
v2

R2

i.e.,

u sin 2
g

(i)

1 R1 g

2 v1

1 10 9.8

2
5

= 19.6 m
2h
T=
g
=

bu2 sin 2
bR ab =
g

h=
=

i.e.,

b sin 2 + a sin 2
1
sin 1

2
a+b

a=g

At time t vertical velocity = v sin


(i)

v sin = u sin = ( g) t
Now, as horizontal acceleration will be
zero.
v cos = u cos
Thus, Eq. (i) becomes
u cos

sin = u sin gt
cos

12. R a =

(ii)

(Proved.)
a=0

au2 sin 2
g

( b sin 2 + a sin 2)
u2 sin 2
= u2
g
( a + b) g

u sin 2
g

Adding Eqs. (i) and (ii), we have


b sin 2 + a sin 2
bR + aR = u2

(
b
sin
2

+
a
sin
2

)
or
R = u2
( a + b) g

v
v

R
2

(Proved)

11. Initial vertical velocity = usin

59

2 19.6
9.8

=2s
R2 = v2 T = 7.5 2
= 15 m

60 | Mechanics-1
= 1.69 s
2u2 sin ( ) cos
R=
g cos2

14. Vertical velocity of balloon (+ bag)

O
12 km/h

2 (20 2)2 sin 15 cos 45


10 cos2 30
= 39 m
2 u sin ( + )
T=
g cos

50 m

vw = 20 km/h

= 12

5
m/s
18

10
m/s
3
Horizontal velocity of balloon (+ bag)
5
m/s
= Wind velocity = 20 km / h = 20
18
50
m/s
=
9
= 5.55 m/s
12
tan = . i.e., sin = 0.51
20
Bag is released at point A.
Let t be time, the bag takes from A to
reach ground.
1
Using, s = ut + at2
2
10
1

( 50) = sin t + ( g) t2
3

16.

i.e.,
t=

5t2 1.7 t 50 = 0
1.7 + ( 1.7)2 4 5 ( 50)
25

= 3.37 s
Vertical velocity of bag when it strikes
ground
10
vB =
+ (10) (3.37)
3
= 37.03 m/s
v = 5.55 m/s
Velocity of bag with which it strikes
ground
vnet = v2B + v2

= 6.31 s
u2
R=
[sin (2 + ) + sin ]
g cos2
(20 2)2
[sin (90 + 30 ) + sin 30 ]
10 cos2 30
= 145.71 m
2 u sin ( + )
17.
T=
g cos
2u sin
(Q = 0)
=
g cos
2u
=
tan
g
2 20
=
tan 30
10
= 2.31 s
u2
R=
[sin (2 + ) + sin ]
g cos2
=

u2 ( 2 sin )
g cos2
20 2.31
uT
=
=
cos 30
cos

2 20 2 sin ( 45 30 )
10 cos 30

[as = 0]

= 53.33 m
u2
18. R =
[sin (2 + ) + sin ]
g cos2

= 37.44 m/s
2 u sin ( )
15. T =
g cos
=

2 20 2 sin ( 45 + 30 )
10 cos 30

u2
[sin {2 ( + ) } + sin ]
g cos2

u2
[sin ( ) + sin ]
g cos2
[Q( + ) =

or

R=

2 u2
tan sec
g

]
2

Projectile Motion
19. (a) Acceleration of particle 1 w.r.t. that of

particle 2
= ( g) ( g)
=0

61

Vertical acceleration of stone = g


Horizontal velocity of stone = v
Path of the stone will be parabolic.
21. (a) g eff = g ( a)

(b) Initial velocity of 1st particle = 20 j m/s

a m/s2

w m/s

Initial velocity of 2nd particle


= (20 2 cos 45 ^i + 20 2 sin 45 ^j) m/s

= (20 ^i + 20 ^j) m/s

Initial velocity of 1st particle w.r.t. that


of 2nd particle
=g+a

= [ (20 ^j) (20 ^i + 20 ^j ) ] m/s

= 10 + 1

= 20 ^i m/s

= 11 m/s 2
2 u sin
T=
g eff

= 20 m/s (downward)
(c) Horizontal velocity of 1st particle
= 0 m/s
Horizontal velocity of 2nd particle

= 20 ^i m/s
Horizontal velocity of 1st particle w.r.t.
that of 2nd particle
= 0 (20 ^i)

= 0.18 s
(b) Dotted path [(in lift) acceleration
upwards]
u

= 20 ^i m/s
Relative displacement of 1st particle w.r.t.
2nd particle at t = 2 s
= 20 ^i 2
= 40 ^i m/s
Distance between the particles at t = 2 s
= 40 m

Full line path [In lift at rest or moving


with constant velocity upwards or
downwards].
(c) If lift is moving downward with
acceleration g.
Path of particle
u

20. (a) As observed by passenger

Vertical acceleration of stone


= g 0= g
Horizontal velocity of stone
=vv=0
Path of the stone will be a straight line
(downwards).
(b) As observed by man standing on
ground

2 2 sin 30
11

g eff = g g = 0
22. Horizontal motion :

x1 = u1 cos 1 and x2 = u2 cos


u1 cos 1 + u2 cos 2 = 20

(i)

62 | Mechanics-1
Vertical motion :

20 + (u1 sin 1) t +
u2
2

or

20

1
( g) t2
2
(ii)
(u1 sin 1 u2 sin 2 ) t = 10
+ (u2 sin 2 ) t +

u1

x1

1
( g) t2 = 30
2

x2

d = 20 m

Objective Questions (Level 1)

1. v = 3 i + 4 j and F = 4 i 3 j

v F = (3 ^i + 4 ^j) ( 4 ^i 3 ^j)

= 14.14 m/s
= 14 m/s (approx)
Option (b) is correct.
u2 sin2
4. H (maximum height) =
2g

= 12 12 = 0

Fv

Path of the particle is circular.


Y

H1 =

u2 sin2 (90 )
u2 sin2
and H2 =
g
2g
=

Option (c) is correct.


5. Equation to trajectory is

1
gx2
2
2 u cos sin
x2
Y = x tan
R

Y = x tan

Option (c) is correct.


2. Projectile motion is uniformly accelerated

everywhere even at the highest point.


Option (a) correct.
Option (b) incorrect.
At the highest point acceleration is
perpendicular to velocity.
Option (c) incorrect.
3. For range to be maximum

i.e.,
or

= 45
ux
1
= cos 45 =
u
2
u
vx =
2

u2 cos2
g

H1 sin2
=
H2 cos2

Thus,

j
i

20
2

(Qvx = ux )

Area =
=

0 Y dx
x2
x tan dx
R

x2
x3
A = tan
3 R 0
2
R2
R3
=
tan
2
3
1
2 tan
=R

3
2

Projectile Motion
=

4v40

sin cos tan 1

2
3
g2

4v40 sin 3 cos sin2 cos2

2
3
g2

2 v40
=
[3 sin2 cos 2 sin2 cos2 ]
3 g2

9. Rmax = 1.6 m

u2
= 1.6
g

or

u cos 60
cos 30
u
v=
3

v=

4 2
10
10 2
10 2
Number of jumps =
=
T
4 2 / 10

1
mv2
2

=
=

1
u2
m
2
3
K
=
3

1
2
or
2 = 30

= 15
Option (a) is correct.
2 u sin
8. T1 =
g
2 u sin (90 )

T2 =
g
2u cos
=
g
2
2 u2 sin cos
Thus,
T1T2 =
g
g
2R
or
T1T2 =
g
1

R = g T1T2
2
Option (d) is correct.
sin 2 =

Q 1 mu2 = K

Option (b) is correct.


u2 sin 2 1 u2
7.
=
g
2 g
or

= 25
Grass hopper would go
= 25 1.6 m i.e., 40 m.
Option (d) is correct.
|Displacement|
10. |Av. velocity|=
time

or
KE at B =

u = 4 m/s
2u sin 45
T=
g

6. v cos = u cos

63

1
R
2
+H
T / 2 2
1
u sin
g

u2 sin cos
u2 sin2

2g

= u cos2 +

sin2
4

u
1 + 3 cos2
2
Option (b) is correct.
=

11. d2 = R2 + u2 T2
T
u

2 /g

Ball

=u

45
90

uT
T
2 2

u
2 u sin 45
= + u2

g
=

u4 2 u4
+ 2
g2
g

3u4
g2

64 | Mechanics-1
d=

u2
g

3=

302
10

3 = 90 3 m

Option (b) is correct.


12. At maximum height

dy
=0
dt
d
(10t t2 ) = 0
dt
or
10 2t = 0
or
t = 5s
Maximum height attained = 10 (5) 52
= 25 m
Option (d) is correct.
u2
13. R =
[sin (2 + ) + sin ]
g cos2
i.e.,

a = g

1
( 10) t2
2
or
5t2 25t 70 = 0
or
t2 5t 14 = 0
( t 7) ( t + 2) = 0
t=7s
( 2 s not possible)
Option (c) is correct.

As = 0 (according to question)
u2
R=
[2 sin ]
g cos2

1000
m
3

sin 2 = sin 3
2 = 2

or
=
2

= = = 30
2 3 6
Maximum height attained by second
particle
u2 sin2
=
2g
102 2 g sin2 30
=

2g
sin2 60
102 1 / 4
=
= 34 m
3/4

(50)2 2 sin 30
10 cos2 30

u2 sin 2 u2 sin 2
=
g
g

Option (d) is correct.


1
15. s = ut + at2
2

if,

( 70) = (50 sin 30 ) t +

16. Initial separation

53

Option (b) is correct.

x
R

14. First particle :

H max = 102 m

u2 sin2
= 102
2g
102 2 g
u2 =
sin2 60

[As =

= 60]
3

Second particle :
Range of the second particle will be equal
to that of particle

5T

x = R + 5T
= (u cos ) T + 5T
= T [(u cos ) + 5]
2u sin
=
[(u cos ) + 5]
g
4
2 80
5 80 3 + 5 = 256 m
=

10
5
Option (d) is correct.

Projectile Motion

65

JEE Corner
Assertion and Reason
1. Assertion is wrong while the explanation

and

as given in reason is correct.


Option (d) is correct.

Thus,

2. Assertion and

Reason both are correct.


both carry the same meaning.
Option (a) is correct.
u2 sin2
3. H =
[H = maximum height]
2g
2

g 2 u sin
g 2

= T
8
g
8

i.e.,
H T2 (Reason)
H will become four times if T is made
two times.
Thus, assertion is correct and also reason
is the correct explanation of the assertion.
Option (a) is correct.

4. Reason is correct as A B = 0 when A and

B are perpendicular to each other.

y=

2y 1 2y
+ ax
ay 2
ay

x = ux

or

x = k1 y + k2 y

or

( x k2 y)2 = k12 y

or

x2 + k22 y2 2k2 y k12 y = 0

or

x2 + k22 y2 (2 k2 + k12 ) y = 0

6.

1
a y t2
2

v2 v1
t2 t1

( u + a t2 ) ( u + a t1)
t2 t1

= a (which is a constant quantity in


projectile motion).
Assertion is correct and the reason for
this also correct, as explained.
Thus, option (a) is correct.
7. Let initial velocity of A = u A

Initial velocity of B = uB
B is projected at an angle with
horizontal.
For time (TA ) taken by A to return to the
point of projection :

u
45
45

In the case mentioned v is perpendicular

to u only at t = T and not at any time t.


Option (b) is correct.

[Note : v will never be perpendicular to u


if angle of projection is less than 45.]
1
5. x = ux t + ax t2
2
y

ay

( u A ) = ( + u A ) + ( g) TA
2u
TA = A = 4 (s)
g

Time taken by B to reach ground


2 uB sin
TB =
= 4 (s)
g
(Q TB = TA given)

uB sin = 2 g
ux

Height attained by B

ax
x

H=

u2B sin2
2g

66 | Mechanics-1
=

(2 g)2
2g

Reason is correct as the given relation


under heading reason leads to
2v
Range = vH V
g

= 2g
= 20 m
Assertion is correct.
Reason for this incomplete as then TA will
not be equal to TB .
Thus, option (c) is correct.
u2
8.
H= V
2g
H1 : H2 = 4 H : H
u12V u22 V
:
= 4 :1
2g 2g

i.e.,
or

energy
(i)

1
1
mv2 + mgh = mu2
2
2
2
v = u 2 gh

Thus, assertion is correct.

For R1 = R2
u1V u1H = u2 V u2 H

or

which would decrease by 10 m/s in the


downward direction.
Acceleration is nothing but rate of change
of velocity written against reason is
correct.
Option (d) is correct.
10. Using law of conservation of mechanical

u1V : v2 V = 2 : 1
2 uV uH
R=
g

u1H u2 V
=
u2 H u1H
u1H 1
=
u2 H 2

Option (a) is correct.


9. Assertion is incorrect as it is the velocity

If the particle is projected with vertical


component of velocity as u y the vertical
component of velocity of the particle at
height h would be u2y 2 gh is correct as
[Using relation (i)]

Assertion is correct.

written against reason but it is not the


correct explanation of the assertion as
nothing is given regarding the change in
the horizontal component of velocity.

Objective Questions (Level 2)


Single Correct Option
1. s = ut +

1 2
at
2

Time for attaining maximum height


u sin
=
g

u P

16.8 m

Q
4.8 m

1
= (u sin ) t + ( gt)2
2
= 24 sin 7 2
For = 30 :
s = 4.8 m

= 1s
Above displacement is for point Q.
For = 90 :
s = 16.8 m
For = 30 : OD = (u cos ) t
= (20 cos 30 ) 1 2
= 20.8
2
QR = (20.8) + (16.8 4.8)2
= 24 m
Option (c) is correct.

Projectile Motion
2. t =

d
1
also h = gt2
u
2
u

h
time = t

5.

1
d2
g 2
2
u
2 u2 h
2
d =
g

h=

Option (b) is correct.


u2
3. R =
[sin (2 + ) + sin ]
g cos2
Substituting v = 10 m/s
g = 10 m/s2
+ = 90
and
= 30
(10)2
R=
[sin (90 + 60 ) + sin 30 ]
10 cos2 30
10 1 1 40
m
=
+
=
3 2 2 3

4

y = x2
dy
dx

= 2x
dt
dt
d2 x dx 2
d2 y
and
= 2 x 2 +
dt
dt2
dt
d2 x dx 2
or
= 2 x 2 +
dt
dt
2
d x
Now, as 2 = 0
dt
(acceleration being along y-axis only)
dx

=
dt
2
Option (d) is correct.

6. As the projectile hits the inclined plane

horizontally
P

Option (c) is correct.


1
x2
4. y = x tan g 2
2 u sin cos

R'

Slope

PQ = H max =
x

dy
1
2x
= tan
dx
2 u2 sin cos
Slope

OQ =

u2 sin cos
g

u2 sin2
u2 sin cos
=
+

g
2g

tan

and

u2 sin2
2g

Range on inclined plane


R = ( PQ)2 + (OQ)2

T = Time of flight

T/2

67

x
Slope of trajectory = tan 2
u sin cos
Substituting x = (u cos ) t
(u cos ) t
dy
= tan 2
dx
u sin cos
t
= tan
u sin
Option (a) is correct.

T
t

u2
g

sin4
+ sin2 cos2
4

u2
g

3 1 2
1 3
+
4 4
2 2

68 | Mechanics-1
=

u2
g

u2 21
8g

9
3
+
64 16

Acceleration of the particle


d2 x
= 2
dt
= 50 m/s2

Option (d) is correct.


7. Let the projectile hits the inclined plane at

[as

dy
= constant]
dt

9. tan = tan + tan

P at time t
u = 10 3 m/s
i.e., v = u

v = 10 3 m/s
v


R/2

R/2

P (time = t)

(u cos ) t
vt

Further,
and
i.e.,

or

or
or

or
or
or

dy
dy dx
+2
=
dt
dt dt
dy dx
(2 y + 2)
=
dt dt
dx
= 10 ( y + 1)
dt
d2 x
dy
= 10
dt
dt2

2y

= 50 m/s 2

10. At any time t,


20 m/s

60

30
203m

OQ tan = PQ
1
t [v cos ] tan = u sin gt t

2
10 [1 cos 60 ] tan 30
1
= 10 sin 60 10

2
5
= 5 3 5t
3
1
t= 3
3
2
s
=
3

Option (c) is correct.


203 m/s

1
PQ = (u sin ) gt2
2
QR = vt (u cos ) t
PQ
tan =
QR

8. y2 + 2 y + 2 = x

= tan + tan = 2 tan


tan
= tan 1

Horizontal distance between particles


x = 20 3 (10 3 + 10 3 ) t
= 20 3 (1 t)
Vertical distance between particles at
time t
y = (30 10) t = 20 t
Distance between particles at time t
D = x2 + y2
D2 = [20 3 (1 t)]2 + [20 t ]2
= 400 [3 (1 t)2 ] + 400 t2
For D to minimum
dD
=0
dt
3 2 (1 t)( 1) + 2 t = 0
or
6t 6 + 2t = 0
3
i.e.,
t= s
4
2
2

3
3
D2min = 400 3 1 + 400
4
4

or

= 75 + 75 3

Dmin = 10 3 m
Option (b) is correct.

Projectile Motion

Substituting v = u and = = 30 in Eq.


(ii)
u cos 30 + u cos 30
T=
g
2 u cos 30
=
g

11. Let time of flight = T


u

69

As, horizontal component of the velocity of


the particle will not change
u cos (90 ) = v cos (90 )
or
(i)
u sin = v sin
Using v = u + at for the horizontal
component of velocity
( v cos ) = ( + u cos ) + ( g) T

gT = u cos + v cos
u cos + v cos
or
(ii)
T=
g
Now, as = ( = 30 ), u = v from Eq. (i)

u 3
g

Option (b) is correct.


2 u2 sin cos
12. PQ =
g
or2a cos =

2 u2 sin (90 ) cos (90 )


g

or

a=

u2 sin
g

or
(as = 30)
u2 = 2ag
or
u2 = 2 4.9 9.8
or
u = 9.8 m/s
Option (a) is correct.

More than One Correct Options


1. Two particles projected at angles and

with same speed ( = ) will have same


range if
+ = 90
Option (a) is correct.
2 u2 sin cos
R=
g
h1 (maximum height attained by first)
u2 sin2
=
2g
h2 (maximum height attained by second)
u2 sin2
=
2g

Option (b) is correct.

Wind
a=g

a = constant

u2 sin cos R
=
2g
4

flight)

l
tic
ar
fp

Option (d) is correct.


2. Horizontal displacement in time t (Time of

o
th

u2 sin sin
2g
u2 sin sin (90 )
=
2g

h1h2 =

Option (c) is correct.


h1 sin
Also,
=
= tan
h2 sin

Pa

2u sin
g
2u sin
t2 (time of flight of second) =
g
t1 sin
sin
=
=
= tan
t2 sin sin (90 )
t1 (time of flight of first) =

x=

1 2
at
2

(i)

70 | Mechanics-1
Vertical displacement in time t
1
y = gt2
2
y g

=
x a
or y = constant x [as a is constant]
Option (a) is correct.
Substituting y = 49 m in Eqs. (ii)
1
49 = 9.8 t2
2

t = 10 = 3.16 s
PQ > PR ( = 40 m )
Option (d) is correct.

(ii)

3. As there will be no change in the

horizontal component of the velocity of the


particle

v
P

v cos

+ = 90

w cos
w

w cos = v cos
or
w cos (90 ) = v cos
or
w sin = v cos
or
w = v cos
Option (b) is correct.
Option (a) is incorrect.
Vertical velocity of particle at Q
= wsin
= wsin (90 )
= wcos
Using, v = u + at
( w cos ) = ( + v sin ) + ( g) T
[where T = time from P to Q]

gT = v sin + w cos
= v sin + v cot cos
cos2
= v sin + v
sin
= v cosec
v cosec

t=
g

Option (c) is correct.


Option (d) is incorrect.

4. v = 10 i + 10 j
v
H
O
a = 10 m/s2

15m

If be the angle of projection


v y 10
tan =
=
=1
vx 10
i.e.,
= 45
Option (a) correct.
1 2
at
2
1
( 15) = 10 t + ( 10) t2
2
2
i.e.,
t 2t 3 = 0
or
( t 3) ( t + 1) = 0

t=3s
Option (b) is incorrect.
Horizontal range of particle
= vx t
= 10 3 = 30 m
Option (c) is incorrect.
Maximum height of projectile from ground
= H + 15
u2 sin2
=
+ 15
2g
Using relation, s = ut +

=
=

v2y
2g

+ 15

(10)2
+ 15
2 10

= 20 m
Option (d) is correct.
5. Average velocity between any two points

cant remain constant as in projectile


motion
velocity
changes
both
in
magnitude and direction.
Option (a) is incorrect.
Similarly option (b) is also incorrect.

Projectile Motion
In projectile motion

a = constant

dv
= constant
dt
Option (c) is correct.

d2 v
=0
dt2
Option (d) is correct.
1
6. ( + h) = ( + u sin ) t + ( g) t2
2
t = t1
A

gt2 (2 u sin ) t + 2h = 0
2 u sin
2h
, t1t2 =
t1 + t2 =
g
g

( t2 t1)2 = ( t2 + t1)2 4 t1t2


4u2 sin2
2h
=
= 4
g
g
t AB =

40
2
= 20 m/s

ucos =

40 m

20 m

u sin = 40 g
= 20 m/s
Option (c) is correct.
(u cos ) tAB = 40

Bt=t

15 m

[where H = Maximum height]


8 ( H 15)
2=
10

H = 20 m
Option (b) is correct.
u2 sin2
i.e.,
= 20
2g

also

71

8 ( H h)
g

(i)

Option (d) is correct.


Substituting values of g, usin and h in
Eq. (i)
10t2 40t + 30 = 0
i.e.,
t2 4 t + 3 = 0

t = 1 or 3
t = 1 s for A and t = 3 s for B.
Option (a) is correct.

Match the Columns


1. (a) At t = 2 s
S1

S2

2 (t = 1 s)
10 m
1 (at t = 1s)
10 m/s

30 m
1 (at t = 2 s)

S3

2
at t = 2 s

S1 : S2 : S3
=1:3:5

(b) Vertical distance between the two


at t = 2 s = 30 m
(b) (s)
(c) Relative
velocity

horizontal

component

of

= 10 0 = 10 m/s
(c) (p)
(d) Relative vertical component of velocity
(at t = 2 s)
= Velocity of 1 at t = 2 s

50 m

horizontal distance between 1 and 2


= horizontal displacement of 2
= 10 1 = 10 m
(a) (p)

Vertical velocity of 2 at t = 2 s
= Velocity of 1 at t = 2 s
Velocity of 1 at t = 1 s
= (2 g) (1 g) = g
= 10 m/s
(d) (p).

72 | Mechanics-1
2. Given, H = 20 m

u2 sin2
= 20
2g
usin = 20

(d) (s)
Time of flight =

2u sin
=4s
g

(a) (s)
R
(given)
2
u2 sin2 1 2 u2 sin cos

=
2g
2
g
u sin

=2
u cos
(b) (q)
u sin
u cos =
= 10 m/s
2
(c) (r)
u2 sin (2 15 )
3.
= 10
2g
H=

(a) (q)

u
= 20 m
2g
Rmax = 20 m

(b) Maximum height =

u2
= 10 m
4g

(b) (p)
(c) Range at ( 45 + ) = Range at ( 45 )
Range at 75 = Range at 15
= 10 m (given)
(c) (p)
u2 sin2 30
(d) Height at 30 =
2g
=

1 u2

4 2g

= 5m
(d) (s)
2 u sin
4. (a) T =
g
2u
=
g

On increasing u to two times the T


(time) of flight will also become two times
i.e., (a) (q).
(b) H =

u2 sin2 u
=
2g
2g

H max will become 4 times.


(b) (r)
2(u ) (u||)
(c) R =
g
R will become two times
(c) (q)
(d) Angle of projection with horizontal
u
tan new = (new)
u||
2u
=
u||
= 2 tan
new = tan 1 (2 tan ) 2
(d) (s)

5. (a) u (at 0) = (u cos ) i + (u sin ) j

v (at A) = (u cos ) ^i

|Change in velocity between A & B|


= usin
(a)
(q)

(b) |Average velocity between 0 and A|

|OA|
=
T /2

R
H2 +
2

T /2
u2 sin2 2 1 2 u sin cos 2

+

4
g

2 g

u sin
g

u
1 + 3 cos2
2
(b) (s)
=

Projectile Motion
(c) Velocity at B

vB

73

Vertical displacement of (2) w.r.t. (1),


^

= (90 ^i 40 ^j) m

= (u cos ) i (u sin ) j

= 50 ^j m

|v B u| = 2 u sin
Thus, (c) (s).
(d) |Average velocity between 0 and B|
Range
=
Time of flight
(u cos ) T
=
= ucos
T
(d) (p).
6. (a) Horizontal displacement of (1) at t = 2 s
Y

(b) (r)
(c) Horizontal velocity of (1) at t = 2 s

v1H = 30 ^i

Horizontal velocity of (2) at t = 2 s

v2 H = 20 ^i

Relative horizontal component of the


velocity of (2) w.r.t. 1

v RH = v2 H v1H

20 j

= ( 20 ^i) ( + 30 ^i)

(2)
20 i

= 50 ^i m/s

30 j

|v RH|= 50 m/s
X

(1)

30 i

Thus, (c) (r)


(d) Vertical velocity of (1) at t = 2 s

S1H = 30 ^i m/s 2 s

v 1 V = 30 ^j + ( 10 ^j) (2)

= + 60 ^i m

= 10 ^j

Horizontal displacement of (2) at t = 2 s

Vertical velocity of (2) at t = 2 s

v 1 V = 20 ^j + ( 10 ^j) (1)

S2 H = 130 i m + ( 20 i m / s) (1 s)
= 110 ^i m

= 10 ^j

Horizontal displacement of (2) w.r.t. (1)


^

= (110 i 60 i) m = 50 i m
(a) (r)
(b) Vertical displacement of (1) at t = 2 s

1
S1V = 30 ^j m/s 2 s + ( 10 ^j m / s2 ) (2 s)2
2
= 60 ^j m 20 ^j m
= 40 ^j m
Vertical displacement of (2) at t = 2 s

S2 V = 75 ^j m + (20 ^j m / s) (1 s)
1
+ ( 10 ^j m / s2 ) (16)2
2
= 90 ^j m

Relative vertical component of the velocity


of (2) w.r.t. 1

v RV = (10 ^j) (10 ^j) = 0

|v RV|= 0 m/s
Thus (d) (s)
2 uy
7. T =
T uy
g
H=

u2y
2g

H u2y

H A = HB = HC
TA = TB = TC
Further, R = ux T
T is same. But
R A < RB < RC
u XA < u XB < u XC

Laws of Motion
Introductory Exercise 5.1

1. N = Normal force on cylinder by plank


R

N sin + w A = N

Force acting on cylinder


f

R = Normal force on cylinder by ground,


f = Force of friction by ground by cylinder,
w = Weight of cylinder.
N cos = f
w + N sin = R,
N ( R) = Reaction to N,

N (R)
B
R'

Force of sphere B

wB

wall,

O
(Force acting on
plank)

R'
w

f'

Ground

i.e.,normal force on plank by cylinder


R = Normal force on plank by ground,
w = Weight of plank,
f = frictional force on plank by ground.
Resultant of f and R, N ( R) and w pass
through point O.

N ( R) = Reaction to N i.e. normal force on


sphere B by sphere A,
wB = Weight of sphere B,
R, N ( R) and wB pass through point O, the
centre sphere B.
3. N = Normal force on sphere by wall,
C

2.

B
A

R Force on sphere A
w

N'

R = Normal force on sphere B by right

N(R)

R = Normal force on sphere A by left wall,


N = Normal force on sphere A by ground,
N = Normal force on sphere A by sphere B,
w A = Weight of sphere A.
N cos = R

Laws of Motion
w = Weight of sphere,
T = Tension in string.
4. Component

7. R cos 30 + 3 = f cos 60
R

of F1

60

30
3N

along x-axis : 4 cos 30 = 2 3 N


along y-axis : 4 sin 30 = 2 N
Component

i.e.,

along y-axis : 4 sin 120 = 2 3 N

or
and

of F3

along x-axis : 6 cos 270 = 0 N

i.e.,

along y-axis : 6 sin 270 = 6 N

or

Component

10 N

60

of F2

along x-axis : 4 cos 120 = 2 N

Component

of F4

along x-axis : 4 cos 0 = 4 N


along y-axis : 4 sin 0 = 0 N
5. Taking moment about point A
T

O
A

T sin 30

30
AB = l

R 3
f
+3=
2
2
R 3 +6= f
R sin 30 + f sin 60 = 10
1
3
R +f
= 10
2
2
R + f 3 = 20

( T sin 30 ) l = w

(i)

(ii)

Substituting the value of f from Eq. (i) in


Eq. (ii)
R + ( R 3 + 6) 3 = 20
4 R + 6 3 = 20
20 6 3

R=
= 2.4 N
4

f = (2.4) 3 + 6
= 10.16 N
8. At

point B (instantaneous
acceleration only)

vertical

AB = l

l
2

T
T

T=w

75

45

6. See figure (answer to question no. 3)

OA
OB + BC
a
1
=
=
a+a 2

sin =

or
or

T cos 30 = w
3
T
=w
2
2
T=
w
3

mg

mg T sin 45 = ma

(i)

At point A (instantaneous horizontal


acceleration only)
(ii)

T cos 45 = ma
Combining Eqs. (i) and (ii)
mg ma = ma
g

a=
2

76 | Mechanics-1
Introductory Exercise 5.2
1. Acceleration of system

a=

( + 120) + ( 50)
1+ 4 +2

n
co
v(

sta

nt)

= 10 m/s2
Let normal force between 1 kg block and
4 kg block = F1
Net force on 1 kg block = 120 N
120 F1

a=
1
or
10 = 120 F1
i.e.,
F1 = 110 N
Net force on 2 kg block = 2 a
= 2 10
= 20 N
2. As, 4 g sin 30 > 2 g sin 30

The normal force between the two blocks


will be zero.
mg
3. N ( R) =
4

T
mg

=30

Tension in spring = mg
= 1 10
= 10 N
5. Pseudo force ( = ma) on plumb-bob will be

as shown in figure
a
T
ma

90 mg

N
a

= 30

ma
A
mg
N (R) = N

N=

mg
4

As lift is moving downward with


acceleration a, the pseudo force on A will
be ma acting in the upward direction.
For the block to be at rest w.r.t. lift.
N + ma = mg
mg
or
+ ma = mg
4
3

a= g
4
4. Angle made by the string with the normal

to the ceiling = = 30
As the train is moving with constant
velocity no pseudo force will act on the
plumb-bob.

T cos = mg + ma cos (90 )


i.e.,
(i)
T cos = mg + ma sin
and
T sin = ma cos
Squaring and adding Eqs. (i) and (ii),
T2 = m2 g2 + m2 a2 sin2 + 2m2 ag sin
(iii)
+ m2 a2 cos2
2
2 2
(Q = 30)
T = m g + m2 a2 + m2 ag
g
g2
g
= m2 g2 + m2
+ m2 g (Q a = )
4
2
2
7 m2 g2
=
4
mg 7
or
T=
2
=5 7 N
Dividing Eq. (i) by Eq. (ii),
ma cos
tan =
mg + ma sin

Laws of Motion
a cos
=
g + a sin
cos
=
2 + sin
cos 30
=
2 + sin 30

6.

2kg

1kg

F=8N

8
= 2 m/s 2
2+1+1
B

T1 T1

1kg
T2 T2

Net force on 1 kg mass = 8 T2

8 T2 = 1 2

T2 = 6 N
Net force on 1 kg block = T1

T1 = 2a = 2 2 = 4 N

3
= tan 1
5

i.e.,

a=

3
5

2kg

Introductory Exercise 5.3


1. F = 2 g sin 30 = g
T1

T2

T1
T1

T2 F

T1

T1

T1

mg

T1

T2
T2

4g

T2

2g

1g

For the system to remain at rest

or

3g

T2

T2 = 2 g

(i)

T2 + F = T1

(ii)

T2 + g = T1

[ii (a)]

T1 = mg

(iii)

Substituting the values of T1 and T2 from


Eqs. (iii) and (i) in Eq. [ii(a)]

and

T1 = 4 g
T2 = 1 g

T1
=4
T2

3. 2 g T = 2a

2g + g = mg
i.e.,

77

m = 3 kg

2. As net downward force on the system is

zero, the system will be in equilibrium

a
1g
2g

8N

78 | Mechanics-1
T 1 g = 1a
Adding above two equations
1g = 3a
g

a=
3

u2 ( g / 3)2
g
=
=
2a
2g
18

Downward displacement of 2 kg block


2
1
1
g
g
= at2 = g =
3
2
2
18

Velocity of 1kg block 1 section after the


system is set in motion
v = 0 + at
g
= 1
3
g
(upward)
=
3

As the two are just equal, the string will


1
again become taut after time s.
3
(i)

F + 1 g T = 1a

4.

On stopping 2 kg, the block of 1kg will go


upwards with retardation g. Time ( t )
taken by the 1 kg block to attain zero
velocity will be given by the equation.
g
0 = + ( g) t
3
1

t = s
3

T
T

1g

2g

If the 2 kg block is stopped just for a


moment (time being much-much less than
1
s), it will also start falling down when
3
the stopping time ends.
1
In t = s time upward displacement of
3
1 kg block

and

(ii)

T 2 g = 2a

Adding Eqs. (i) and (ii),


F 1g = 3a
20 10 10
ms 2

a=
=
3
3

Introductory Exercise 5.4


(i)

2T = 2 a

1.
a
2 kg

Acceleration of 1 kg block
2 g 20
2a =
=
ms 2
3
3
Tension in the string
g 10
N
T= =
3
3

2T 2T
T
T
2a
1g

and

1 g T = 2a

Solving Eqs. (i) and (ii),


g
a=
3

(ii)

(i)

Mg T = Ma

2.

T
T

M
a

T
T
a
mg

Laws of Motion
(ii)

T = Ma
Solving Eqs. (i) and (ii)
g
a=
2
and
T = Mg

T
T
M

or

Substituting value of above value of T in


Eq. (i),
24
M=
5
= 4.8 kg
T
4.
= m1 2a
2

3. Block of mass M will be at rest if

T/2

Mg

T/2
T

2a

T/2 T/2

T/2

T/2

i.e.,

T/2
T/2

79

g
T = 4 g +

5
24 g
=
5

(ii)

F T = m2 a

or

3g

or

2g

T = Mg

(i)

T = 4 m1a

(i)

For the motion of block of mass 3 kg


T
(ii)
3g
= 3a
2
For the motion of block of mass 2 kg
T
(iii)
2 g = 2a
2
Adding Eqs. (ii) and (iii),
g = 5a
g
i.e.,
a=
5
Substituting above value of a in Eq. (iii),
T
= 2 ( g + a)
2

F 4 m1a = m2 a
F
a=
4 m1 + m2
=

0.40
( 4 0.3) + 0.2

0.40
1.4
2
= ms 2
7
=

T = 4 m1a
= 4 0.3
2.4
7
12
N
=
35

2
7

80 | Mechanics-1
Introductory Exercise 5.5
1 2
at
2
Displacement of block at time t relative to
car would be

1. Block on triangular block will not slip if


m1a cos

2. (a) Using s = s0 + ut +

m1a
N sin

m1 g

m1g sin
cos

m1a sin

a = 5i ms2

a = 5i ms2
u = 10i ms1

v=0

(at t = 0)

T
N

Car

x0

1
( 5) t2
2
x = x0 + 10 t 2.5 t2
x = x0 + 10 t +

M
Mg

m1a cos = m1 g sin


i.e.,
(i)
a = g tan
N = m1 g cos + m1a sin (ii)
For the movement of triangular block
(iii)
T N sin = m2 a
For the movement of the block of mass M
(iv)
Mg T = Ma
Adding Eqs. (iii) and (iv),
Mg N sin = ( m2 + M ) a
Substituting the value of N from Eq. (ii) in
the above equation
Mg ( m1 g cos + m1a sin ) sin
= ( m2 a + Ma)
i.e., M ( g a) = m1 g cos sin
+ ( m2 + m1 sin2 ) a
Substituting value of a from Eq. (i) in the
above equation,
M (1 tan ) = m1 cos sin
+ ( m2 + m1 sin2 ) tan
m cos sin + ( m2 + m1 sin2 ) tan
M = 1
(1 tan )
Substituting = 30 , m1 = 1 kg and
m2 = 4 kg
(cos 30 sin 30+4 + sin2 30 ) tan 30
M=
(1 tan 30 )
0.443 + ( 4.25) (0.577)
=
0.423
= 6.82 kg

or

Velocity of block at time t (relative to car)


will be
dx
v=
= 10 5t
dt
(b) Time ( t) for the block to arrive at the
original position (i.e., x = x0 ) relative to
car
x0 = x0 + 10 t 2.5 t2

t=4s
3. (a) In cars frame position of object at time

t would be given by
In cars frame
(at t = 0 s)
O x0
z0
a = 5 ms2

a = 5i ms2
v=0

u = 10k ms1

1
( 5) t2
2
i.e.,
(i)
x = x0 2.5 t2
and
(ii)
z = z + 10 t
Velocity of the object at time t would be
dx
(iii)
vx =
= 5t
dt
dz
and
(iv)
vz =
= 10
dt
(b) In ground frame the position of the
object at time t would be given by
x = x0 + 0 t +

Laws of Motion

Net deceleration = (3 + 5) m/s


= 8 m/s 2
Displacement of object at any time t
(relative to car)
1
x = x0 + 10t + ( 8) t2
2
or
x = x0 + 10t 4 t2

In ground frame
O x0

At t = 0
a = 5i ms2
v=0

z0
u = 10k ms1

Thus, velocity of object at any time t


(relative to car)
dx
vx =
= 10 8 t
dt

x = x0
z = z0 + 10 t

and

Velocity of the object at time t would be


dx
vx =
=0
dt
dz
and
vz =
= 10 ms 1
dt

The object will stop moving relative to car


when
10 8 t = 0 i.e., t = 1.25 s

vx = 10 8 t for 0 < t < 1.25 s

4. m = 2 kg
O

For block not to slide the frictional force


( f ) would be given by

u = 10i ms1

x0

a = 5i ms2
v=0

a = 3 ms2

ma
mg sin

= 37

Normal force on object = mg


Maximum sliding friction = s mg
= 0.3 2 10 = 6 N
6
Deceleration due to friction = = 3 m/s2
2
Deceleration due to pseudo force = 5 m/s2

f + ma cos = mg sin
f = mg sin ma cos
3
4
= m 10 m 3
5
5
18 m 9
=
=
mg
5
25

or

AIEEE Corner
Subjective Questions (Level 1)

T1
and T2 =
2
T
At point Q, F2 = 1 and W =
2

1. FBD is given in the answer.

At point P, F1 =

2. FBD is given in the answer.


3. FBD is given in the answer.
4.

5.
T2
F1

81

NB
45
T1

30
T1
45

NA
F2

N A sin 30 = N B
and N A cos 30 = W

T1
2
T1
2

82 | Mechanics-1
3T
=W
2
T
H=
2
Net moment about O = zero
l
3T
.
W =
l
2
2
100 40
10. (a) a =
= 3 m/s2
6 + 4 + 10

6. T2

V +

45
T1

T2
= W and
2
T2
= T1
2

(c) N 40 = F10 = 30

N = 70 N.
F
60
11. a =
=
= 1 m/s2
m1 + m2 + m3 60
(a)

40 N
x
N
10 cm

N = 40
f = 20
f 10 = N x

(i)
(ii)
(iii)

T1 = m1a = 10 N
T2 T1 = m2 a

T2 10 = 20

T2 = 30 N
(b) T1 = 0 . New acceleration
60
a =
= 1.2 m /s2
50
T2 = m2 a = 24 N
12. (a)

8.

T1

30

0.1 kg

1.9 kg

2g

T1 2 g = 2a
30

(b)

T2

9.

T + f cos 30 = N sin 30

(i)

N cos 30+ f sin 30 = W


T R = fR

(ii)
(iii)

3T
2
H

T/2
W

(iii)

F6 = 18 N, F4 = 12 N and F10 = 30 N

and

(ii)

(b) Net force = ma

7.

W = 20 N

(i)

2.9 kg
a

0.2 kg
1.9 kg

T2 5 g = 5a

5g

Laws of Motion
200 16 g
13. (a) a =
16
(b) T1 11 g = 11a
(c) T2 9 g = 9 a

20.

83

a
T2 = T1
1
T1

14. If the monkey exerts a force F on the rope


upwards, then same force F transfers to
bananas also. If monkey releases her hold
on rope both monkey and bananas fall
freely under gravity.

T1

a
2

2T1

15. Tension on B = T
Tension on A = 3 T
1
Now in these situations a
T

T1
ar

T1
2

16. x A + xC + 2xB = constant.

Differentiating twice w.r.t. time we get the


acceleration relation.

aA

2T
a
2

aA
= sin
aB

g
M

a A = aB sin

21.

aB

(i)
(ii)
(iii)

T1 = 1a
T1 20 = 2 ( ar a / 2)
30 T1 = 3( ar + a / 2)

17.

18. x + y = 6

30

2M

2 Mg

(i)

T Mg sin 30 = Ma
a
2Mg 2T = 2M
2

(i)

yx=4
Solving, we get
x = 1 m/s2

n
si

(ii)

(ii)

22. T = 1a
10 T = 1a

23.

ar

(i)
(ii)

2T

2T

50

40

1
x
2a

a
y
2
y
3

19.

7g 3g
= 4 m/s2
10
40 T1 = 4 a
30 + T1 T2 = 3 a
T3 10 = 1a

a=

50 2T = 5a
T 40 = 4(2a)
24. (a) N = 40 N, s N = 24 N
F < sN
f = 20 N and a = 0
(b) N = 20 N, s N = 12 N
and k N = 8 N

(i)
(ii)

84 | Mechanics-1
F > sN
f = kN = 8 N
20 8
a=
= 6 m/s 2
2

and

27. f = (06
. )(2)(10) = 12 N
a2
2 kg

1 kg

s N = 8 N and
Since, F cos 45 > s N

and

f = kN = 4 N
20 4
a=
6
8
= m/s2
3

25. a = g = 3 m/s2
(a) v = at
6 = 3 t or t = 2 s
1
(b) s = at2
2
1

s = 3 4 = 6 m.
2
26. f = 0.4 1 10 = 4 N
a1
1 kg
f
a2
2 kg
f

a1 =

f
= 4 m/s2
1

a2 =

f
= 2 m/s2
2

(a) Relative motion will stop when


v1 = v2
or
2 + 4 t = 8 2t

t = 1s
(b) v1 = v2 = 2 + 4 1 = 6 m/s
1
(c) s1 = u1t + a1t2
2
1
s2 = u2 t a2 t2
2

+ve

a1

(c) N = 60 20 = 40 N
kN = 4 N

ve

f
f

12
= 6 m/s2
2
12
and a1 =
= 12 m/s2
1
(a) Relative motion will stop when
v1 = v2
or
u1 + a1t = u2 + a2 t
or
3 6t = 18 + 12t
7

t= s
6
(b) Common velocity at this instant is
v1 or v2 .
1
(c) s1 = u1t + a1t2 and
2
1
s2 = u2 t + a2 t2
2
a2 =

28. N = 20 N
s N = 16 N
and k N = 12 N
Since, W = 20 N > s N, friction k N will
act.
20 12

a=
= 4 m/s2
2
29. N = 20 N
N = 16 N
Block will start moving when
F = N
or
2t = 16
or
t = 8 s.
After 8 s
2t 16
a=
= t8
2
i.e., a-t graph is a straight line with
positive slope and negative intercept.
30. N = 60 N, s N = 36 N, k N = 24 N
Block will start moving when
F = sN

Laws of Motion
or

(b)

4 t = 36
t=9s

85

12 N

After 9 s
a=

4 t 24 2
= t4
6
3

52 N

F + 12 = 52
F = 40 N

31. N = mg cos = 30 N

mg sin 30 = 30 3 N 52 N.
s N = 18 N and
k N = 12 N
F
(a)

(c)

52 N

18 N

12 N

F 52 12 = 6 4

F = 88 N

52 N

F = 52 18 = 34 N.
Objective Questions (Level-1)
Single Correct Option

mg F
F
=g
m
m
m A > mB

a A > aB
or ball A reaches earlier.
4 g 2g g
2. a =
=
6
3
g
Now, 2 g T = 2
3
4g

T=
= 13 N
3

mg T = ma

1. a =

Tmax
m

2
mg
g
=g3
=
m
3
10 g 5 g g
5. a =
=
15
3
g
10 g T = 10
3
20 g

T=
= Reading of spring
3
balance.
2mg sin 30 mg
6. a =
=0
3m

T = mg

T1

3.

amin = g

30
T2

7. a1 = g sin g cos = g sin 45 g cos 45


100 N

3 T1
= 100
2
T1
= T2
2
4.

T
a

mg

(i)
(ii)

a2 = g sin = g sin 45
2s
1
Now t =
or t
a
a
t1
a2

=
t2
a1
g sin 45
g sin 45 g cos 45
3
Solving, we get =
4
or

2=

86 | Mechanics-1
8. F1 = mg sin + mg cos

14. f = mg sin

F2 = mg sin mg cos
Given that F1 = 2F2

(if block is at rest)

15. 2T cos 30 = F
60

9. For equilibrium of block, net force from


plane should be equal and opposite of
weight.

T
30

10. No solution is required.


11. Angle of repose = tan 1( ) = 30

F
3
T cos 60
a=
m
F
=
2 3m

T=

16.

h = R R cos = 1
R
2

12. Net pulling force = 15 g 5 g = 10 g = F


Net retarding force = (0.2)(5g) = g = f
Ff
9

a=
=
g
25
25
9
T1 5 g = 5a = g
5
34

T1 =
g
5
27
15 g T2 = 15a =
g
5
48
g

T2 =
5
T1 17

=
T2 24
13.

N = mg F sin
N = (tan ) N = (tan )( mg F sin )
F cos = N
17. mg = 0.2 4 10 = 8 N
At t = 2 s, F = 4 N
Since F < mg
Force of friction f = F = 4 N
18. a1 = g sin
a2 = g sin g cos
t=

t1
=
t2

1
=
2

Relative to lift, ar = ( g + a) sin along the


plane.
2s
Now, t =
ar
2L
=
( g + a) sin

25
1
or t
a
a
a2
a1

g sin g cos
g sin

19. Net accelaration of man relative to ground


= a + a = 2a
T mg = m (2a)

T = m( g + 2a)
20.

a=0
3 F = (50 + 25) g = 75 g
F = 25 g = 250 N

Laws of Motion
21.

N1

1 + 2

3m

N2

or

22.

tan =

1 + 2

mg

f = 30 N
30 N > mg
Normal reaction N = mg = 40 N

N2 = W = 250
fmax = N2 = 75 N

1
Mg
g
2
=
M
2

Mg

(always)

Net contact force = (30)2 + ( 40)2


= 50 N
26.

2 kg

0.2

8 kg

25 N

F
f

1 + 2

Since

25. mg = 32 N

N2

1 + 2
1
= mg
2

N1

mg

Fmin =

N1 = N2
N2 = W
Net moment about B should be zero.
3

W = N1 4
2

23. a =

N2

4m

87

1 M
M
Mg

g=
a=
2 2
2
4
Mg
T=
2
F

24.

N = mg F sin
F cos = N = ( mg F sin )
mg
F=
cos + sin
dF
For F to be minimum,
=0
d

0.5

fmax = (0. 5)(8 + 2)(10) = 50 N > 25 N


Blocks will not move and therefore force
of friction between two blocks = 0.
3
27. mg sin = 10 10 = 60 N
5
This 60 N > 30 N
Force of friction is upwards.
Net contact force is resultant of friction
and normal reaction.
28. In first figure T = F
and in second figure, T = 2F .
mg
29. F = 4
upwards
2
W = 2mg downwards
F =W

a=0

88 | Mechanics-1
30.

t=

m
m

2 m g
a

31. a1 = g sin 30 = g/2

F = at

a2 = g sin 60 =

Maximum acceleration of upper block due


to friction
mg
amax =
= g
m
F
at
=
= g
2m 2m

3
g
2

Angle between a 1 and a 2 is 30.

ar = |a 1 a 2|=

( g/2)2 + ( 3 g/2)2 2 ( g/2)


( 3 g/2) cos 30

= g/2
JEE Corner
Assertion and Reason

1. Even if net force = 0, rotational motion can


take place.
2. No solution is required.
3. a1 = g sin + g cos
a2 = g sin g cos
a1 1 +
as = 45 and sin = cos
=
a2 1
1
we have,
3
a1 1 + 1 / 3 2
=
=
a2 1 1 / 3 1

Substituting =

T = m1( g a)
2m1m2 g
=
( m1 + m2 )

Mathematically, we can prove that


m2 g < T < m1 g
Similarly if m2 > m1, then we can prove
that,
m1 g < T < m2 g

4. There is no force for providing ( 2 i$) m/s2


to the block.
5. If we increase F1, maximum value of
friction will increase. But if we increase F2
friction acting on the block will increase.
6. No solution is required.
( m1 m2 ) g
7. a =
m1 + m2

m1 g T = m1a

(if m1 > m2 )

8. If accelerations of both the frames are


same then one frame as observed from
other frame will be inertial.
Further, a frame moving with constant
velocity is inertial.
9. No solution is required.
10. No solution is required.
11. Force of friction is in the direction of
motion.

Laws of Motion

89

Objective Questions (Level 2)


Single Correct Option
3. N sin will accelerate B towards left.

1. Tmax = Fmax + fmax + 2T max


A
T

T'

T'

T T

2T

N sin

90
ma (pseudo
force)

T' T'

mg
co

T
mg

= ( m A + mB ) g + m A g + 2m A g
= ( 4m A + mB ) g
But,
Tmax = M max g

M max g = ( 4 m A + mB ) g
i.e.,
M max = ( 4 m A + mB )
= 0.3 [( 4 100) + 70] = 81 kg
Option (c) is correct.

Option (b) is correct.


4. If a g

v = 8 t ^i 2 t2 ^j

m2

m1

a=

Let a be the acceleration of B.


Due to acceleration of B towards left
pseudo force equal to ma will act on block,
toward right.
Thus,
N + ma sin = mg cos

N = mg cos ma sin

2.

dv
dt

m2a

m1a

= 8 i 4t j
y

m1

2i
4j

Force (pseudo) on sphere

Fs = 1 ( 8 ^i + 4 ^j )
= 8 ^i + 4 ^j
Gravitational force on sphere
= mg ^j = 10 ^j
Net force on sphere

F = 8 ^i + 4 ^j 10 ^j = 8 ^i 6 ^j

|F |= 10 N
Option (b) is correct.

N=0
f2

f1

N=0

m2

When a is f1

f2

Zero
g
2

Zero
g
m1
2

Zero
Zero

Zero
g
m2
2

m1 g

Zero

m2 g

(Not yet
attained
limiting
value
which is
2 m1 g)

(Attained
limiting
value)

Now, if a > g, the block of mass m2 would


be just greater like to move towards left
than g as a then
(max)
m2 a > f2
While the block of mass m1 will remain at
rest as then f1 has margin to increase.
Thus, m1 will apply normal force ( N ) or m2
and so will do m1 on m2 to stop its motion
(towards left).
Option (d) is correct.

90 | Mechanics-1
Fig. 3

5. N + ma sin = mg cos
N

T
a3

ma sin
ma (pseudo
force)

in
gs

m1

mg sin mg

The block will fall freely if N = 0


i.e.,
ma sin = mg cos

a = g cos

and

Option (c) is correct.


6. Speed of A w.r.t. C = v1

Speed of C w.r.t.
ground = v0
Speed of A w.r.t.
ground = v1 v0
Now, speed of B w.r.t.
ground = speed of A is
w.r.t. ground

v2 = v1 v0

v1 v2 = v0
Option (a) is correct.

7. Fig. 1
m1

T
T
T

a3

m2g

m2 g T = m2 a3
T m1 g sin = m1a3
m m1 sin
a3 = 2
g
m1 + m2

Substituting m1 = 4 kg, m2 = 3 kg
and
= 30
3
a1 =
g
4+3
3
= g
7
43
a2 =
g
4+3
1
= g
7
1
3 42
2g
a3 =
4+3
1
= g
7

a1 > a2 = 3
Option (b) is correct.
8. F fmax T = ma (For lower block)

m2g

and

T
T

T
mg
2
mg
1

T fmax = ma (For upper block)

F 2 fmax = 2ma
or
F 2 mg = 2ma
F
or
a=
g
2m
Option (c) is correct.

Fig. 2
and

m2 g T = m2 a1
m2
a1 =
g
m1 + m2
m1 g T = m2 a2
T m2 g = m1a2
m m2
a2 = 1
g
m1 + m2

a2

a1

9. a2 cos(90 ) = a1

i.e., a2 sin = a1 [ < 0]


Option (b) is correct.

a2

Laws of Motion
10. Had been 90.

v1
v1 90
m

v2

B
w

T = m ( g + a)
Mm

= m g +
g
M + m

M m

= mg 1 +
M + m

ma cos

12. At the position of maximum deflection the

net acceleration of bob (towards its mean


position will be)
g sin a cos
as explained in figure.

10
20

F (force of friction)
mg sin 10 = 0.174 mg
mg sin 20 = 0.342 mg

30
40

mg cos 30 = 0.500 mg
mg cos 40 = 0.442 mg

60
90

mg cos 60 = 0.287 mg
Zero

a cos
O
a
90

T mg = ma

T
m

= 30

ma cos > mg sin


i.e.,
a > g tan
amin = g tan [cylinder will be at the
point of rising up the inclined plane]

13. Mg T = Ma

For frictional force ( F ) = mg cos


[ = angle of repose]
For example
1
1
Let =

tan =
3
3

inclined plane if

2T
2T

14. For 0 < frictional force ( F ) = mg sin

11. The cylinder will start rising up the

ma
(Pseudo
furce)

91

mg
= mg [1 + 1]
{as m << M , M m M and
Mg
M + m M}
= 2 mg
Tension in the string suspended from
ceiling = 2T = 4 mg
Option (a) is correct.

For every x displacement of wedge ( w) the


vertical fall in mass would have been 2x as
the string passes through pulley B.
i.e.,
v1 = 2 v2
For the situation given the speed of mass
would thus be
v1 = 2 v2 cos (90 )
i.e.,
v1 = 2 v2 sin
Option (c) is correct.

Mm
a=
g
M+m

g cos (90 )
= g sin

90

Option (b) is correct.


mg
15.
aB =
2m
g
=
2

Condition
Increase but not
linearly only
Decrease but not
linearly only

92 | Mechanics-1
mg

F F

Am

cos
ma
F

B
mg

2m

F
F

mg

For no slipping
a A = aB
F mg g
=
m
2
3
i.e.,
F = mg
2
2 F
=
3 mg

16. Fnet (downward) = mg sin + ma cos


a
ma

21

3
1
10 3
_ 10
2
2

1
s
=
5

t=

Option (b) is correct.


w
w
18. N + sin 30 = w +
2
2

ma cos
mg sin

= m ( g sin + a cos )
g eff = g sin + a cos

Time ( T) required to cover 2L distance


along inclined would be
2L
T=
g eff
2L
( g sin + a cos )

Option (c) is correct.


17. Fnet on block along incline in the upward

direction

Substituting s = 1 m,
= 30,
a = 10 3 m/s 2 and g = 10 m/s 2

For no slipping.
Option (c) is correct.

sin

Time ( t) to move s distance would be


given by
1
s = g eff t2
2
2s
2s
i.e., t =
=
g eff
( a cos g sin )

Slipping will obviously be m2 there if is


greater than above mentioned value
2 F

min =
3 mg

= ma cos mg sin
= m ( a cos g sin )
g eff = a cos g sin

w/2

w sin 30
w/2
2
30

w cos 30
2

f
w

f = frictional force
5w
4
5w
fmax = N =
4
The block will remain stationary if
5w
w
cos 30
2
4
w 3
5w
or

2 2
4
or
3 5
3
or

5
3
Block will move if <
5
Option (d) is correct.
N=

Laws of Motion |
T

19. F1 = 2t
f1

F = 15N

20. 2 mg sin T = 2ma


y

f2 2 = 0.5

1 = 0.6

60

At t = 2 s
F1 = 4 N
4N

P
T

2m

T
T

15N

= 30

f1 and f2 are the frictional forces

and

( f1) max = 0.6 1 10


= 6N
= 10 N

At t = 2 s

Force ( R) applied by clamp on pulley


would be

Net external force ( Fnet ) on system


= 15 N 4 N

= 11 N
As Fnet > ( f1) max + ( f2 ) max , the system will
remain at rest and the values frictional
forces on the blocks will be given
T = 4 + f1 and T = 15 f2
4 + f1 = 15 f2

(i)

f1 + f2 = 11 N

|T1|= |T2|= T

mg
mg
T1 =
(cos 30 ) ^i +
(sin 30 ) ^j
3
3
mg 3 ^ mg ^
=
i+
j
6
6

mg ^
|T2|=
j
3
Force by clamp on pulley P
y
T2

Let direction being + ive for Eq. (i)

T1

Option (a) f1 = 4 N, f2 = = 5 N
f1 + f2 = 1 N

wrong

60
x

Option (b) f1 = 2 N, f2 = + 5 N
f1 + f2 = 3 N

wrong

Option (c) f1 = 0 N,f2 = + 10 N


f1 + f2 = 10 N

wrong

Option (d) f1 = + 1 N, f2 = + 10 N
f1 + f2 = 11 N correct.
OR
As the likely movement would be towards
right f2 will be at its maximum.

f2 = 10 N

f1 = 1 N

Option (d) is correct.

T = ma
2 mg sin = 3 ma
2 g sin 30 g
a=
=
3
3
mg
T=
3

( f2 ) max = 0.5 2 10

T
T

93

T1

T2

mg 3 ^ mg ^ mg ^
i+
j+
j
6
6
3
mg 3 $ 3 mg $ mg
=
i+
j =
( 3 ^i + 3 ^j)
6
6
6
=

Option (b) is correct.


21. f1(max) = 0.3 4 10 = 12 N
4 kg

2 kg
f2

1 = 0.6

f1

2 = 0.3

f1 and f2 are frictional forces.


f2 (max) = 0.6 2 10 = 12 N
As, f1(max) + f2 (max) < 16 N ( Fext )

16N

94 | Mechanics-1
T

2 kg

f2

4 kg

16N

x=

f1

The system will remain at rest.


For the equilibrium of 4 kg mass :
(i)

16 = T + f1
As f1 will be at its maximum value
f1 = 12 N

T = 16 12
= 4 N [from Eq. (i)]
Further, for the equilibrium of 2 kg mass.
T = f1

f1 = 4 N
Option (c) is correct.
22. For the rotational equilibrium of rod

Option (a) is correct.

A
ma
mg

(i)

25. N = ma sin + mg cos


N
ma sin
ma
a
mg sin

mg cos

Now, as the block does not slide


ma cos = mg sin
i.e.,
a = g tan
Substituting the found value of a in Eq. (i)
N = m ( g tan ) sin + mg cos
sin2

= mg
+ cos = mg sec
cos

Taking moment about O.


l
l
R cos = s sin
2
2
s (= ma)
R (= mg)
O

1
m
2

When, the block stops a = 0, the value of


normal force will be
N = mg cos
N mg cos

=
N
mg sec

or
mg cos = ma sin

a = g cot
Option (d) is correct.

Option (c) is correct.


26. For the rotational equilibrium of the block
N

23. v = 2 t

dv d
=
(2 t2 ) = 4 t
dt dt
At t = 1 s, a = 4 ms 2
As
a = sg
a
4
s = =
= 0.4
g 10
a=

A
mg

be zero.
P

R2
8m

1m
10 g

x
1m

Taking moment about point Q


(10 g) ( 4) = (80 g) ( x)

mg cos

= 45

24. Just at the position of tipping off, R1 will

mg sin

Option (c) is correct.

R1

Taking moment about O.


a
Nx = f
2
a
or
( mg cos ) x = ( mg sin )
2
a
or
x = tan
2
x
or
= tan
a/2
or
or

tan = tan
=

Laws of Motion |
Thus, the normal force ( N ) will pass
through point A.

9 ms2

Option (a) is correct.

[Note : The cube will be just at the point of


tilting (about point A). The cube will tilt if is
made greater than 45].

N sin

Moment of couple ( N , mg)


= Moment of couple ( F , f )

mg
F (= 3 )

f=F

mg

Option (b) is correct.

aA

aA = | a A |

28. Taking moment about point O.


N1

l/4

l/6

mB a A cos
sin tan
mB a A
=
tan2
10 9
=
= 160 N
2
3

4

5
37
4

31. Net downward force on ring = mg ma

= m ( g a)
T

N = ma

f = N
= ma

a = 4 ms2
L = 1m

mg

ma

mg

( T / 2) = ma Box
ma mg
T+
=
Pendulum with
2
2

respect to
box
ma mg
2ma +
=
2
2
a= g/3

Option (c) is correct.

a
T

aB

N cos =

mg

l l
l
N1 = N2
2 4
2
l
l
N1 = N2
4
3

N1 : N2 = 4 : 3
Option (c) is correct.

aB = | a B |

N2

29.

(ii)

a A = aB tan
aA
N sin = mB
tan
mB a A
N=
sin tan
Force on rod by wedge

N (= mg)

= 37 B

27. For the rotational equilibrium of the cube

mgx = Fa
mg
or mgx =
a
3
a
i.e., x =
3

(i)

N sin = mB aB

30.

95

g eff = g a
2L
t=
g eff
=

2L
g a

21
10 (0.5 4)

1
= 0.5 s
2

96 | Mechanics-1
32. The direction of the normal reactions

33. T Mg = Ma

between any one hemisphere and the


sphere will be along the centres of the two.
The three centres of the hemisphere and
that of sphere will form a tetrehadron of
edge equal to 2R.
In figure, C1, C2 and C3 are the centres of
the hemispheres and C is the centre of the
sphere
C1C2 = C2C3 = C3C1
= C1C = C2C = C3C = 2R
N

C3

C2

COC2 = 90
2R
C2O =
3
C2O
cos =
C2C
2R / 3
=
2R
1
=
3
is the angle which any N makes with
vertical
= 90
3
1
sin = cos =
1
3

2
cos =
2
3
For the vertical equilibrium of the sphere.
3 N cos = mg
2
or
3N
= mg
3
mg
or
N=
2 3
Option (b) is correct.

( 500 + 80n )g

i.e.,

N
N

C1

T = M ( g + a)

2 104 M ( g + a)

or

2 104 (500 + 80n)(10 + 2)

or

14.58 n

or

n = 14

Option (b) is correct.


[Note : Tension in lift cable will increase
when the lift is accelerated upwards].
34. Normal reaction between the surface and

the particle will be zero throughout the


motion if the path of the particle is that of
a projectile motion (particle is free from
surface).
v sin

u sin u

v cos

a=g

+ u cos

v2 = u2 + 2as
(v sin )2 = (u sin )2 + 2 ( g) h
v sin = u2 sin2 2 gh
= (20)2 (sin2 60 )2 2 10 5
3
= 400 100

4
= 10 2
v cos = u cos = 20 cos 60 = 10
v sin 10 2
=
v cos
10

Laws of Motion |
tan = 2
= tan 1 2
Option (c) is correct.
35. Acceleration

of block B will be g
throughout its motion while that of block
A will increase from 0 to g and as such
t A < tB
Option (b) is correct.

vertical

Option (c) is correct.


38. String is winding on the motor shaft the

36. f1 (max) = 0.5 10 10 = 50 N


100 N

= 0.5f1

Velocity along PQ = 20 m/s.


Velocity along PO = 20sin
3
= 20 = 12 m/s
5
Velocity of sphere (along
direction)
12
12
=
=
= 15 m/s
cos 4 / 5

97

= 0.25f2

block B will move up. Further, as shaft is


also moving down, B will further.

f1

2 m/s
f2

Here, f1 and f2 are friction forces.


As, f1 (max.) < Fext . (100 N), block A will
move.
f2 (max.) = 0.25 (10 + 20) 10 = 75 N
As, f1 (max.), [driving force for block B]
< f2 (max.), the block will not slop over
block C.
As, there is no friction between block C
and surface below it, both the blocks B and
block C will move together with
acceleration
f1 (max.)
a=
(mass of B + C)
50
=
(20 + 30)

= 1 ms 2
a A = 1 ms 2

2 m/s

Thus,
Velocity of block B
= Velocity of lift + Velocity of winding of
string on shaft + Velocity of moving down
of shaft
q= 2 m/s + 2 m/s + 2
m/s
= 6 m/s.
Option (d) is correct.
39. F = F cos

F is resultant of two equal forces F and F


2
F = 2F cos
2

Option (c) is correct.


R
R
5 =4
37. cos =
R
5

F'
20 m/s
N
O

R/5

F''

F''

= 2F cos
= 2F cos (90 )
= 2F sin
F
= 2
sin
cos

5
4

98 | Mechanics-1
= 2F tan
= 2( ma ) tan
5 4
= 2 03. = 2 N
2 3

For A to remain in contact with B, B must


accelerate in the ive x-direction.

aA (new)

15 j

Option (b) is correct.

40. Velocity of block A = 2 v cos 37

bi

15 i

Let acceleration of B = b ^i

2v

Due to this pseudo force with act on A in


the + ive direction.
15
tan =
15 + b
3
15
or
=
4 15 + b

37
A
v

= 2 10

4
5

= 16 m/s
Option (d) is correct.
41. As

the mass is applying maximum


possible force without moving, the blocks
would at the point of slipping,
fA
T' T'

T T

fC

fB

= 45

As, < the block will leave contact with


B.
aA

B
2F

A
N sin

N sin

Fnet on block A = Fnet on block B


F N sin = N sin 2F
2 N sin = 3 F
3F
N=
2 sin
(as = 30)

= 3F
Option (d) is correct.

a A = |5 ^i + 15 ^j|

15 j

Option (d) is correct.

Option (d) is correct.


[Note : The value fB will be 300 N and the
values of T and f A will be zero]

Acceleration of B = 5 ^i

T = fC (max) = 0.5 60 10 = 300 N


fB (max) = 0.3 (60 + 60) 10
= 360 N
As, T < fB (max) the value of T will be
zero.

42.

45 + 3 b = 60
3 b = 15
b= 5

43. For block A not to slide on block B

fAT T

or
or
or

44. Equation to circle is


Y
(x, y) B

15 i
x

B
= 37

A
3
tan = tan 37=
4

Laws of Motion |
2

x + y = r (where r = OB)
dx
dy
2
+2
=0
dt
dt
dx
dy
=
dt
dt

(i)

l1

99

l2
2T
2T

M1

= ( u) = + u
=u

Speed of bead B
2

dx
dy
= +
dt
dt
2

M3g

M2 g

M1 gl1 = 2Tl2
2M1M 3
M1 gl1 = 2
g l2
M2 + M 3

= (u) + ( u)

or

=u 2

M2

M2

3 3l
M1l1 = 4
1
M2 + M2

3
M1
=3
M2

Option (a) is correct.


45. At maximum acceleration value of a, the

or

block would be in a position to move


upwards.
ma cos

47. fmax = k N 2

ma
ma sin

N2
mg sin
fm

mg cos

ax

Trongh

= 45

f = frictional force
N = ma sin + mg cos
ma cos = fmax + mg sin
or

ma cos = N + mg sin

or

ma cos = ( ma sin + mg cos )


+ mg sin (Q = 45 )

or

a = ( a + g) + g

or

a (1 ) = (1 + ) g
1+
a=
g
1

i.e.,

Block
N = mg cos

NN

Option (b) is correct.


46. For the beam to have no tendency to

= k 2 mg cos
ma = mg sin 2 k mg cos
i.e., a = g (sin 2 k cos )
Option (c) is correct.
48. f1 (max) = 0.5 3 10

= 15 N
f2 (max) = 0.3 (3 + 2) 10
= 15 N
F
3kg
f1

f1
2kg

f2

f2
1 kg

B
f3

rotate
f3 (max) = 0.1 (3 + 2 + 1) 10
=6N

100 | Mechanics-1
Value of maximum frictional force is
between block 1 kg and the ground.
Increasing from zero when F attains 6 N,
the block of mass 1 kg will be at the point
of slipping over ground below it.
Option (c) is correct.
49. f2 (max) = ( m1 + m2 ) g
m1
1

f1
m2
f2

f1
F (= 30N)

Option (a) is correct.


50. F = s mg cos mg sin

4
3 1
= mg

2
3 3 2
mg
=
6
Option (b) is correct.

51. s = 2 k
F

2
s
mg

= 0.5 (1 + 2) 10
= 15 N
aS = Acceleration of both as one
a A = Acceleration of A
f1 (max) = 1m1 g = 0.2 1 10
= 2N
F f2 (max) 30 15
aS =
=
= 5 m/s2
m1 + m2
3
m g
a A = 1 1 = 1 g = 0.2 10 = 2 m/s2
m1
As,
F > f2 (max.)
both will move.
Further, as aS > a A both will
accelerate as one unit.
F 2 ( m1 + m2 ) g 1m1 g
aB =
m2

2s
2.1
2
=
= s
a AB
9/2 3

Required time t =

in

SN

= 30

Force required
downwards
a=

to
0

in
gs
m
2

just

slide
F

KN

= 30

not

Acceleration of A w.r.t. B
a AB = a A AB
f _ 2 ( m1 + m2 ) g 1m1 g
= 1 g
m2
1m2 g F + 2 ( m1 + m2 ) g + 1m1 g
=
m2
F ( 1 + 2 ) ( m1 + m2 ) g
=
m2
30 (0.2 + 0.5) (1 + 2) 10
=
2
9
2
= m/s
2
Negative sign indicates that the direction
of a AB will be opposite to that of a A .

F = s N mg sin
= s mg cos mg sin
F = mg sin k N
= mg sin kmg cos
Thus, s mg cos mg sin
= mg sin kmg cos
( s + k) mg cos = 2mg sin
+ s = 2 tan
s

2
3
2
4
s =
s =
2
3
3 3
Option (a) is correct.
F + mg sin k mg cos
52. a =
m
mg mg
2
3
+

mg
2
3 3
2
= 6
m
g
=
3
Option (d) is correct.

block

Laws of Motion |
53. Minimum force required to start the

For retarted motion

motion upward
= mg sin + k mg cos
1
4
3
= mg +

2 3 3 2
7
= mg
6

02 = v2max 2a s2
v2
s2 = max
2a
v2max 1 1
s1 + s2 =
+
2 a a
v2 8 6
25 = max +
2 g g

54. Minimum force required to move the block

up the incline with constant speed


= mg sin + k mg cos
1
2
3
= mg +

2
3
3
2

5
= mg
6
(5.22)2
55. S1 =
= 14.3 m
9.8
2

vmax =

50 9.8
= 5.92 ms 1
14

Option (c) is correct.


57. mg sin = 170 10

8
= 906.67 N
15
= 0.4
T

= 0.2
mg

Option (c) is correct.


1200 g 1000 g
56.
a =
1200
g
=
6

101

sin

si
mg

f2

f1

15
= 300 N
17
15
f2 (max) = 0.4 170 10
17
= 600 N
f1 (max) = 0.2 170 10

17
T' = 1000g

a'

Speed
= vmax
T = 1350g

15
25 m

1200 g

1200 g

1350 g 1200 g
1200
g
=
8

a=

For accelerated motion

Stops

v2max = 02 + 2as1
v2
s1 = max
2a

The whole system will accelerate as


mg sin is greater than both f1 (max) and
f2 (max).
Total force of friction
= f1 (max) + f2 (max)
= 900 N
Option (a) is correct.
58. mg sin 300 T = ma

and mg sin + T = ma

(i)
(ii)

Substituting Eq. (i) by Eq. (ii),


2T + 300 = 0
T = 150 N
= 150 N, compressive.
Option (a) is correct.

102 | Mechanics-1
More than One Correct Options
1. (a) Normal force between A and B = m2 g
F
2N

T
2N

i.e.,
T =3N
Option (d) is correct.
2. At point A

T T

= 1 10 = 10 N
Force of limiting friction by B on A (or by
A on B)
= 10 = 2 N

T1
T1
A
T2

Total force opposing applied external force


F = 2N + T
= 2N + 2N
= 4N
Thus, if F 4 N
The block A will remain stationary and so
block B also. The system will be in
equilibrium.
Option (a) is correct.
(b) If F > 4 N
F T 2=1a
and
T 2 = 1a
Adding above equation
F 4 = 2a
i.e.,
F = 4 + 2a

(i)

T2
B F

mg

(i)

T1 cos = T2 cos + mg
and T1 sin = T2 sin

(ii)

At point B
T2 cos = mg

(iii)

T2 sin = F mg

(iv)

Using Eq. (iii) in Eq. (i),


T1 cos = 2T2 cos

(ii)

=
mg

(v)

Dividing Eq. (ii) by Eq. (v),


(vi)

2 tan = tan
Option (a) is correct.

For F > 4 N

Squaring and adding Eqs. (iii) and (v),


2a + 4 > 4

or

a>0

T 2> 0

i.e.,
T > 2N
Option (b) is incorrect.
(c) Block A will move over B only when
F > 4 N and then the frictional force
between the blocks will be 2 N if a is just 0
[as explained in (b)].
Option (c) is correct.
(d) If F = 6 N using Eq. (ii)
2a = 6 4

a = 1 m/s 2

Using Eq. (i),


T 2=1

T12 = 4 T22 cos2 + T22 sin2


Dividing Eq. (iii) by Eq. (iv)

(vii)

2
1

tan = 1
1
cos =
= sin
2

Substituting the values of sin and cos


in Eq. (vii)
1
1
T12 = 4 T22 + T22
2
2
5
= T22
2

Laws of Motion |

2T1 = 5T2
Option (c) is correct.
3. Displacement of block in 4 s
v (ms1)

t (s)

S = Area under curve


= 16 m.
K =Workdone by frictional force
1
1 42 = 1 10 16
2

= 0.1
Option (a) is correct.
Option (b) is incorrect.
Acceleration, a = tan
= tan ( )
= tan
= 1 m/s 2
If half rough retardation = 0.5 m/s2
1

16 = 4 t + ( 0.5) t2
2
2
i.e.,
t 16t + 64 = 0
or
t=8s
Option (d) is correct.
Option (c) is incorrect.
4. Let acceleration of wedge ( A) = a
ma sin N F = mg cos

ma (Pseudo
force)

N sin F

103

mg

N + ma sin = mg cos
N = mg cos ma sin
Acceleration of
N sin
a=
M
or Ma = ( mg cos ma sin ) sin

or a ( M + m sin ) = mg cos sin


mg cos sin
i.e., a =
M + m sin2
0.6 g cos 45 sin 45
=
1.7 + (0.6 sin2 45 )
3g
=
17 + 3
3g
=
20
Let aB = Acceleration of block B
Net force on B (along inclined plane)
maB = ma cos + mg sin

aB = a cos + g sin
Thus, ( aB ) V = ( a cos + g sin ) cos
= a cos2 + g sin cos
1
= ( a + g)
2
3g
1

=
+ g
20
2
23 g
=
40
( aB ) H = ( a cos + g sin ) sin a
23 g 3 g
=

40
20
17 g
=
40
5. f1 (max.) = 1m A g
T(pull) = 125 N
f1

f1
B

f2

= 0.3 60 10
= 180 N
Fnet on B = f1 (max.) + T
= 180 + 125
= 305 N
A will remain stationary as
T < f1 (max.)

f1 = 125 N
Force of friction acting between A and
B = 125 N
Options (c) and (d) are incorrect.

104 | Mechanics-1
(a)

f2 (max) = 2 ( m A + mB ) g

7.5 N

= 0.2 (60 + 40) 10

17.5 N
F

= 200 N

T
17.5 N

f1 + T = 125 + 125 = 250 N


As, f1 + T > f2 (max. block B /along the A
as A is stationary) will move towards right
with acceleration.
Option (a) is correct.
( f + T) f2 (max.)
aB = 1
mB + m A
=

250 200
40 + 60

Option (b) is correct.

9. N sin = ma = 1 5 = 5
N cos = mg = 1 10 = 10

6. (See solution to Question no. 4).


A

T = 17.5 + 7.5 = 25 N
F = T + 37.5 + 17.5 = 80 N
(c) T 7.5 17.5 = 4 a
F T 37.5 17.5 = 8 a
F = 200 N
Solving these equations we get,
a = 10 m/s 2

m
a

N = mg cos ma sin
Option (c) is correct and option (d) is
incorrect.
As angle between the directions of a and
g sin will be less than 90, acceleration of
block A will be more than g sin .
A

anet > g sin

Option (a) is correct and option (b) is


incorrect.

(i)
(ii)

Solving these two equations we get,


1
tan = and N = 5 5 N.
2
10. Let f1 = friction between 2 kg and 4 kg
f2 = friction between 4 kg and ground
( fs1) max = 0.4 2 10 = 8 N
( f k1) = 0.2 2 10 = 4 N
( fs2 ) max = 06
. 6 10 = 36 N
Fk2 = 0.4 6 10 = 24 N
(b) At t = 1 s, F = 2 N < ( fs2 ) max
Both the blocks are at rest.
f1 = 0

7. Maximum value of friction.


f1 = between A and B
= 0.25 3 10 = 7.5 N
f2 = between B and C
= 0.25 7 10 = 17.5 N
f3 = between C and ground
= 0.25 15 10 = 37.5 N

(i)
(ii)
(iii)

8. Maximum value of friction available to


block is less than the maximum value of
friction available to man.

= 0.5 m/s 2

ma

37.5 N

(c) At t = 4 s, F = 8 N < ( fs2 ) max


Both the blocks are at rest.
f2 = F = 8 N,
11. a = 0, T1 = 10 N,
T2 = 20 + T1 = 30 N,
T3 = 20 N.
12. fmax = 0.3 2 10 = 6 N
(a) At t = 2 s, F = 2 N
f = 2N
(b) At t = 8 s, F = 8 N > 6 N
f =6N

Laws of Motion |
(c) At t = 10 s, F = 10 N and f = 6 N
10 6
a=
= 2 m/s2
2
(d) Block will start at 6 s. After that,
net impulse
1
= 4 (6 + 10) + 2 10 6 6
2
= 16 N-s = mv
16

v=
= 8 m/s.
2
13. fmax = 0.4 2 10 = 8 N
(b) At t = 3 s, F = 6 N
Common acceleration

6
a = = 1 m/s 2
6
Pseudo force on 2 kg
=21=2 N

105

(backward)

14. N = Mg F sin
F cos = N = ( Mg F sin )
Mg

F=
cos + sin
For F to be minimum,
dF
=0
d

Match the Columns


1. Acceleration after t = 4 s

At t = 4 s, F = 8 N

Fmax = 8
i.e.,
s mg = 8
8
8

s =
=
= 0.4
mg 2 10
(a) (r)
At t = 4 s,

a = 1 ms 2

t = 4 s,
F =8N
F k N = ma
F ma F ma
i.e., k =
=
N
mg
8 (2 1)
=
= 0.3
2 10
(b) = (q)
At t = 01
. s, F = 0 . 2 N
Force of friction (at t = 01
. s) = 0.2 N
(c) (p)

At t = 8 s,
F = 16 N
F kmg

a=
m
16 (0.3 2 10)
=
=5
2
a
i.e.,
= 0.5
10
(d) (s).

2. At = 0, dragging force = 0

Force of force = 0

(a) (s)
At = 90
Normal force on block by plane will be
zero.
Force of friction = 0
(b) (s)
At = 30
Angle of repose = tan 1
= tan 1(1) = 45
As < angle of repose, the block will not
slip and thus,
force of friction = mg sin
= 2 10 sin 30 = 10 N
(c) (p)
At = 60
As > angle of repose
Block will accelerate and thus force of
friction = N
= 1 2 10 cos 60
= 10 N
(d) (p).
3. All contact forces (e.g., force of friction and

normal reaction) are electromagnetic in


nature.
(a) (q), (r)
(b) (q), (r).

106 | Mechanics-1
Nuclear force is the force between
nucleons (neutrons and protons). Between
two protons field force also acts.

5. (a) As block A is stationary, f = 10 N


T

4. (a) N R 10 = 2 ( + 5)

T
T=F

C
A

5 m/s2

F = 10N

+ ive

(a) (p)
10 N

(b) As block C is stationary force of friction


between C and ground will be zero.
(b) (s)
(c) Normal force ( N C ) on C from ground
N C = N B + mC g
= mB g + mC g
= ( mB + mc ) g
= (1 + 1) 10 = 20 N

NR

N R = 20 N

(a) (f)

(b) mg ( = 20 N) > F ( = 15 N)

(c) (q).
(d) As block A is stationary
T = F (as shown in figure)
= 10 N

f
NR
20 N

(a) (p).
6. If friction force ( f ) is less than the applied

F (= 15 N)

force ( F ).

Block would be slipping in the downward


direction.
Force of friction will be in the upward
direction.
Frictional force = 20 15
=5N
(b) (p)
(c) If F = 0, the block will slip downwards
due to mg ( = 20N)
Limiting friction = s N R
= 0.4 20
=8N
Minimum value of F for stopping the block
moving down = 20 8
= 12 N
(c) (s).
(d) F = mg + 8
= (2 10) + 8
= 28 N
(d) (s)

mg
F

fmax = 8 N

Net force on body = F f


The body will be in motion and thus the
friction will be kinetic.
(a) (q)
(b) If friction force ( f ) is equal to the force
applied, the body will be at rest. If the
body is at the point of slipping the
force of force will be limiting too.
Emphasis is being given to the word
may be as when a body is moving and
the external force is made just equal to
the frictional force, the body would still
be moving with friction force at its
limiting (kinetic) value.
(b) (p), (r)
(c) If object is moving the friction would
be kinetic as explained in (a).
(c) (q).

(d) If the object is at rest, then friction


may be static and limiting as
explained in (b).
(d) (p), (r)

Laws of Motion |
7. (a) Normal force between A and B = m A g

f1 (max.) = 1m A g (towards left)


f1

f1( R)
B

3 kg

f2
C

(d) Friction force on 3 kg block due to 5 kg


block = f2 (max)
= 5 N, towards left.
(d) (q), (s).
8. (a) and (b)

F
2

5 kg

f2(R)

= 0.2 2 10
=4N
Normal force between B and C
= ( m A + mB ) g
f2 (max) = 2 ( m A + mB ) g
= 0.1 (2 + 3) 10
=5N
Total friction force on 3 kg block
= f1 (max) + f2 (max) = 4 + 5
= 9 N towards left
(a) (q), (s)
(b) Friction force on 5 kg block
= f2 ( R) = f2 (max)
= 5 N, towards right
(b) (p), (s)
(c) Friction force on 2 kg block due to 3 kg
block = f1( R)
= f1 (max)
= 4 N, towards right
(c) (p), (s)

107

2 kg
s
mg

in

30

T
=

3
2

30

T
T T
B
3kg

3
2

3
= 40.89
2
Now as, angle of incline (30 ) < angle
repose (= 40.89 )
The block A and so also B will remain
stationary.
(a) (r),
(b) (r)
(c) Tension ( T) in the string connecting
2 kg mass = mg sin 30
1
= 2 10
2
= 10 N
(c) ( s)
(d) Friction force on 2 kg mass = zero.
(d) (r).
Angle of repose = tan 1

Work, Energy and Power

Introductory Exercise 6.1

1. Work done by F = F x

2. Work done by F = F l

|F|= F ,|N|= N, |W |= W


|F| = F, |N| = N

|W| = W, |T| = T

|l| = l

|T|= T and |x|= x

=Fl

=Fx

= |N||x| cos

=0

Work done by T = T x

= |T||x|cos 0
=Tx

= |W || l | cos +
2

Work done by W = W x

=0
2

Work done by |W |= W l

= |W ||
x| cos

= |N||
l | cos

=0

Work done by N = N l

Work done by |N|= N x

= |F||
l | cos

= |F ||x| cos

= W l sin

Work done by |T|= T l

= |T|| l |cos
= T l cos

Work, Energy and Power |

Substituting value of N from Eq. (ii) in


Eq. (i).
(W F sin 45 ) = F cos 45
1
1
1
or
W F
=F

4
2
2
F
F
or
W
=4
2
2
5F
or
=W
2
2
2
or
F=
W =
mg
5
5

g
4

3. W T = m

g
a=4

W = mg

Work done by force F = F s

2
1
mgs
=
mg s
=
2
5
5

1.8 10 2
=
5
= 7.2 J

|g|= g and | l |= l

|T|= m g

mg
4

Work done by friction = N s


3
mg
4

= |N||s|cos
= Ns
= F cos 45 s
= 7.2 J

Work done by string = T l

= |T|| l |cos
3
= m g l cos
4

3
= mgl
4
F sin 45
N

Work done by gravity = g s

= |g||
s | cos
2
=0
(i)

N = F cos 45

4.

= |F||s |cos 45 = F s

5.

F = mg sin 45

2ms1

45

F cos 45

mg sin 45

mg cos 45
45

N = |N|, W = |W |

= 1 10

s = |s |, F = |F|, g = |g|
N + F sin 45 = W

109

(ii)

1
2

=5 2N
Displacement of lift in 1s = 2 m

1
2

110 | Mechanics-1

Work done by force of friction ( F ) = F s

7. Work done = Area under the curve


F(N)

= |F||s |cos 45
= F s cos 45 = 5 2 2

6. Total work-done by spring on both masses


m

10

1
= 10 Nm
2
4

A2

A3

A4
2

A1

x (m)

m
10

x0
m

= A1 + A2 + A3 + A4

= PE of the spring when stretched by 2 x0


1
= k (2x0 )2
2
= 2 k x20

2 10 2 10
=
+
+ [10 2]
2
2

2 10
+
2

Work done by spring on each mass


2 k x20
=
= k x20
2

= 30 Nm

Introductory Exercise 6.2


1. a =

20 ms
2s

= 10 ms 2

fP

F = ma = 2 kg 10 ms = 20 N
s = Area under curve
1
= 25 20 ms 1
2
= 20 m
Work done = F s = ( 20 N) (20 m )
= 400 Nm

According to B (non-inertial frame)


work done
= Work done by F + Work done by f p
(pseudo force)
= ( mas) + ( mas) = 0
As P is at rest , K = 0
K = W (Work -Energy theorem)

2. According to A (inertial frame)


P

A
Q

m
a

Acceleration of P = a
Force on P = ma

Work done over s displacement = mas


Now,
v2 = u2 + 2as
(Qu = 0)
= 2as
1
1
2
Gain in KE = mv = m 2 as = mas
2
2
K = W (Work -Energy theorem)

Note In inertial frames one has to also to consider


work-done due to pseudo forces, while applying
Work-energy theorem.
3.

v= x
dv
1 dx
a=
=
dt
2 x dt
1
2
=
x=
2 x
2
2

F = ma = m
2
m2
W =
b
2

Work, Energy and Power |

111

4. K = Work done by F

or

+ Work done by gravity


F = 80 N

1
x
mv20 = A
2
2
m
x = v0
A

7. (a) If T = mg, the block will not get

accelerated to gain KE. The value of T


must be greater that Mg.

4m

5g

= 80 4 cos 0 + 5 g 4 cos
= 320 + ( 200)
K f K i = 120 J
(as K i = 0)
K f = 120 J

or
or

5. Change in KE = Work done


R(1 cos)

ma = mg
mg
R sin
mg

R
mg
O

Hand
Mg

Ans. False
(b) As some negative work will be done by
Mg, the work done by T will be more
that 40 J.
Ans. False

1
mv2 = mgR (1 cos ) + mgR sin
2
v = 2 gR (1 cos + sin )
6. K = W

or

1
mv20 =
2

0 Ax dx

(c) Pulling force F will always be equal to


T, as T is there only because of pulling.
Ans. True
(d) Work done by gravity will be negative
Ans. False

Introductory Exercise 6.3


Spring is having its natural length.

1. In Fig. 1

T
A

mAg

B m
Ground

T
T
B m
Ground
mg

Fig. 1

Fig. 2

In Fig. 2 A is released. A goes down byx .


Spring get extended by x. Decrease in PE
of A is stored in spring as its PE.
1

mAg x = k x2
2
Now, for the block B to just leave contact
with ground
kx = mg
i.e.,
2m A g = mg
m

mA =
2

112 | Mechanics-1
1
1
mv2C = 500 00924
.
2
2
500
or
vc =
0.0924 = 2.15 m s 1
100
m
4. Work done by man =
gh + Mgh
2
m
= + M gh
2

l
2
1
l
mv2 = mg
2
2
v= gl

or

2. Decrease in PE = mg

i.e.,

3. OA = 50 cm
C 20 cm B 40 cm
A

5. When block of man M goes down by x, the

30 cm

spring gets extended by x. Decrease in PE


of man M is stored in spring as its PE.

Extension in spring (when collar is at A)


= 50 cm 10 cm = 0.4 m
Extension in spring (collar is at B)
= 30 cm 10 cm
= 20 cm = 0.2 m
KE of collar at B
= PE of spring PE of spring
(collar at A)

37
T

KE of collar at C
= PE of spring PE of spring
(Collar at A)

(Collar at C)

1
= 500 [(0.4)2 (026
. )2 ]
2

mg sin 37

Extension in spring (collar arrives at C)


= [ (30)2 + (20)2 10] cm = 0.26 m

T
T

(collar at B)

1
K [(0.4)2 (0.2)2 ]
2
1
1
or
mv2B = 500 12
m
2
500 012
.
or
vB =
= 2.45 s 1
10

mg cos 37
37
mg

1 2
kx
2
or
kx = 2Mg
For the block of man m to just slide
k x = mg sin 37 + mg cos 37
3 3
4
or
2Mg = mg + mg
5 4
5
3
or
M= m
5
Mgx =

Introductory Exercise 6.4


1. Velocity at time t = 2 s
1

v = g t = 10 2 = 20 ms
Power = Force velocity
= mgv = 1 10 20 = 200 W

2. Velocity at time = a t =

Ft
(acceleration being constant)
2m
F2 t
= F vav =
2m

vav =
Pav

F
t
m

Work, Energy and Power |


Instantaneous power
= Force instantaneous velocity
Ft F 2 t
=F
=
m
m
Energy
3. Power =
Time
KE
KE = P t
P=
t
1

mv2 = Pt
2
2 Pt
or
v=
m
ds
2 P 1/ 2
or
=
t
dt
m
2P
1/ 2
or
ds = m t dt
2 P t 3/ 2
or
s=
+c
m 3/2
At t = 0, s = 0, c = 0
8 P 3/ 2
Thus,
s=
t
9m
4. P = 2 t

KE = P dt = 2t dt
= t2 + c

113

As at t = 0, KE = 0, c = 0
KE = t2
1
2
i.e., mv2 = t2 or v =
t
m
2
2.0 + 2 t
Pav =
=t
2
5. U = 20 + ( x 2)2

PE( = U) is minimum at x = 2.
Equilibrium position is at x = 2 m
dU
= 2 ( x 2)
dx
d2U
=2
dx2
Equilibrium is stable.
6. F = x 4

For equilibrium, F = 0
i.e.,
x 4=0
i.e.,
x=4m
dU
As, F =
dx
dU
= ( x 4)
dx
d2U

= 1
dx2
Thus, equilibrium is unstable.

AIEEE Corner
Subjective Questions (Level I)
(a) Work done by a constant force

3. m1 g T = m1a

(i)

1. (a) Work done by a constant force

T m2 g = m2 a

(ii)

Work done by applied force = F s cos 0


= 40 2 = 80 Nm
Work done by force of gravity = mgs cos
= 2 10 2 1 = 40 Nm

2. r21 = r2 r1
^

= (2 ^i + 3 ^j 4 k) (1 ^i + 4 ^j + 6 k)
^

= i j 10 k

Work done = F r21


^

= (6 ^i 2 ^j + k) ( ^i ^j 10 k)
= 6 + 2 10 = 2 Nm

Solving Eq. (i) and Eq. (ii),


m m2
a= 1
g
m1 + m2
41
=
g
4+1
Q m1 = 4 kg

and m2 = 1 kg
T
a
3
= g
T
5
1
s = ut + at2
2

T
a
T
m2g
m1g

114 | Mechanics-1
=

1 3 2
g 2
2 5

= 2 10 2 sin 60
= 20 3 Nm
= 34.6 Nm
Work done by force of friction
= f s cos
= Ns cos
= ( mg cos ) s
1
= (2 10 cos 60 ) 2
2
= 10 Nm

= 12 m
Work done by gravity on 4 kg block
= 4 g 12 cos 0
= 480 Nm
Solving Eq. (i) and Eq. (ii),
2 m1m2
T=
g
m1 + m2
4
=2 g
5
= 16 N
Work done by string on 1 kg block
= 16 12 cos 0
= 192 Nm

(b) Work done by a variable force.


W =

F dx
x = 4
=
2x dx
x=2

6.

x= 4

x2
= 2
2 x = 2

4. Work done by applied force = F s cos 45


N

F = 16 N

= [( 4)2 (2)2 ]
= 12 Nm

45

W =

F dx
x=2 4
=
dx
x = 4 x2

7.
s = 2.2 m

mg

x=2

x =4

4x 2 dx
x=2

x 2 + 1
= 4

2 + 1 x = 4

1
= 16 2 2
2
= 24.9 Nm
Work done by normal force
= N s cos 90 = 0
Work done by force of gravity
= mgs cos 90 = 0
Total work done on the block
= 24.9 + 0 + 0
= 24.9 Nm

x=2

1
= 4
x x = 4
1 1
= 4 = 1 Nm
2 4
(c) Work done by area under F- x graph
8. (a) W = 3 (5 10) = 15 Nm
Fx(N)

5. Work done by gravity = mgs sin


N

f=

3
N

x(m)

s
mg
= 60

10

12

(b) W = 3 (1 0 5) = + 15 Nm

14

16

Work, Energy and Power |


3 (12 10)
(c) W =
= 3 Nm
2
(10 4) + (12 0)
(d) W =
3
2
= + 27 Nm
(3 2) + (3 0)
9. (a) W =
2
2

minimum PE = ( 4 4) 16
= 16J
Now,
(KE) max + (PE) min =
mechanical energy
or
(KE) max + ( 16) = 4
or
(KE) max = 12 J
(c) PE max = KE max = 12 J

Fx(N)

PEmax
2

x(m)
O

115

= 4 Nm
(b) W = 0 Nm
1
(c) W = (6 4) ( 1 0) = 1 Nm
2
(d) W = 4 Nm + 0 Nm + ( 1) Nm
= 3 Nm.
Conservative force field and Potential
Energy.
d
10.
F=
U
dr
d
=
Ar1
dr
A
= ( ) ( A ) r 1 1 = 2
r
11. U = ( x 4)2 16
PE (at x = 6.0 m)
= (6 4)2 16 = 12 J
KE (at x = 6.0 m) = 8 J
(a) Total mechanical energy
= ( 12) + (8) = 4 J
(b) KE will be maximum where, PE, is
minimum.
For U to be minimum,
dU
=0
dx
d
i.e.,
[( x 4)2 16] = 0
dx
or
2 ( x 4) = 0
or
x=4m

PEmax

Total

x=4
KEmax

U = ( x 4)2 16
or
12 = ( x 4)2 16
or
x2 8 x 12 = 0

x= 4+2 3
and
= 42 3
(d) U = ( x 4)2 16
dU
= 2 ( x 4)
dx
dU

Fx =
= 2 ( x 4)
dx
or
Fx = 8 2x
(e) Fx = 0
i.e.,
8 2x = 0
or
x=4m
Kinetic energy and Work-energy theorem
p2
12. K =
2m
2
p + p

K =
2m
(K is the KE when momentum p is
increased by 50%)
9 p2
or
K =
4 2m
9
or
K = K
4
9
5
or
K K = K K = K
4
4
K K 5

=
K
4
5
= 100% = 125 %
4

116 | Mechanics-1
13. p = (2 mK )1/ 2

p = [2m ( K + 1% of K ) ]1/ 2
( p is the momentum when KE i.e., K is
increased by 1%)
1

= [2 mK (1 + 1%) ]2
1
= (2 mK )1/ 2 1 +

100

1/ 2

1/ 2

1
p = p 1 +

100
1
1
= p 1 +

2 100
1
= p 1 + %

2
1
p = p + % of p
2
Increase in momentum = 0.5 %.
i.e.,

102
= 2.5 ms 2
2 20

Now, mg F = ma

F = m ( g a)
= 5 (10 2.5)
= 37.5 N
Work done by push of air
= F s cos
= (37.5 20)
= 750 Nm

F (Push of air)
a

mg

17. (a) W =

F dx
2
= (2.5 x2 ) dx
0
2

x3
= 2.5x
2 0

23
= (2.5 2)
3
= 2.33 Nm
(1 Nm = 1 J)
K = 2.33 J
i.e., KE (at x = 2) KE (at x = 0) = 2.33 J
KE (at x = 2) = 2.33 J
(b) Position of maximum KE
F = 2.5 x2
F decreases as x increases and F is zero
when x = 2.5 m
Thus, work will be +ive from x = 0 to
x = 2.5 m an so KE will be maximum at
x = 2.5 m.
KE (at x = 2.5 m)

14. s = (2t2 2t + 10) m

ds
= 4t 2
dt
d2 s
= 4 ms 2
dt2
d2 s
F=m 2
dt
= 2 4= 8 N
s (at t = 0 s) = 10 m
s (at t = 2 s) = 2.22 2.2 + 10 = 14 m
s = 14 m 10 m = 4 m
Work = F s
=8N 4m
= 32 Nm
v2 (0.4)2 0.16
15. a =
=
=
= 0.04 m s 2
2s 2 2
4
T
mg T
=a
m
i.e.,
T = m( g a)
= 30 (10 0.04)
= 298.8 N
Work done by chain = T s cos
= (298.8 2) Nm
= 597.6 Nm

16. a =

2.5

(2.5 x2 ) dx

x3
= 2.5x
3 0

2.5

2.5 2.5
= 2.5 2.5

2
= 2.5 2.5
3
= 2.635 J

Work, Energy and Power |


18. a =

F mg
m
g
F
=
g
10 m
11
F=
mg
10

or
or

T
F
F

mg

(a) Work done on astronaut by F


11mg
=
15
10
11
=
72 10 15
10
= 11642.4 Nm
(b) Work
done
on
astronaut
by
gravitational force
= mghcos
= 72 9.8 15 = 10584 Nm
(c) Net work done on astronaut
= (11642.4) + ( 10584) = 1058.4 Nm
KE = 1058.4 J
1
(d) mv2 = 1058.4
2
1058.4 2

v=
= 5.42 ms 1
72
19.

and

(i)
(ii)

T = m2 a
m1 g T = m1a
a
m2

Adding Eq. (i) and Eq. (ii),


m1 g = ( m1 + m2 ) a
m1
a=
g
m1 + m2
1
=
g (Q m1 = 1 kg and m2 = 4 kg)
1+ 4
g
= = 2 ms 2
5
Now,
v2 = u2 + 2as
i.e.,
(Qu = 0 ms 1)
v2 = 2as
or
v = 2as
= 221
= 2 ms 1
50
20. T 50 =
a
10
T

a
50 (N)

T'
A
300 (N)

or
and
i.e.,

T 50 = 5a
T = 2T
T
T=
2

(iii)

300 T = 30a

(iv)

a = 5 ms 2
1m

(ii)

T 100 = 10a
300
300 T =
a
10

Solving Eq. (iii) and Eq. (iv),

m1g

(i)

From Eq. (i) and Eq. (ii),


T
50 = 5a
2

i.e.,
a

T'

Also,

T
T

or
T

117

T = 100 + 10a = 150 N

118 | Mechanics-1
1
mv2s
2
1
( mg cos ) R ( cos ) + mgR = mv2s
2
1
2
2
or
mvs = mgR (1 cos )
2

21. NR cos (180 ) + mgR =

1 10 1 = k 4 10 2 +
or

1
1 (0.3)2
2

10 = 80 k + 0.045
10 0.045
k =
= 0.124
80

or

24. f = force of friction while disc in slipping

over inclined surface = mg cos 30

s(
say
)

s/2

mg

30

2R

Stops

f'
0.50 m

22. For 45 kg mass to drop 12 mm, the

increase in length of the spring will be


24 mm.
Now, decrease in PE of 45 kg mass
= Increase in KE of 45 kg mass + Increase
in PE of spring
1
1
45 9.8 12 103 = 45 v2 + 1050
2
2
[(75 + 24)2 752 ] 106
i.e.,
5.292 = 22.5 v2 + 2.192
or
22.5 v2 = 3.0996
or
v2 = 0.13776
or
v = 0.371 ms 1
(b) With friction (when mechanical
energy does not remain conserved)
23. Decrease in PE of 1 kg mass

= Work done against friction due to 4 kg


mass + Increase in KE of 1 kg mass

25. m A g sin m A g cos 2 T = m A a


T
T

T
T
T

2T

2T

or
= 2 gR sin
or
vs = 2 gR sin
(vs is the speed with which sphere hits
ground)
1
1
mv2w = mgR mv2s
2
2
1
= mgR m 2 gR sin2
2
= mgR (1 sin2 ) = mgR cos2

vw = 2 gR cos
(vw is the speed of wedge when the sphere
hits ground)

f = force of friction while disc is slipping


over plane surface = mg
Now, decreases in PE of disc = Work done
against frictional force
s
mg = fs + f (0.5)
2
s
or mg = ( mg cos 30 ) s + mg (0.5)
2
or mgs (0.5 cos 30 ) = mg (0.5)
0.5

s=
0.5 cos 30
0.15 0.5
=
= 0.2027 m
3
0.5 0.15
2
Work performed by frictional forces over
the whole distance
s
50
0.2027
= mg =
10
2
1000
2
= 0.051 J

co
s

mAg cos

Ag

v2s

B
mBg

or 300

3
4
0.2 300 2 T = 30a
5
5

Work, Energy and Power |


or
180 48 2T = 30a
or
(i)
132 2T = 30a
Also,
T mB g = mB a
or
T 50 = 5a
or
(ii)
2T 100 = 10a
Adding Eq. (i) and Eq. (ii),
32 = 40a
or
a = 0.8 ms 2
Speed (v) of block A after it moves 1 m
down the plane
v2 = 2as
2
or
v = 2 0.8 1
or
v = 1.12 ms 1

Three types of Equilibrium


dU
28. (a) F =
dr

26. Work done by frictional force acting on

block
= mgs
= 0.25 3.5 9.8 7.8
= 66.88 J
Increase in thermal energy of block-floor
system
= 66.88 J
As the block stopped after traversing
7.8 m on rough floor the maximum kinetic
energy of the block would be 66.88 J (just
before entering the rough surface).
Maximum PE of spring
= Maximum KE of block
1 2
kxmax = 66.88
2
2 66 . 88
m

xmax =
640
Maximum compression in the spring
= 0.457 m
27. Decrease in PE of mass m2 = Work done

against friction by mass m1 + Increase KE


of mass m1 + Increase in KE of mass m2
10 kg

m1

m2 = 5 kg

119

1
1
m2 g 4 = m1 g 4 + m1v2 + m2v22
2
2
1
5 10 4 = 0.2 10 10 4 + (10 + 5) v2
2
Solving,
v = 4 ms 1

Point

dU
dr

+ ive

ive

+ ive

ive

ive

+ ive

ive

+ ive

zero

zero

(b) x = 2 m point is of unstable equilibrium


(U being + ive)
x = 6 m point is of stable equilibrium
(U being lowest ive)
x3
29. U =
4x + 6
3
For U to be maximum (for unstable
equilibrium) and minimum (for stable
equilibrium)
dU
=0
dx

d x3
i.e.,
4 x + 6 = 0

dx 3

x2 4 = 0
x= 2
d2U
d 4
=
( x 4) = 2x
dx
dx2
At x = + 2 m,
d2U
= 2 ( + 2) = + 4
dx2
U is minium.
At x = 2 m,
d2U
= 2 ( 2) = 4
dx2
U is maximum.
x = + 2 m point of stable equilibrium.
x = 2 m point of unstable equilibrium.
or
or

120 | Mechanics-1
30. F =

dU
dx

Thus, the equilibrium of the charge q


is unstable if it is slightly displaced along
x-axis.
(b) If charge q is displaced slightly along
Y axis, the net force on it will be along
origin O and the particle will return to
its original position. And as such the
equilibrium of the q is stable.

i.e.,

U = F dx
= (Area under F - x graph)
The corresponding U Vs x graph will be as
shown in figure

F'

F'

+q
E

+q
Fnet

B
( a, 0, 0)

32. (a) Velocity at t = 0 s is 0 ms 1


U

Velocity at t = 2 s is 8 ms 1 (using
v = 0 + at)
vav = 4 m s 1 (as acceleration is constant)
Pav = F vav
= ma vav
= 1 4 4 = 16 W
(b) Velocity at t = 4 s is 16 ms 1 (using
v = u + at)
Instantaneous power of the net force at
t = 4 s will be
P = mav
= 1 4 16
= 64 W

Thus, point C corresponds to stable


equilibrium and points A and E
correspond to unstable equilibrium.
charge placed at origin is in
equilibrium as is two equal and opposite
forces act on it.

31. q

y
q

+q
B
( a, 0, 0)

+q
F

A
(+ a, 0, 0)

(a) But, if we displace it slightly say


towards +ive x side, the force on it due
to charge to B will decreases while that
due to A will increase.
q
O F2

33. Power = Fmin vmax

Fmin = (r)

Fmax
r

Initially

+q

Finally

P = r vmax

F1 > F2

Due to net force on q towards right the


change q will never come back to original
O, its origin position.

vmax

vmax =

P
r

Objective Questions (Level 1)


Single Correct Option

1
mv2
2
m is always +ive and v2 is +ive.
(even if v is ive).
KE is always + ive.

1. In KE =

Below reference level (PE = 0) PE is ive.


Mechanical energy which is sum of KE
and PE may be ive.
Correct option is (a).

Work, Energy and Power |


2. Yes, this is work-energy theorem.

Correct option is (a).


3. On a body placed on a rough surface if an

external force is applied the static body


does not move then the work done by
frictional force will be zero.

4. W = F r21

= F ( r2 r1)
^

= ( ^i + 2 ^j + 3 k) [( ^i ^j + 2 k) ( ^i + ^j + k)]
^

= ( ^i + 2 ^j + 3 k) ( 2 ^j + k)
= 4+3
= 1J
Correct option is (b).
5. W =

F dx
=

t =2

t = 0 4 t

dt

t4
= 4
4 0

0 7 2x + 3 x dx

= [7 x x2 + x 3 ] 50 = 135 J
Correct option is (d).

= 16 J
Correct option is (c).
10. Range = 4 height

6. P = F v

= (10 i + 10 j + 20 k) (5 i 3 j + 6 k)

or

u2 sin 2
u2 sin2
= 4
g
2g

= 50 30 + 120

or

sin 2 = 2 sin2

= 140 W

or

2 sin cos 2 sin2 = 0

or

sin (cos sin ) = 0

Correct option is (c).


7. Work done in displacing the body

= Area under the curve


1 10
= (1 10) + (1 5) + (1 5) +

2
= 15 J
Correct option is (b).
1
mgh + mv2
W
2
8. P =
=
t
t
800 kg 10 ms 2 10 m +
1
800 kg (20 ms 1)2
2
=
1 min
=

121

80000 + 160000
=
60
= 4000 W
Option (c) is correct.
t3
9. x =
3
dx

v=
= t2
dt
dv
and
a=
= 2t
dt
F = ma = 2 2 t = 4 t
W = F v = 4 t t2 = 4 t 3
Work done by force in first two seconds

(800 10 10) + [ 400 (20)2 ]


60

or

cos sin = 0 (as sin 0)

or

tan = 1

i.e.,

= 45

Now, K = KE at highest point


1
= m (u cos )2
2
1
= mu2 cos2
2
1
= mu2 cos2 45
2
1
1
= mu2
2
2
1
2

mu = 2 K
2
i.e., Initial KE = 2K
Correct option is (b).

122 | Mechanics-1
11. P = 3 t2 2t + 1

15. x = 2.0 m to 3.5 m

dW
= 3 t2 2t + 1
dt
dW = (3 t2 2 t + 1) dt

i.e.,

W =

2 (3 t

2t + 1) dt
4

t 3 2t2

= 3
+ t
2
3
2
= [ t 3 t2 + t ] 42
= ( 4 3 42 + 4) (23 22 + 2)
= 52 6
= 46
K = 46 J
Correct option is (b).
1
12. K i = 10 102 = 500 J
2
Work done by retarding force,
or

W =

30

20 0.1 x dx
30

x2
= 0.1
2 20

= 0.05 [(30)2 (20)2 ]


= 25 J
Final kinetic energy = K i + W
= 500 + ( 25)
= 475 J
Correct option is (a).
13. KE of 12 kg mass : KE of 6 kg mass

1
1
m12v2 : m6v2
2
2
[Acceleration being same (equal to g) both
will have same velocities]
= m12 : m6
= 12 : 6
= 2 :1
1
2
14. W = k [ x2 x12 ]
2
10 2 5 2
1
= 5 103

100
2
100

5 103
=
(100 25) = 18.75 Nm
2 104
=

Correct option is (c).

dU
6
=
=4
dx
1.5

F =+4N
x = 3.5 m to 4.5 m
dU 2
= =2
dx 1

F = 2N
x = 4.5 m to 5.0 m
dU
=0
dx

F =0N
Work done = ( 4 1.5) + ( 2 1) + 0
=4J
1
2

mv = 4
2
1
or
1 v2 = 4
2

v = 2 2 ms 1
16. KE at highest point

1
m (u cos 45 )2
2
2
1
1
= mu2
2
2
1 1
= mu2

2 2
E
=
2
Correct option is (a).
=

17. Work done by person

= [Work done by gravitational pull on rope +


gravitational pull on bucket]
h

= mg + ( Mgh)

m
= M + gh

2
Correct option is (a).
1
18.
mv2 = Fx
2
1
mv 2 = Fx
2
x v 2 (2 v)2
= 2 = 2 = 4 x = 4 x
x
v
v
Correct option is (b).

(i)
(ii)

Work, Energy and Power |


19. Vertical velocity (initial) = v0 sin

At an altitude h
(vertical velocity)2 = (v0 sin )2 + 2 ( g) h
(horizontal velocity)2 = (v0 cos )2
(Net velocity)2 = v20 2 gh
Net velocity = v20 2 gh

t02

21.

v = 0 + at
F
= at =
t
m
Instantaneous power,
P = Fv
F
=F
t
m
F3
or
P=
t
m
or
P = constant t.
Correct option is (b).

22. While ball comes down

t0

Fig. 1

20. Maximum power will be at 2 s

velocity (at t = 2 second) = a 2


F
=
2
m
2F 2F 2
Power (at t = 2 s) = F
=
m
m
Correct option is (d).

123

While ball goes up after elastic


collision with the surface it strikes
a = + g
H
u = gt
0

Using,

i.e.,

v = u + at
v = ( gt0 ) + ( + g) ( t t0 )
v = g ( t 2t0 )
1
KE = mg2 ( t 2t0 )2
2
K = ( t 2t0 )2
K
t02

u=0
+

a=+g

t0

2t0

Fig. 2
v = gt0

at t = t 0 when ball strikes the surface


Using
i.e.,

where,

v = u + at
v = 0 + gt
v = gt.
1
1
KE = mv2 = mg2 t2
2
2
K = t2
1
= mg2 = constant.
2

From the expression for KE


at
t = 2t0 , K = 0
as shown in Fig. 2.
Correct option is (b).
23. (block) = 3000 kgm 3 and (water)

= 1000 kgm 3

vg

vg

( )g

124 | Mechanics-1
1
=
3

i.e.,

3 y + x = 5
y
x
+
=1
5/3 5/

(External force applied to move the block


upward with constant velocity).

From above figure


15 3
=
20 4
Work done will be zero, if
=
i.e., tan = tan
5/ 3
or
=
5/3 4
3 3
or
=
4

=4
Correct option is (d).

Work done = F s

tan =

= v ( ) gs
= v ( ) 10 3
5
= ( ) 10 3

(Qv = 5)

= 5 1 10 3

= 5 1

1
10 3 = 100 J
3

Correct option is (a).


Net work done
24. pav =
Time of flight ( T)
mgH cos + mgH cos 0
=
T
H = Maximum height attained by the
projectile
(0)
= zero
=
T
1
25.
W ( a) = 2 k ( x)2
2

27. Let x be the elongation in the spring.

Increase in PE of spring = Decrease in PE


of block
1 2

k x = mgx sin
2
2mg sin

x=
k
Correct option is (a).
s = 2 t2 ^i 5 ^j

ds = 4 t ^i dt

2k'

k'

l/3
(a)

2l/3
(b)

Force Constant

28.

W = F ds

1
k ( x)2
2
W ( a) > W ( b)

W ( b) =

Given
Straight line along which
work is zero.

F = 20i + 15j

5/3

29.

0 12 t

dt

= [ 4 t 3 ] 20
= 32 J
Correct option is (b).

or

0 (3 t i + 5 j) 4 t i dt

t3
= 12
3 0

Correct option is (c).


26.

W = mgh + mgd
= mg ( h + d)
Fav d = mg ( h + d)
h

Fav = mg 1 +

Work, Energy and Power |


1
30. Energy stored in A = E = kA x2A
2
kA

125

34. Decrease in PE of mass m

kB
d

2kA = 2kB
F
F

F
A

F
1
kA
2
kA

k (spring) = 100 N/m

B
2

i.e.,

F
2kA

or

Energy stored in B =

F2
2kB

F2
4 kA

=
(Q kB = 2 kA )

36. Work done by normal force is zero, being

(i)
(ii)

perpendicular to the displacement.


Work done by string is + mgh while that
due to gravity it is mgh.
Net work done = ( + mgh) + ( mgh)
=0
Correct option is (c).
37. Work done on floor = 0 (as displacement is

n=8
Correct option is (d).
1
1
33. mgh = mv2 + kx2
2
2
k 2
2
i.e.,
v = 2 gh
h
m

or
v = 0.866 m s 1
Correct option is (b).

Correct option is (c).


then the KE gained by the block will
change into the PE of the spring.
Due to inertia the spring will not start
compressing the moment the block just
touches the spring and as such the block
will still be in the process as increasing its
KE. Thus v of the block will be maximum
when it compresses the spring by some
amount.
Correct option is (b).

Correct option is (a).


1 2 1
31.
kx = mv2
2
2
m
x=v
k
0.5
= 1.5
= 0.15 m
50

= 2 10 0.15

100 22
= 4 m.
2 10 10 sin 30

35. PE of block will change into its KE and

E
1 F2
=
=
2 2kA 2

Correct option is (a).


1
32.
mv2 = F (2x)
2
1
m (2 v)2 = F ( nx)
2
Dividing Eq. (i) by Eq. (ii),
(2 v)2 n
=
2
v2

= Increase in PE of spring
1
mgd sin = kx2
2
kx2
d=
2mg sin

zero).
38. Acceleration =

10
(0.15)2
0.1

(0 20) ms 1
(10 0) s

= 2 ms 2
Therefore, net force on particle
= 2 kg 2 m s 2
=4N
i.e., the net force on the particle is opposite
to the direction of motion.
Correct option is (a).

126 | Mechanics-1
Net work done = F s
= F (Area under v-t graph)
1
= 4 20 10 = 400 J
2

Thus net work done may not be wholly


due to frictional force only.
Further net force 4 N may not be wholly
due to friction only.
39. Height of bounce = (100 20) % of 10 m

=8

Correct option is (c).

1
mv2 = mgh
2
KE just after bounce = 80% of mgh
When the ball attains maxmum height
after bounce
Gain in PE = mgh = Loss of KE
or
mgh = 80% of mgh
or
h = 80% of h = 80 % of 10 m
=8m
KE just before bounce =

JEE Corner
Assertion and Reason
4. In circular motion only the work done by

1. P = Fv

For power to be constant, the velocity


must also be constant. Thus, assertion is
false.
According to 2nd low of motion, net
constant force will always produce a
constant acceleration. Reason is true.
Correct option is (d).
2. As displacement is opposite to force

(reason) the work done by force will be


negative.
Thus, Assertion is true. Further, as reason
is the correct explanation of the Assertion.
Correct option is (a).
3. Conservative force has nothing to do with

kinetic energy. (If a non-conservative force


acts on a particles, there would be loss of
KE). Thus, assertion is false.
Work done by conservative force decreases
PE (reason is true).
A

The centripetal force is zero. Assertion is


false, centripetal force acts towards centre
while the velocity acts tangentially.
Reason is true.
Correct option is (d).
5. As the speed is increasing (slope of graph

being increasing) there must be net force


in the +ive direction of displacement.
Thus, work done to all forces will be
positive, Assertion is true and also as
explained above reason besides being true
is the correct explanation for the
Assertion.
Correct option is (a).

6. Work done by constant force F when the

body shifts from A to B.

S'

mg (a conservative force)
h

S'

S
D

mg

W = mgh
If PE at A is zero.
The PE at B would be mgh.
Correct option is (d).

W AB = F s = F s cos

Similarly, W AC = F s

= F s cos (90 + )
= F s sin

WCD = F s = F s cos

Work, Energy and Power |


WDB = F s cos(90 )
= F s sin
W AC + WCD + WDB = F s cos = W AB
Work done by a constant force is path
independent. thus, Assertion is true.
The Reason is false. Kinetic frictional
force remains constant but is a
non-conservative force.
Correct option is (c).

127

greater than 90 and that between N


(normal force on wedge) and its
displacement will be less than 90 as given
in reason.
N
A

N
B

7. It is true that work-energy theorem can be

applied to non-inertial frame also as


explained in the answer to question no. 2
of introductory exercise 6.2.
Earth is non-inertial which is also true is
a separate issue and has nothing to do
with the assertion.
Thus, option (b) would be the answer.
8. When block is depressed the excess of

upthrust force will act as restoring force


and will bring the block up. The velocity
gained by block will take the block above
its equilibrium and the block will oscillate
about its equilibrium position (as given in
reason). Thus, the block will be in
equilibrium in the vertical direction.
Thus, assertion is also true and the reason
being correct explanation of the assertion.
Correct option is (a).
9. As displacement ( = s2 s1) is not equal to

zero, the work done by all forces may not


be zero. Therefore, assertion is false.
Work done by all the forces is equal to
change in KE is as per work-energy
theorem. Thus, reason is true.
Correct option is (d).
10. When the block comes down the wedge,

the wedge will move towards left and the


actually displacement of normal force will
not be along the wedge (see chapter on
CM). It will along AB as shown in figure.
Thus, the angle between N (normal force
on block) and its displacement AB will be

For this the work done byN (or block) will


be negative and that by N (on wedge) will
be positive as given in assertion.
Reason also being the correct explanation
of the assertion.
11. There will be increase in length of the

elastic cord.
elastic
cord

Block
Plank
f2

f1
s

f1
f

Work-done by static force


f1 (on block) = f1s
Work done by static force
f1 (on plank) = + f1s
Work done by static force f1 on the
system (block + plank + cord) = 0
Thus, reason is true.
The work done by the force F will be used
up in doing work against friction f2 and
also increasing the elastic potential
energy of the cord. Thus, assertion is true.
Further, as the reason is the correct
explanation of the assertion.

128 | Mechanics-1
12. Decrease in KE =

1
mv2 0
2

v
Slope

f
Rough

Case-I
gh
s E=m
p
o
P
St

v'
30
f'

1
mv2
2
Change in mechanical energy
=

1
mv2
2
Decrease in ME is used up in doing work
against friction.
In this case the mechanical energy is
being used up in doing work against
friction and in increasing the PE of the
block.
1
Change ME = mv2 mgh
2
Thus, assertion is true.
As explained above the reason is false.
( does not change with the increase in
angle of inclination)
Correct option is (c).
PE + KE =

Objective Questions (Level 2)


Single Correct Option
1. Increase in KE of bead = Work done by

gravity + work done by force F


1
mv2 = mgh + FR
2
(Displacement of force F is R)
1
= 10 5 + 5 5 = 50
2
100

u=
m
= 200 = 14.14 ms 1
Correct option is (a).
2. P = Fv

= mav
dv dv ds
dv
dv

= m v v a =
=

=v
dx

dt ds dt
ds
m 2

ds =
v dv
p
m 2v 2
i.e., ds =
v dv
p v
2v

or

m v 3
s=

p 3 v
=

m
7 mv 3
[(2v) 3 (v) 3 ] =
3p
3p

Correct option is (a).

3. Loss of PE of block = Gain in PE of spring


d
k
A

1
k 22
2
1 1
or 10 10 ( d + 2) = 100 22
2 2
d = 2m
Let, v A = velocity of block when it just
touches spring
mg ( d + 2) sin 30 =

d
2m
m

30

1
mv2A = mgd sin 30
2
1
v2A = 10 2 2
2
v A = 20 ms 1
Correct option is (a).

Work, Energy and Power |


m
4. Mass per unit length of chain =
R/2
m
dm = R d
R/2
yCM =

129

a2 = 0
Correct option is (b).
6. Let x be the expansion in the spring.
kx
A

y dm

dm

kx
B

T T
T

M1g

CM

R 12

R (1 cos )
/2

R (1 cos )

m
R d
R/2

m
2R / 2
=
(1 cos ) d
0
2R
=
1

2
2
= R 1


Now,
or

1
mv2 + mg yCM
2
v2 = 2 g yCOM
2
v = 2 gR 1

Correct option is (c).


5. The moment string is cut
T
T = mg

A
mg
F (= mg)

mg
F

F = kx = mg
F = mg
B
mg (weight)

Net force on A = 2mg

a1 = 2 g
Net force on B = 0

mg

(downward)

Increase in PE of spring = Decrease in PE


of block C
1 2
kx = M1 gx
2
i.e.,
kx = 2M1 g
For block A to remain at rest
kx = min Mg
or
2M1 g = min Mg
2M1

min =
M
Correct option is (c).
7. Ti = mg
2mg
KX i =
= mg
2
When one spring is cut. It means KX i
becomes zero. Downward acceleration,
KX i mg g
a=
=
=
2m
2m 2
Now drawing FBD of lower mass :
mg
mg Tf = m. a =
2
mg

Tf =
2
mg
or
T = Tf Ti =
2
mg sin mg cos
8. a =
m
= g sin g cos
For v to be maximum
x
a
dv

=0
os
mg sin
dx
gc
m

dv
or
v
=0
dx

130 | Mechanics-1
4 mg
x
Correct option is (c).

dx dv

=0
dt dx
dv
or
=0
dt
or
a=0
i.e., g sin = g cos
or
sin = cos
3 3
4
or
=
x
5 10 5
10

x=
= 2.5 m
4
Correct option is (d).
or

9. (a) Between points E and F,

x=

12. Power delivered by man = T v

= T v cos

T
v

|T|= T
m

and |v|= T
13. = 3 x + 4 y
y
3N

dU
is ive.
dr

dU
, the force between E and F
dr
will be + ive i.e., repulsive.
(b) At point C the potential energy is
minimum. Thus, C is point of stable
equilibrium.
Correct option is (c).

P (6m, 8m)

4N
Fnet = 5N

Now, F =

10. Power = F v

mg

mg

= m g ( u + g t)

= mg u cos (90 + ) + m g g t
P = mg u sin + mg2 t

i.e.,

[|u|= u,|g|= g]

Therefore, the graph between P and t will


be as shown in option (c).
11.
x
2x
mg

PE of spring due to its compression by x


= | Work done by frictional force when
displaced by 2x|
1 2
i.e.,
kx = mg 2x
2

6m

= 3N
x

and
Fy =
=4N
y
PR 5
=
PQ 4
5

PR = PQ = 10 m
4
Work done by the conservative force on
the particle
= Fnet PR = 5 N 10 m
= 50 Nm = 50 J
Correct option is (c).

Fx =

14. Both at x = x1 and x = x2 the force acting

on the body is zero i.e., it is in equilibrium.


Now, if the body (when at x = x1) is moved
towards right (i.e., x > x1) the force acting
on it is + ive i.e., the body will not come
back and if the body (when at x = x2 ) is
moved toward rght (i.e., x > x2 ) the force
acting on it is ive i.e., the body will
return back. Then,x = x2 is the position of
stable equilibrium.
Correct option is (b).

Work, Energy and Power |


i.e.,

15. The man will stop when

6b x

= 12a x

131

12

x
mg sin

i.e.,

mg cos

or
or

2a 6
x =
b

Correct option is (a).


19. For the rotational equilibrium of the rod

mg cos = mg sin
( 0 x) mg cos = mg sin
tan
x=
0

kx

l
l/2
mg

16. Taking moment about A

l
= kx l
2
mg
x=
2k

mg
i.e.,

k1x

k2x

PE stored in the spring =

1 2
kx
2

F = (k1 + k2)x

AC F = ( k2 x) l
( k x) l
AC = 2
F
( k2 x) l
=
( k1 + k2 ) x
k2
=
l
k1 + k2

Correct option is (d).


17. mgh + work done by the force of friction

1
= mv2
2
Work done by the force of friction
1
= mv2 mgh
2
1
= 1 22 (1 10 1)
2

= 8J
Correct
option
is (c).

a
b
18. U = 12 6
x
x
U = ax 12 bx 6
dU
=0
dx
b ( 6) x 7 = 0

For stable equilibrium,


i.e.,

a ( 12) x 13

1 mg
( mg)2
k
=
2 2k
8k

Correct option is (c).


m m2
20. a = 1
g
m1 + m2
Speed (v) with which mass m1 strikes the
floor
m m2
= 02 + 2 gh = 2 1
gh
m1 + m2
Correct option is (a).
21. F = ax + bx2

dU
= ax + bx2
dx
or
dU = ( ax bx2 ) dx
or
U = ( ax bx2 ) dx
ax2 bx 3
or
U=

+c
2
3
At x = 0, F = 0
U = 0
and so, c = 0
ax2 bx 3
Thus,
U=

2
3
ax2 bx 3
U = 0, when
=
2
3
3a
i.e., at x =
2b

132 | Mechanics-1
dU
= 0, when F = 0
dx
i.e.,
ax + bx2 = 0
a
i.e., at
x=
b
Graph between U and x will be

3a
2b

a
b

1 2
(i)
k xmax = mg ( h + xmax )
2
From above Eq. (i),
xmax depends upon h and also xmax
depends upon k.
KE of the block will be maximum when it
is just at the point of touching the plank
and at this moment there would no
compression in the spring.
Maximum KE of block = mgh
Correct option is (c).

25. Gain in KE of chain = Decrease in PE of

chain

Correct option is (c).


W A Fs 1
22.
=
=
WB Fs 1

CM (i)

Correct option is (c).


1
mv2A
WA
2
=
=1
WB 1 4 mv2
B
2
v2A 4

=
v2B 1
vA 2

=
vB 1
WA 1
=
WB 1
KA 1

=
KB 1

2r

r/2
r

CM (f)

23. Ui at (1, 1) = k (1 + 1) = 2k

U f at (2, 3) = k (2 + 3) = 5k
W = U f Ui
= 5k 2k = 3 k
Correct option is (b).
24. Gain in PE of spring = Loss of PE of block

When the whole chain has justcome out of


the tube.
1
2 r
or
mv2 = mg
+

2
2
2

v + 2 gr +
2
Correct option is (b).

xmax
PE = 0 level
for block

26. Acceleration of the block will decrease as

the block moves to the right and spring


expands the velocity (v) of block will be
maximum, when
1
1
mv2 = kx2
2
2

Work, Energy and Power |


At this moment, F = k x
F
i.e.,
x=
k
2
k F
F2
or
v2 =
=
m k
km
F

v=
mk

29. Work done on block A in ground frame

= 0.2 45 10

30. N = mg cos
N
= 53 20 m

27. Force on each block = k x


A

kx

kx

10 ms1

1
= 20 N
10
Power of A = 20 4 = 80 W
Power of B = 20 6 = 120 W
Total power = 200 W
i.e., rate of energy transfer = 200 Js 1
Correct option is (c).
28. From O to x compression in the spring
A
m kx

kx

B
3m

+ ive

Average acceleration of A
kx F
aA =
2m
Average acceleration of B
kx
aB =
2 (3 m)
As at maximum compression of the spring
both the blocks would be having same
velocity.
2 a A x = 2 aB x [using v2 = u2 + 2as]
i.e.,
a A = aB
kx F
kx
=
2m
6m
kx
or
kx F =
3
2 kx
or
F=
3
3F
i.e.,
x=
2k
Correct option is (c).

s=|s|

6ms1

= 200

(50 30)
100

= 18 J
Correct option is (b).

Correct option is (b).


4ms1

133

= 37

= 10 10 cos 37
4
= 10 10
5
= 80 N
Work done by N in 2 s
= N cos
= 80 20 cos 53
3
= 80 20
5
= 960 Nm
= 960 J
Correct
option
is
(b).

T + mg
31.
(given)
=5
m
Upper spring
cut

Initially

Lower spring
cut

T = T' + mg
+ ive
T'
mg

T'
mg

T + m 10 = 5 m
i.e., T = 5 m
mg T
a=
m
mg ( T + mg)
=
m
T
=
m

mg

134 | Mechanics-1
( 5 m)
m
= 5 ms 2
Correct option is (b).

33. Total ME = 40 J

PE (max) = 40 J
U (x)
50 J

32. Total ME = 25 J

PE (U) max = 25 J [as KE cant be ive.]

25

U (x)

10

50 J
PE (max)

10

15

PE (max)
PE (max)
6

PE (max)

x (m)

25
10

x (m)

35

10

15

Particle cant be found in the region above


PE (max) line.
10 < x < 5 and 0 < x < 15
Correct option is (a).

Particle can't be found in the regions


above PE (max) line.
It is not possible. Option (d).

More than One Correct Options


1. (i) Acceleration

U = 7 x + 24 y
U
Fx =
= 7
x
U
Fy =
= 24
y
7
ax =
5
24
ay =
5

7 ^ 24 ^
a= i
j m/s2
5
5

i.e.,
i.e.,

v = 3i + 4j
7i

Correct option is (b).


(ii) Velocity at t = 4 s

24 ^
7
= ( 8.6 ^i + 23.2 ^j ) + ^i
j 4
5
5
= 8.6 ^i + 23.2 ^j 5.6 ^i 10.2 ^j
= 3 ^i + 4 ^j
|v|= 5 ms 1

Correct option is (a).

24j

2. U = 100 5x + 100x2

v=u+at

i.e., a is not perpendicular to v.


Options (c) and (d) are incorrect.

|a |= 5 ms 2

i.e.,

(iii) As a v 0, the path of the particle


cant be a circle.

Fx =

=
(100 5x + 100x2 )
x
x

= [ 5 + 200 x ]
= 5 200 x
Fx 5 200 x

ax =
=
0.1
0.1
= 50 2000 x
(i) At 0.05 m from origin
x = + 0.05 m
ax = 50 2000 (0.05)
= 50 100

(first point)

Work, Energy and Power |


2

= 50 m/s
= 50 m/s 2 towards ive x
Correct option is (a).
(ii) At 0.05 m from origin
(second point)
x = 0.05 m
ax = 50 2000( 0.05)
= 50 + 100
= 150 m/s 2
Correct option is (c).
Mean Position
50
m = 0.025 m
a = 0 at x =
2000
(iii) For point 0.05 m from mean position
x = (0.05 + 0.025) m
= 0.075 m
ax ( at x = 0.075 m ) = 50 2000 (0.075)
= 50 150 = 100 m/s 2
= 100 m/s 2 towards ve x-axis.
Correct option is (b).
(iv) For second point 0.05 m from mean
position
x = 0.025 m 0.05 m
= 0.025 m
ax = 50 2000 ( 0.025)
= 100 ms 2
Option (d) is incorrect.
3. (i) If the spring is compressed by x, elastic

1 2
kx gets stored
2
in the spring. Now, if the compressed
spring is released the energy stored in the
spring will be lost. When the spring
attains to natural length.
Work done by spring = Energy stored in
the spring
1
= kx2
2
Correct option is (a).
(ii) If the spring is extended by x, energy
1
stored in the spring would be kx2 . If the
2
extended spring is released the energy
stored in the spring will be lost when the
spring attains its natural length.
potential energy equal to

135

Work done by spring = Energy stored in


the spring
1
= kx2
2
Correct option is (b).
(iii) If spring is initially its natural length
and finally compressed.
1
Work done on (not by) the spring = kx2
2

will be stored in the spring.


Option (c) is incorrect.
(iv) If spring is initially at its natural
length and finally extended.
1
Work done on (not by) the spring = kx2
2

will be stored in the spring.


Option is (d) is incorrect.
4. (i) Work-Energy theorem states that Wnet

(Work done by all forces conservative or


non-conservative, external or internal)
= (KE)
Correct option is (d) is incorrect.
Correct option is (c).
(ii) Work done by non-conservative forces
(i.e., all forces except conservative
forces) lead to decrease in KE and thus
change in mechanical energy takes
place.
Correct option is (b).
(iii) Work done by a conservative force may
be + ve, ve or zero.

s
90
mg
h

mg

mg (a conservative force)

W = m gh
= mghcos
= mgh
(PE increases)

136 | Mechanics-1

W = m gh
= mghcos 0
= + mgh
(PE increases)

W = m g h = mghcos

=0
2

(PE remains same)


Option (a) is incorrect.
5. F is the force by hand or upper disc
( l)
k (l + )
k ( + l)
F
k ( + l)
N

6. At maximum extension x :

k(l + )
3mg

When F is removed, the upper disc


accelerates upwards and when it attains
the position as in figure 2, its acceleration
reduces to zero and the velocity gained by
it takes it further upwards. Restoring
force on the upper plate now acts
downwards and that on the lower plate
acts in the upward direction and would lift
it (lower plate) if
k ( l) > 3 mg
i.e.,
k > 3 mg + kl
or
k > 3 mg + mg
4mg
or
>
k
Correct option is (d).
Correct option is (a) obviously being
incorrect.

3mg
kl = mg
l

decrease in potential energy


increase in spring energy
1

(2m)( g)( x) = kx2


2
4 mg
or
k=
k

of

B=

7. Total work done by internal forces of a

mg

system, which constitute action and


reaction pairs, is always zero and if it is
not so the total work done will not zero.
Correct options are (b) and (c).

Artist
to make
it 3/2 times
of l

8. (i) Work done by conservative forces may

kl
3mg

Uncomprssed/
Unstretched
spring

N = 3 mg + k ( l + )
2mg
= 3 mg + k l +

k
= 3 mg + kl + 2mg
= 3 mg + mg + 2mg
2mg
= 6mg for =

k
Correct option is (b).
N = kl + 3 mg
= mg + 3 mg = 4mg
Correct option is (c).

be + ive, ive or zero as explained in the


answer to question no. 4
Option (a) is incorrect.
Correct option is (b).
Correct option is (c).
(ii) In pure rolling work done by frictional
force (a non-conservative force) is always
zero.
Correct option is (d).
9. In moving from 1 to 2 work done by

conservative force
= U1 U2
= ( 20) ( 10)
= 10 J
Option (a) is incorrect.

Work, Energy and Power |


Option (b) is correct.
Work done by all forces
= ( K 2 + U2 ) ( K 1 + U1)
= [20 + ( 10)] [10 + ( 20)]
= 20 J
Correct option is (c).
Option (d) is incorrect.
N

10.

137

Work done by Normal force ( N ) on block A


W = Ns cos
In motion 2 + ve

as < 90

In motion 1 + ve

as < 90

Correct option is (b).


Work done by force of friction (f )
In motion 1 may be ive if f is directed
upwards along the plane as shown in
figure 1. (Motion 1 being retarded)

N
2

Correct option is (c).

mg

mg

In motion 1 may be + ive if f is directed


downwards along the plane if motion 1 is
accelerated.

Work done by gravity


In motion 2 ive

[as = ]

Correct option is (d).

In motion 1 zero[as = ]
2
Correct option is (a).
Match the Columns
1. As body is displaced from x = 4 m to

x =2m

(d) (p, s).


2. W = Fs cos

(a) Work done by N will be + ive as < 90


(a) (p)
(b) Work done by mg will be ive as =

s = 2im

(a) F = 4 ^i

W = F s = 4 ^i 2 ^i

= 8 unit
(a) (q)
and |W |= 8 unit
(a) (s)

mg

(b) F = 4 ^i 4 ^j

(b) (q)
(c) Work done by force of friction (f )
will be zero as f will be zero.
(c) (r)
(d) Work done by tension (T) will be + ive
as < 90
(d) (p)

W = F s = ( 4 i 4 j) ( 2 i) = 8 unit
(b) (q, s)

(c) F = 4 ^i

W = F s = 8 i unit

and
(c) (p, s)

|W|= 8 unit

(d) F = 4 ^i 4 ^j

W = F s

= ( 4 ^i 4 ^j) ( 2 ^i) = 8 unit


and

|W |= 8 unit

3.
+q
A
( a, 0)

FB

FA = FB
+Q
Fnet = 0

+q
B
(+ a, 0)

F A = force on + Q by + q placed at A
FB = force on + Q by + q placed at B

138 | Mechanics-1
As FA = FB
equilibrium.
(a)

charge

+Q

will

be

FA'

FB'

+q

in

+q
B

+Q

Due to increase in distance between + q (at


A) and+ Q
FA < FA
Due to decrease in distance between + q
(at B ) and + Q
FB > FB
Using FA = FB , we have FB > FA . As there
will net force on + Q which will being + Q to
origin.
Thus, equilibrium will be a stable one.
Y
(b)
Fnet
FB

FA
+Q

+q

+q

As Fnet will be along the increasing


direction of Y , the charge + Q will not
return to origin.
Thus, equilibrium will be an unstable one.
(b) (q)

(c) As explained in (b) the equilibrium will


be an unstable on.
(c) (q)

(d)

x = y line

Fnet

4. (a) From A to B :

FA

kx

Mean
position

mg

1 2
kx
2
Decrease in gravitational. PE of block
= mgx
= ( kx) x = kx2
(a) (q)

(b) From A to B
Increase in KE of block
= Decrease in gravitational PE of block
Increase in spring PE
1 2
2
= kx kx
2
1 2
= kx < Decrease in gravitational PE of
2
block.
(b) (p)

(c) From B to C
1
1
Increase in spring PE = k (2x)2 kx2
2
2
3 2
= kx
2
1
Decrease in KE of block = kx2
2
(KE of block at C will be zero)
(c) (p).
(d) From B to C
decrease in gravitational PE = mgx
Increase in spring PE =

= ( kx) x

A
+q

= kx2

+q

Increase in spring PE =
(d) (p)

3 2
kx
2

Work, Energy and Power |


m m2
21
5. a = 1
g=
g
m1 + m2
2+1

T
a

T
a

T
m2

m1
1g

2g

1
g
3

10
=
m s 2
3
2 m1m2
T=
g
m1 + m2
12
=2
10
1+2
40
N
=
3
Displacement of blocks,
1
s = ut + at2
2
1 2
= at
2
1 10
=
(0.3)2
2 3
= 0.15 m
(a) Work done by gravity on 2 kg block
= mgs cos 0
= 2 10 0.15 (1)
=3J
(a) (r)
(b) Work done by gravity on 1 kg block
= mgs cos
= 1 10 0.15 ( 1)
= 1.5 J
(b) (p).

139

(c) Work done by string on 2 kg block


= Ts cos
40
=
0.15 ( 1)
3
= 20 J
(c) (s).
(d) Work done by string on 1 kg block
= Ts cos 0
40
=
0.15 1
3
=2J
(d) (q).
6. (a) Work done by friction force ( f )

(w.r.t. ground)
f
f' (= f )

= f s cos
=fs
(a) (q).
(b) Work done by friction force on incline
(w.r.t. ground)
= f 0 cos 0
=0
[There being no displacement of incline
w.r.t. ground]
(b) (r).
(c) Work done by a man in lifting a bucket
(T = Tension in rope)
= Ts cos 0
= a + ive quantity
= 0 as T and s both would be in upward
direction.
(c) (p).
(d) Total work done by friction force in (a)
w.r.t. ground
= f s+0= f s
(d) (q).

Circular Motion
Introductory Exercise 7.1

1. In uniform circular motion the magnitude

v2
of acceleration = does not change
r

(iii) Magnitude of net acceleration


= ( ac )2 + ( aT )2
= ( 4 cms 2 )2 + (2 cms 2 )2

while its direction (being always towards


the centre of the circular path) changes.
2. If 0 and are in rad s 1 the value of
2

must be in rad s . But, if 0 and are in


degree s 1 the value of must also be in
degree s 2 . Thus, it is not necessary to
express all angles in radian. One way
change rad into degree using rad = 180.
3. During motion of an object along a curved

path the speed and magnitude of its radial


acceleration may remain constant. Due to
change in direction of motion the velocity
of the object will change even if its speed
is constant. Further, the acceleration will
also change even if the speed is constant.
4. (i) Radial acceleration ( ac )

v2 (2 cms 1)2
=
r
1 cm

= 4 cms 2
(ii) Tangential acceleration (aT )
dv d
=
=
(2t)
dt dt
= 2 cms 2

= 2 5 cms 2

5. |v1|= |v2|= v (say)

2r
T=
v

|PQ|
|v av|=
tPQ
=

r 2
r 2
=
T / 4 r / 2 v

2 2
v

v2

v1

r
O

|v av| 2 2
=
v

6. t = 0 + 4 t

Centripetal acceleration
= tangential acceleration
r 2t = r

2t =
( 4 t)2 = 4
1
t= s
2

Circular Motion |

141

Introductory Exercise 7.2


1. In uniform circular motion of a body the

(a) At rest :

body is never in equilibrium as only one


force (centripetal) acts on the body which
forces the perform circular motion.
9.8 250 (1.5 / 2)
gra
2. vmax =
=
h
1.5

Required CPF = w N =

= 35 ms

l
l

T sin T

T cos
mg

3. (a) T sin = mg

T cos = mr2
g

tan =
r 2
g
g
=
r tan
( l cos ) tan

or

2
=
T

l sin
g
T = 2
l sin
g

1
2
2
= 2
9.8
1
9.8
rve s 1
f = =
1
2
=

9.8
60 rev min 1
2

= 29.9 rev min 1


mg
(b) T =
= 2 mg
sin
= 2 5 9.8 = 69.3N
4.

N=w
2

w'
w

w
2
mv2 w
=
r
2

(i)

At dip :
Required CPF = N w
mv2
(ii)
= N w
r
Comparing Eqs. (i) and (ii),
w
N w =
2
3w 3

N =
= 16 kN
2
2
= 24 kN
(b) At crest on increasing the speed (v), the
value of N will decrease and for
maximum value of v the of N will be just
zero.
mv2max
Thus,
= w0
r
wr

vmax =
m
(as w = mg)
= gr
= 10 250
= 50 ms 1
(c) At dip :
mv2
r
= w + mg
= 2w = 32 kN

N = w +

5. Case I. If the driver turns the vehicle


r
mg

142 | Mechanics-1
m v12
mg
r
[where v1 = maximum speed of vehicle]

v1 gr
Case II. If the driver tries to stop the
vehicle by applying breaks.
Maximum retardation = g

v22 = 0 2 + 2 ( g) r

v2 = 2 gr
= 2 v1
As v2 > v1, driver should apply breaks to
stop the vehicle rather than taking turn.

g
l sin

If is the angle made by the string with


the vertical
+ = 90
i.e.,
= 90

sin = cos
g

=
l cos
g

cos =
l 2

6. In the answer to question 3(a) if we

replace by

Introductory Exercise 7.3

1. |v1|= u

and |v2|=

2. Ball motion from A to B :

v (say)

v = v2 + ( v1)

v1

v2

2R
C
v

L
g

v1

= v2 v1

v u = ( u 2 v 1 ) ( v 2 u 1 )

= u2 v2 + u1 v1 2 v2 v1

|v2 |2

0 2 = u2min + 2 ( g) (2R)

= 4 gR
Ball motion from C to A :
v2 = u2min + 2 ( g) h
= 4 gR 2 gh

v = 2 g (2R h)
u2min

+ |v1| = v + u
2

3. Decrease in KE of bob

= Increase in PE of bob

2
2

= u 2 gL + u
2

|v| = 2 (u2 gL)

|v|= 2 (u2 gL)

v0

KE of bob
= 1/2 mv02
PE of bob
= K (say)

Circular Motion |

143

v20

= 2 gh
v0 = 2 gl (1 cos )
= 2 9.8 5 (1 cos 60 )
= 7 ms 1

PE of bob = K + mgh

1
mv20 = mgh
2

AIEEE Corner
Subjective Questions (Level 1)
1. v = 4 t2

dv
= 8t
dt
i.e.,
aT = 8 3 = 24 ms 2
v = 4 32 = 36 ms 1 (at t = 3 s)
v2 (36)2

= 24 ms 2
ac =
=
4
54

Angle between a net and a t

1 |a c|
tan
=

|a T|

tan 1 1 = 45

2. v = 16 ms 1 and r = 50 m

v2 (16)2
=
= 5.12 ms 2
r
50
= a2c + a2T = (5.12)2 + 82

ac =

anet

(given aT = 8 ms )
= 9.5 ms 2
3. Speed (v) at the highest point ( P)
v = u cos

u cos

v2
= 21.65
R
i.e.,
v = 21.65 2.5 (Q R = 2.5 m)
= 7.36 ms 1
1
(b) aT = a sin 30 = 25 = 12.5 ms 2
2

5. (a) v = 2.0t = 2.0 1

= 2.0 cms 1
v2 (2.0)2

ac =
=
= 4 cms 2
R
1
(b) v = 2.0t
dv

= 2.0
dt
i.e.,
aT = 2.0 cms 2
(c) anet = a2C + a2T
= 4.47 cms 2
2h
6. R = uT = u
g

ac = g

r
P

Centre of curvature

Now,

i.e.,

v = u cos
ac = g
v2
=g
R
v2 u2 cos2
R=
=
g
g
3
2
= 21.65 ms 2

4. (a) ac = a cos 30 = 25

u
h = 2.9 m
h
Range (R) = 10 m

i.e.,

u=R
u2 =

g
2h

R2 g
2h

144 | Mechanics-1
Thus, centripetal acceleration of the stone
(at point P) while in circular motion
u2 R2 g 1
=
=

r
2h r
2
10 9.8
=
2 2.9 1.5
= 112.6 ms 2
18.5
7. v = 18 km / h =
ms 1 = 5 ms 1
18
v2
Angle of banking () = tan 1
rg

mv2
= mg
r
i.e.,

9.

2 T1 cos mg cos
mr

mg
sin
4 100
= 200
4

5

T2 = T1

v
(5)
1
=
= = 0.25
gr 10 10 4

v = r g tan

= 150 N

= 50 10 tan 30
= 17 ms 1
10.

or =

= 8.37 rad s 1
1
8.37
rev
2

=
1
min
60
= 39.94 rev min 1.
(b) From Eq. (ii),

(5)2
10 10
1
= tan 1
40

or T1 cos mg cos = mr2 T1 cos

3
3
2 200 4 10
5
4
=
43

= tan 1

8.

mr 2min = N (i)
and mg = N
(ii)
Solving Eqs. (i) and (ii),
g
min =
r
=

12. (a) For the block not to slip


N
L

N
N

mg

= 4.7 rads 1
11. (a) ( T1 + T2 ) cos = mr2

(i)
T1

T1 sin

4m

4m

mg

10
0.15 3

T1 cos

mr2 T1 cos
T1 sin mg

cos =

T2 cos
T2 sin
T2
mg

mL 2max = N = mg
g
max =
L

(b) If the angular speed is gradually


increased, the block will also have
translational
acceleration
( = L)
besides
centripetal
acceleration
(= L 2 ).
L
anet
2

At
the t) time
of
just
slipping
At t

and ( T1 T2 ) sin = mg
i.e.,
T2 cos = mr2 T1 cos
and
T2 sin = T1 sin mg

(ii)

Circular Motion |
The block will be at he point of slipping,
when
manet = mg
2 2

L24 + 2 L2 = 2 g2
2 g2
4 + 2 = 2
L

or

if

i.e., ( L ) + ( L) = g
or

2 g 2
4
= 2 2
L

or

i.e.,

Ff

= 4 ( 4)2
= 64 N

2r
T
a/ 3
2r
a3
T=
= 2
= 2
1
v
3 Gm
GM 2
a

Now, v =

14. N sin = mr2

T Ff = 1 1 2
64 Ff = ( 4)2

Ff = 48 N
(b)
T = 2 2 2max
T Ff max = 1 1 2max

42max Ff max = 1 2max


i.e.,
32max = mg
mg

max =
3
0.8 1 10
=
3

O'P = r

N cos

O
N

O'

P
N sin
mg

and
N cos = mg
2
sin ( R sin ) 2
sin r
or

=
=
cos
g
cos
g
g
i.e.,
cos =
R 2
For > 0 : cos < 1

and

= 1.63 rad s 1
2 2 2max

2m

Gm 2 GM 2
v=
= a
r 3

or

= 60

15. (a) T = 2 2 2

60
=F 3
2
PQ = QR = RP = a = 2 PO cos 30

a=r 3
For the circular motion of P, Q or R
mv2
= F = F 3
r
mv2 G mm
or
=
3
r
( r 3 )2
1

g
R

i.e., bead will remain at the lowermost


position.
2g
g
1
For
: cos =
=
=
2
g

R
R 2
R

13. F = 2F cos

g
< 1 or >
R2
g
: = 0

i.e.,

145

(c) 100 =

max = 5 rad s 1
Therefore, if the string can sustain a
tension of 1100 N, the angular speed of
the block system will be 5 rad s 1.
In this case the frictional force ( Ff ) on the
block of mass 1 kg will be given by the
relation
100 Ff = 1 1 52

Ff = 75 N
gr l
16.
vmax =
h
10 200 0.75
(2 r = 1.5 m)
=
1.5
= 31.6 ms 1

146 | Mechanics-1
17. If the block just leaves the surface of

19. At point P

sphere at point C
mv2
(i)
= mg cos
r
[v = velocity of block at point C]
2

v = gr cos

v0
P
mg sin
mg

v0

Tmax = 2W
= 2 mg

mv2
l
m 2
=
[v0 + 2 gh]
l
m 2
or 2mg mg sin =
[v0 + 2 gl sin ]
l
52 + 2 g 2 sin
or
2 g g sin =
2
1
Solving, sin =
4
1
i.e,.
= sin 1
4
T mg sin =

vel = v
r

Further, v2 = v20 + 2 gh
(ii)
= v20 + 2 gr (1 cos )
Comparing Eqs. (i) and (ii),
v20 + 2 gr (1 cos ) = gr cos
or (0.5 gr )2 + 2 gr (1 cos ) = gr cos
1
+ 2 (1 cos ) = cos
4
3
i.e.,
= cos 1
4
2
mv
18. T mg cos =
r

h = l sin

20. 8 = 5 + 5 cos
v

T=0

5 cos
8m

= 30

5
4

5m

mg cos

mg

2.5 mg mg cos =

ac =

Thus,

mv2
r

v2
= 25 5 3 ms 2
r
at = g sin = 5 ms 2

anet = ( ac )2 + ( at )2
= (25 5 3 )2 + (5)2
= 625 + 75 + 25 10 3
= 26.60 m/s2

3
5
mv2
T + mg cos =
r
cos =

t = cos 1

3
5

T is just zero.
mv2
r
2
i.e,.
v = 5 g cos
= 30

v = 5.42 ms 1
The height (H) to which the particle will
rise further.

mg cos =

Circular Motion |
2

v sin
30
16
=

2g
2 10 25
24
=
= 0.96 m
25
mv2
8 gR
21. (a) N =
=m
= 8 mg
R
R
v2 = 02 + 2. g.4 R
= 8 gR

F=

H=

147

N + ( mg)

= 64 m2 g2 + m2 g2
= 65 mg
mv 2
(b) N + mg =
R
m
mg + mg =
2 g ( h 2R)
R
or
2R = 2 ( h 2R)
i.e.,
h = 3 R

4R
h = 5R
N
mg
F

Objective Questions (Level 1)


Single Correct Option
1. (a) As the speed (v) is increasing uniformly

the magnitude of centripetal acceleration


v2
= will also keep on increasing besides
r
its direction as usual.
(b) v = kt
(k = constant)
dv

=k
dt
i.e.,
at = k
Although the magnitude of the tangential
acceleration will remain constant but its
direction will keep on changing as the
direction of velocity would be changing.
(c)
v = kt
R = kt
= k t
k
(k =
= constant)
R
d
= k
dt
= constant.
As the direction of (angular acceleration)
is perpendicular to the plane of rotation of
the body, it will remain constant both in
magnitude and direction.
Option (c) is correct.

2. |p1|= |p2 |= mv

p1

p2

p1
p

p2

p1

= 45

p2

p1

p2

p1

= 45, |p|= 2 mv sin 22

1
2

= 0.765 mv

= 90, |p|= 2 mv sin 45


= 1.414 mv

= 180 , |p|= 2 mv sin 90


= 2 mv

= 270, |p|= 2 mv sin 135


= 1.414 mv

(max)

148 | Mechanics-1

= 360, |p|= 2 mv sin 180


= 0 (min)
Option (c) is correct.
3.

PEmax = mgh = mgl (1 cos)

KE = 0

6. Normal acceleration

= Tangential acceleration
v2
= 5 (cms 2 )
R
(Q R = 20 cm)

v = 10 cms 1
Using,
v = 0 + 5t
10 = 5t

t =2s
Option (b) is correct.
7. Mass = 2 kg

Mass per unit length =


KE max = mgl (1 cos )
Option (c) is correct.

[as here PE = 0]

2 kg
2 (0.25) m
T

4.

B
OB

OA
l=

v2 = u2 + 2( g) (2l)
= 4gl 4gl
=0

T
Light O
rigid
rod

1
r =
0.25
1
=
kgm 1
0.25
rev
= 300
min
2 rad
= 300
60 s

m =

u = 4gl

= 4 gl 4 gl
=0
If the mass m is given velocity u ( = 4 gl ) at
point A it will complete circle to reach point
B (the highest point) with zero velocity.
[If in place of light rod these is light string
the minimum value of u at A for the mass
to reach point B will be 5gl and the
minimum velocity at B will be g l.]
5. As explained in question 4 speed at lowest

point
u = 5 gl
At the lowest point
mu2
T mg =
l
m 5 gl
=
l
= 5 mg

T = 6 mg
Option (d) is correct.

(Q r = 0.25)

= 10 rad s 1
From figure
180
2
2T cos
= ( m) r

or
2T sin
= ( m) r2
2

or
2T
= ( m) r2
2
or
T = ( ) r2
or
T = r 2
= 0.25 (10)2
(Q 2 10)
250 N
Option (d) is correct.
8. Maximum speed of car = gr

= 0.3 10 300
= 30 m/s

Circular Motion |
18
km/h
= 30
5
= 108 km/h
Option (c) is correct.

149

Frictional force = manet


= 0.36 103 kg

9. T sin = mr2 = m ( l sin ) 2

725
102 ms 2
3

= 32.4 N
Option (a) is correct.
12. At the lowermost point

T cos

T sin

u=0
h
v2 = 2gh

and

i.e.,

T cos = mg
g
cos =
l 2
2
l = revs 1

2
= (2 rad) s 1

= 4 rads 1
10
5
cos =
=
1 ( 4)2 8
5
= cos 1
8

mg

mv2
l
m 2 gh
T=
+ mg
l
2h

= mg 1 +

T mg =
or

Option (d) is correct.


2
13. A =
rad min 1
3

Option (d) is correct.


A

10. From above question no. 9.

t = ml
100
8
=
1 42 = N
1000
5
Option (b) is correct.
1
11. = rads 1 and R = 25 cm
3
25

at = R =
cms 2
3
Thus,
t = 0 + t
1
[Q t = 2 s]
= 2
3
2
2

aN = R 2t = 25 cms 2
3
725
Thus, anet = a2N + a2t =
cms 2
3

B = 2 rad min 1
BA = B A
2

1
= 2
rad min

3
4
rad min 1
=
3
Time required for B to complete one
revolution w.r.t. A
2
3
=
= min
4 / 3 2

and

= 1.5 min
Option (c) is correct.

150 | Mechanics-1
14. Let all the particles meet at time t

(seconds)
B 2.5 ms1

A
1 ms1

Distance travelled by B in t second :


Distance travelled by C in t second
= 2.5t : 2t
= 5:4
Option (c) is correct.

C
2 ms1

JEE Corner
Assertion and Reason
1. For stopping car :

Maximum retardation
Maximum frictional force
=
m
N mg
=
=
= g
m
m
v2 = u2 + 2as
02 = v2 + (2)( g) d [Q Initial velocity = v]
v2

d=
2 g
For circular turn of car :
Centripetal force
= Maximum frictional force = mg
mv2

= mg
d
v2
i.e., safe radius = d =
d = 2d
g
Thus, Assertion and Reason are both
correct and Reason is the correct
explanation of the Assertion.
Option (a) is correct.

2. a av =

vB v A
t AB

|BD| v 2
=
t AB
t AB

|AB| R 2
=
t AB
t AB

i.e.,

|a av |=

Now,

|v av|=

|a av|

|v av|

v 2
= (angular velocity)
R 2

Thus, assertion is correct.


In circular motion, when speed is
constant, the angular velocity will
obviously be constant; but this reason does
not lead to the result as explained.
Option (b) is correct.
3. A frame moving in a circle with constant

speed can never be an inertial frame as


the frame is not moving with constant
velocity (due to change in direction).
Reason that the frame is having constant
acceleration is false.
Option (c) is correct.

will always be 90

vB

vA

BD
=
t AB

4. If speed is constant angle between v and a

a = ac

vA

va = 0

Circular Motion |

If speed is increasing angle between v

and a will be less than 90.

(2)2
=2m
2

a T = ^i ms 2

ac

Speed is decreasing (as a T is ive) at a


rate of 1 ms 1 per second i.e., 1 ms 2 .
Both Assertion and the Reason are correct
but reason has nothing to do with the
assertion.
Option (b) is correct.

v a will be positive.

v
ac

R=

aT

aT

6. a = a T + a c (Reason)

ac

ac

Assertion is correct.

Reason
as
both
are
v =0
perpendicular to each other.
Reason is also true but not the correct
explanation of the assertion.
Option (b) is correct.

v2
+ g2
r

|a |=

|a |> g (Assertion)

We see that both Assertion and the


Reason are correct and the reason is the
correct explanation of assertion.
Option (a) is correct.
7. At

points A and C :
momentarily at rest.

5. v = 2 i ms 1

aT = g

v a will be negative.

If speed is decreasing angle between v and

a will be greater than 90.

151

The

bob

is

anet

x
2j
v = 2z

a = ^i + 2 ^j ms 2

ac

= |a c |=

a c = 2 j ms

2 ms

v2
R

i.e.,

v = |v|= 2 ms 1
ac =

A
B

v = 0 (Reason)
v2

=0
R
but net acceleration is not zero (see figure)
i.e., Assertion is false.
Option (d) is correct.

|a c |=

152 | Mechanics-1
8. v (speed) = 4 t 12

For t < 3 (time unit) speed is negative,


which cant describe a motion. Thus,
assertion is correct.
As speed can be changed linearly with
time, the reason is false.
Option (c) is correct.
9. In

circular motion the acceleration


changes regularly where as in projectile
motion it is constant. Thus, in circular

motion we can't apply v = u + a t directly,


whereas in projectile motion we can say
reason that in circular motion gravity has
no role is wrong.
Option (c) is correct.

10. N = mg cos
N
O
N sin

mg cos

mg

Therefore, assertion is wrong.


Particle performs circular motion due to
N sin
mv2

= N sin
r

N cos is balanced by mg (weight of


particle).
Acceleration is not along the surface of the
funnel. It is along the centre O of the
circle. Thus, reason is true.
Option (d) is correct.
mv2
11. Centripetal force
= N + mg
r
N
mg

i.e., Centripetal force (reason ) wt ( mg) of


water
for
N = 0, v = gr
If at the top of the circular path v gr
i.e., if bucket moved fast, the water will
not fall (Assertion).
As assertion and reason both are true and
reason is the correct explanation of the
assertion of the option would be (a).

Objective Questions (Level 2)


Single Correct Option

1
2

1
2

1. KE mv2 = Change in PE k( x)2

x = Length of spring (Collar at B)


Length of spring (Collar at A)
= (7 + 5)2 + 52 m 7m = 6 m
1
1
Thus,
2 v2 = 200 62
2
2
2
i.e.,
v = 3600
mv2
Normal reaction =
r
2 3600
=
= 1440 N
5
Option (a) is correct.

2. The particle will remain in equilibrium till

is constant. Any change in the value of


will displace the particle up (if increase)
and down (if decreases). Thus, the
equilibrium is unstable.
Option (b) is correct.
3. Centripetal force = mg

or
or
or

mr2 = mg
5a 2
= g
4
4g
2 =
15a

Option (d) is correct.

as =

Circular Motion |
4. Acceleration at B = Acceleration at A

v2

= g sin
r
2 gr (1 cos )
or
= g sin
r
or
2 (1 cos ) = sin
or
[2 (1 cos )]2 = 1 cos2
or
(5 cos 3) (cos 1) = 0
As cos = 1, i.e., = 0 is not possible.
3
cos =
5
1 3
i.e.,
= cos
5
Option (c) is correct.

5. At point P (for the circular motion)

mv2
R
If at point P skier leaves the hemisphere.
N =0
mv2

mg cos =
R
m
R

or
mg cos =
2g h +

R
4
m
R

or
mg cos =
2 g R (1 cos ) +
R
4

cos = 2 (1 cos ) +
4

1 5
i.e.,
= cos
6
Option (c) is correct.
mg cos N =

6. Velocity at B

v = 2 gh
= 2 gR (cos 2 cos 1)
= 2 gR (cos 37 cos 53 )
4 3
= 2 gR
3 5
=

2 gR
5

For the circular motion at B, when block


just leaves the track
v2
= g cos 2
R1

v
2R
=
g cos 2 5 cos 2
2R
=
5 cos 37
2R
=
4
5
5
R
=
2
Option (c) is correct.
mv2
7.
mg cos =
a
a
42 + 2 g
4
or
g cos =
a
a
a
g
u2 + 2 g
4
4
or g
=
a
a
3
a
or
g a = u2 + 2 g
4
4
ag

u=
2
Option (c) is correct.
v2
4
8.
= 2
r
r
2

v=
r
or

R1 =

Momentum = mv
2m
=
r
9.

N cos = mr2
and N sin = mg

N = m g2 + r24
2
= m g2 + r2
T

= 10 10 + (0.5)
1.5
a

= 128 N
Option (b) is correct.
1
10.
= 0 t + t2
2
1

= 0 + t t
2

153

154 | Mechanics-1
v 1 1 v2 4 R
= +

t
R 2 R 4 R v
v
v 4R
=
R 2R v

2H
g

2
g
= 2
T
2H
2g
=
H
2
12. (minute hand) =
rad s 1
3600
2
rads 1
(second hand) =
60
For second hand to meet minute hand for
the first time.
2 + Angle moved by minute hand in t
second
= Angle moved by second hand in t second
2
2
or
2 +
t=
t
3600
60
t
t
1=

60 3600
t
59
1=

60 60
3600
s

t=
59
Option (d) is correct.
=

13.

( PR)2 + (QR)2 = ( PQ)2


(v A 2)2 + (vB 2)2 = 302
v2A + v2B = 225

60

30

B'

PR
= cos 30
PQ
vA 2
3
=
30
2

v A = 7.5 3 ms 1
Substituting value of v A in Eq. (i),
vB = 7.5 ms 1
mv2
14. mg cos N =
r
When breaks off N = 0
mv2

mg cos =
r
Further,

T=

vB

30 m

t=2s

vB
= AD

= 4
= 2 rev.
1
H = gT2
2

11.

(i)

sin
mg

or

g cos =

Acceleration
= g sin

mg

2 gr (1 cos )
r
3

or

cos = 2 (1 cos )
2
or
cos =
3
Acceleration of particle when it leaves
sphere
= g sin
g 5
=
3
Option (b) is correct.

Circular Motion |
15. For minimum velocity (v) :

tan = tan 45 = 1
=1
As
= tan
is the angle of repose.
N

(given)

N cos

N cos
N

N sin

Therefore, the automobile will be at the


point of slipping when its velocity is zero
For maximum velocity (v )
mv 2
N sin + N cos =
r
Also
N cos = mg + N sin
i.e.,
(ii)
N (cos sin ) = mg
Dividing Eq. (i) by Eq. (ii),
sin + cos v 2
=
cos sin gr

aT = g
anet = a2c + a2T
= (3 g)2 + g2 = g 10

Option (a) s correct.


17.

and

N sin

N = mr2
N = mg
g
=
r2
10
= 0.2
=
0.2(5)2

19. At point A : Velocity is zero and as such its

vertical component will also be zero.


At point B : Velocity is completely
horizontal and as such its vertical
component will again be zero.
T

u = 5 gR
Particle's velocity (v) when it is at B i.e.,
when its velocity is vertical would be given
by the relation
v
R

u = 5gR

ac =

v2
= 3g
R

mg

= 31.62 m/s

the value of

At B :

16. For the particle to just complete the circle

v2 = u2 + 2 ( g) R
= 5 gR 2 gR
= 3 gR

Option (c) is correct.


gra
18. vmax =
h
10 200 075
.
=
1.5

Now, as = 45 and = 1
v =
Option (d) is correct.

155

v
v1

In figure, TP = R cos
At point P :
v2 = 2 g ( R cos )
v = 2 gR cos1/ 2

v = v sin = 2 gR cos1/ 2 sin


For v to be maximum
d
v = 0
d
d
i.e.,
2 gR cos1/ 2 sin = 0
d
1
or cos1/ 2 cos + cos 1/ 2 ( sin ) sin = 0
2
sin2
or
cos cos
=0
2 cos

156 | Mechanics-1
2 cos2 sin2 = 0
3 cos2 = 1

or
or

= cos 1

or

21. At highest point B


B

1
3

T
mg

Option (b) is correct.


20. At any time

at = ac
dv v2
i.e.,
=
dt R
dt
2

v dv = R + k
v 2 + 1
t
i.e.,
=
+k
2 + 1 R
1 1
or
=
+k
v R
At t = 0, v = v0
(given)
1
0

=
+k
v0 R
1
i.e.,
k=
v0
1 t
1
Thus,
=

v R v0
1 tv R
or
= 0
v
Rv0
Rv0
or
u=
R tv0
v0
or
=
R tv0
d
v0
i.e.,
=
dt R tv0
v0
or
d =
dt
R tv0
2
T
v
0 d = 0 R 0tv0 dt
or
or
or
or

2 = [log e ( R tv0 )]T0


= log e R log e ( R Tv0 )
R Tv0
log e =
= 2
R
T
1 v0 = e2
r
R
T=
(1 e2 )
v0

Option (c) is correct.

R
u

mv2
l
(where v = velocity at point B)
Thus,
T =0
mv2
if
mg =
l
2
i.e.,
v = gl
If u = velocity at the lowest point A
T + mg =

v2 = u2 + 2 ( g) (2l)
or

gl = u2 4 gh

u2 = 5 gl

i.e.,

u = 5 gl

(If T = 0, T a = 0)
Option (b) is correct.
22. For any value of u at the lowest point both

T and a will be towards the centre of the


circle and thus

T a will be positive.
Option (d) is correct.
23. Change in PE of the system
m
M

l
l/2

l
+ mgl
2
M

=
+ m gl
2

= Mg

Circular Motion |
2

24. Decrease in KE = Increase in PE

1
M

mu2 =
+ m gl
2

i.e.,

Initial speed given to ball,


M + 2m
u=
gl

m
25. Maximum and minimum velocities will be

respectively at the lowest and the highest


points.
v

or
i.e.,

= (2 v) 4 gL
3v2 = 4 gL
4
gL
v = gL = 2
3
3

157

(Qu = 2 v)

Option (b) is correct.


gL
26. u = 2 v = 4
3
KE at the lowermost position =

(Min. velocity)

1
mu2
2

8 mgL
3

27. Let = velocity of the particle when

u
(Max. velocity)

v2 = u2 + 2 ( g) 2L
= u2 4 gl

moving downwards.

2 = v2 + 2 gL
4 gl
=
+ 2 gl
3
10 gl
or
=
3
Option (a) is correct.

More than One Correct Options


1. At point C :
vmin

v'

A aT = 0
g
ac(min)

B
T
mg

B
aT = g sin

C
aT = g
ac(max)

g
2

vmax

or

D aT = 0

or
aT is maximum and ac is somewhere
between maximum value (at D) and
minimum value (at A).
Option (b) is correct.
At point D :
ac is maximum while be aT is minimum.
Option (d) is correct.
2. At point B :

T + mg =

mv 2
l

i.e.,

2mg + mg =

mv
l

v 2 = 3 gl
v = 3 gl

Option (d) is correct.


Now,

v2 = v 2 + 2 g (2l)

v2 = 3 gl + 4 gl
= 7 gl

i.e.,
v = 7 gl
Option (b) is correct.

158 | Mechanics-1
4. Particle

cant have uniform motion


because of change in direction of motion
i.e., its velocity value. [Option (a)]
Particle cant have uniformly accelerated
motion as acceleration changes direction
even if speed is constant. [Option (b)]
Particle cant have not force equal to zero
as centripetal force would be required for
the circular motion. [Option (c)]

3. N sin = mg
N
r

N sin

N cos
mg
h

5. For this see figure in answer 3.

N cos = mr2
g
g
=
tan =
r tan
r2
2
g
=
T
r tan
r tan
T = 2
g

i.e.,
or

If is increased N cos will increase.


Thus, N will increase (as is constant)
[Option (b)]
And as such net force
= N 2 + m2 g2 + 2 N cos (90 + )

If is increase r will also increase and as


such T will increase.
Option (c) is correct.
h tan2
T = 2
g

on the block will increase.


Option (a) is correct.
As N increases, the value of N sin
(acting opposite to mg) will increase and
the block will upwards i.e., h will increase.

Thus, if h is increase, T will increase.


Option (a) is correct.
Match the Columns
1. At point B :
2

v = u 2 gl
= 12 gl 2 gl
i.e.,
v = 10 gl
= 10 10 1
= 10 ms 1
b
(a) (p).
A
Acceleration of bob :
v2 10 g
ac =
=
= 100 ms 2
l
1
aT = g = 10 ms 2
anet = a2c + a2T
= 1002 + 102 ms 2
(b) (s).
Tension in string :
mv2 1 100
T=
=
= 100 N
l
1

(c) (r).
Tangential acceleration of bob :
aT = g = 10 ms 2
(d) (p).
2. v = 2 t

dv
=2
dt

i.e., a = 2 ms 2
At t = 1 s : v = 2 ms 1
v2 22

ac =
=
= 2 ms 2
r
2
aT = 2 ms 2

anet = 22 + 22 = 2 2 ms 2

Angle between a net and v = 45


[As ac = aT ]

Circular Motion |

F=f
F
=1
f

(a) a v = |a ||v|cos 45
1
2
1
= 2 2 2
2

= a net v

= 4 unit.
Thus, (a) (r).

(b) |a |= |v|||sin 90

[ will be perpendicular to the plane of


circle]
= anet
v
= anet
r
2
=2 2
r
= 2 2 unit
Thus, (b) (p)

(c)

v = |v|||sin 90
v
= v = v
r
2
(2)
=
2
= 2 unit
Thus, (c) (q)

i.e.,

(c) (r).
If v is increased F will increase which will
automatically increase the value of f .
(d) (p).
4. Speed of particle is constant

= 4 unit
Thus, (d) (r)
M
v2 h
3. N1 =
g

2
ra
On increase v, the value of N1 will
decrease.
Thus, (a) (q).
M
v2 h
N2 =
g +

2
ra
On increasing v, the value of N2 will
increase.
(b) (q).
As the centripetal force ( F ) would be
provided by the frictional force (f )

|a |=

|v|2

|r |
( 4)2 + ( a)2

( 6)2 + ( b)2 =

(3)2 + ( 4)2

36 + b2 =

16 + a2
5

(i)

(A) r v = |r ||v|cos 90 = 0
(3 ^i 4 ^j) ( 4 ^i a ^j) = 0

i.e.,

12 + 4 a = 0
a= 3
(a) (s).
(B) Substituting value of a ( = 3) in Eq. (i)
36 + b2 = 5
36 + b2 = 25

(d) |v a |= |v||a |sin 45


1
= v anet
2

159

b = 25 36
(b) (s).

(C) r = 3 ^i 4 ^j

r = |r |= 5

(c) (r).

(D) ( v a ) = |v||a |sin 90 k


^

= |v||a | k

Thus, r ( v a )

= r|
v||a |k

= (3 ^i 4 ^j) |v||a|k = 0
^

[as ^i k = 0, ^j k = 0]
(d) (s).

160 | Mechanics-1
5. (A) As speed is constant

Average speed = 1 ms

v2

v1

v mg

R 2
R 2
2 2
=
1 =
AB arc
R

2
speed

(b) (q)
(C) Modulus of average acceleration

v1

= |a c | [as speed is constant]


A

|v v1| |v| v 2
= 2
=
=
R
t AB
t AB

2
= 2 [ Q v = 1 ms 1 and R =

(a) (s)
(B) Modulus of average velocity =

AB
t AB

(c) (r)

2
]

Centre of Mass, Conservation


of Linear Momentum, Impulse
and Collision

Introductory Exercise 8.1


1. If

a body is placed in a uniform


gravitational field, the CG of the body
coincides with the CM of the body.
n

rCM

mi ri

i =1
n

mi

i =1
n

while

mi g i ri

r CM =

i =1
n

mi g i

i =1

If a body is placed in a uniformly

increasing gravitational field ( g ) in the


upward direction the CG of the body will
be higher level than the CM. And, if the
body is placed in a uniformly decreasing
gravitational field in the upward direction
the CG will be at a lower level the CM.

2. The centre of mass of a rigid body may lie

inside, on the surface and even outside the


body. The CM of a solid uniform sphere is
at its centre. The CM of a solid ring is at
the centre of the ring which lies outside
the mass of the body thus, the statement
is false. (For further details see answer to
1 Assertion and Reason JEE corner).
3. Centre of mass always lies on the axis of
symmetry of the body, if it exists. The
statement is thus true.
4. Statement is true.
5. As more mass is towards base.
r
Distance < .
4
6. If

two equal masses are kept at


co-ordinates (R, 0) and ( R, 0), then their
centre of mass will lie at origin.

7.

3 kg (1/2, 3/2)

CG
CM
g
increasing

g
decreasing

CM
CG

CG shifts from CM according to the


magnitude
and
direction
of
the
gravitational field (by some other agency
eg, earth) in which the body is placed.
In zero gravitational field CG has no
meaning while CM still exists, as usual.

1 kg
(0, 0)

2 kg
(1, 0)

m1 X1 + m2 X2
m1 + m2
m1 y1 + m2 y2
=
m1 + m2

X CM =
YCM
2
r = X 2CM + YCM

= Distance of centre of mass from A

162 | Mechanics-1
y

8.

YCM =

A1 y1 A2 y2
A1 A2

9. A1 = 4 a2 , x1 = a, y1 = a
3a
3a
A2 = a2 , x2 =
, y2 =
2
2
x

Introductory Exercise 8.2


1. Method 1
t=0s
(10 x)
CM (i)

1 kg

2 kg
y

2m
t=1s

7y
CM (f)

1x = 2 (10 x)
20
x=
cm
3

60

2.

in

At t = 0s

2 kg

+v
e

mg
s

1 kg
A
t=0s

60

mg
s

in

30

30

1m
t=1s

1 y = 2 (7 y)
14
cm

y=
3
Displacement of CM
= Position of CM ( f ) Position of CM ( i)
= ( y + 2) ( x)
14
20
=
+ 2
3
3
=0
Method 2
M vCM = m1v1 + m2v2
= 1 ( 2 ms 1) + 2 ( 1 ms 1)
=0
As velocity of CM is zero, there will not be
any change in the position of the CM.

mg sin 60 T = ma
T mg sin 30 = ma
Adding above equations,
mg (sin 60 sin 30 ) = 2 ma
3 1
a= g

a1 and a2

a CM =
or

are at right angle

m a1 + m a2 1
= ( a1 + a2 )
2m
2
a
g( 3 1)

|a CM|=
=
2
4 2

(i)
(ii)

Centre of Mass, Conservation of Linear Momentum, Impulse and Collision |

163

Introductory Exercise 8.3


1. vCM

20 v + 60 ( + 3)
=
=0
20 + 60

vCM =
60 kg + ve

$
20 20 $i + 30 20 $i + 40 20 k
20 + 30 + 40
y
(10i + 20k) cms1

20 kg

30 g
v

3 m/s
20 kg

60 kg

[As there is no force along horizontal


direction].
Velocity of trolley (v) = 9 ms 1
Total energy produced by man
= KE of man + KE of trolley
1
1
= 60 32 + 20 ( 9)2 = 1.08 kJ
2
2
2. On streching and then releasing the

spring the restoring force on each block at


instant will be same (according to
Newtons 3rd law of motion). Now, as force
is same momentum p of each block will
also be same (t being same) [As according
to Newtons second law of motion rate of
change of momentum of a body is directly
proportional to the net force applied on
the body.]
p2
Now, as KE =
,
2m
KE of blocks at any instant will be
inversely proportional to their respective
masses.
3. As no external force acts on the system of

particles, the velocity of CM shall not


change. Thus,
y
20 cms1
30 g
20 cms1
j
x
20 cms1

20 g

20 g

vCM =

20 0 + 30 (10 ^i + 20 k) + 40 v
20 + 30 + 40
^

300 ^i + 600 ^j+ 40 k = 400 ^i + 600 ^j+ 800 k

40 v = 100 ^i + 600 ^j + 200 k

v = ( 2.5 ^i + 15 ^j + 5 k ) cms 1

i.e.,

4. Velocity of projectile at the highest point

before explosion = 10 2 ^i ms 1

^
v = 10 2 i

20
45

20 cos 45
= 102 ms1

R/2

As no extra external force would be acting


during explosion, the velocity of CM will
not change
m m
0+
v
2
2 = 10 2 ^i
m

v = 20 2 ^i
R
Range of rest half part =
2
2
(as the velocity of the projectile has
doubled at the highest point)
u2
(as = 45)
= R=
g

=
z

Rest

(20)2
= 40 m
10

164 | Mechanics-1
Therefore, the rest half part will land at a
40
distance of 40 +
m i.e., 60 m from the

= Height of point P + Extra maximum


height attained
by second half part

point of projection.

1
( 2000)2 sin2
= 20 1 g 12 +

2
2g
2000 1
= 15 +

2 10 5

20 2 i
u

= 15 + 20
= 35 m

5. At point P

6. Momentum of platform + boy + stone

along x-axis after throwing stone = before


throwing stone
1

(60 + 40) v + 110 = 0

u = (40)2 + (20)2
P
20 ms1
t=0s

202 ms

1
= 2000 ms
t = 15

15 m

45

v=

5 2
ms 1
100

60 kg

Horizontal velocity = Initial horizontal


velocity
1
= 20 2 cos 45 ms
= 20 ms 1
Vertical velocity
= 20 2 sin 45 g 1 (as t = 1s)
= 70 ms 1

40 kg

10 cm

1 kg
X

Time of flight of stone

u = 2 v = 40 ^i + 20 ^j ms 1

Angle of projection () of 2nd half part


after explosion at point P.
20
= tan 1
5
40
= tan

1 1

10 ms1

v (velocity of projectile at point P i.e., at


t = 15 just before explosion)
Now, as the projectile breaks up into two
equal parts and one part comes to rest, the
velocity of other half part after explosion
will be

45
1 kg

= 20 ^i + 10 ^j

10 ms1

T = 2 s

2 10 sin 45
= 2s
g

Horizontal displacement of platform


(+ boy)
=vT
5 2
1
m = 10 cm
=
2=
100
10
7. Thrust due to the upward component of
the velocity of the bullet will rotate the
movable end of the barrel and thus the
bullet leaving the barrel will travelling at
an angle greater than 45 when it comes
out of the barrel.

1
sin =
5
Maximum height attained by second half
part

Bullet
Bullet

= 45

' >

Centre of Mass, Conservation of Linear Momentum, Impulse and Collision |

165

Introductory Exercise 8.4


1. To just lift rocket off the launching pad

Thrust force = Weight


dm
v
= mg
dt
dm mg

=
dt
v
(20 + 180) 9.8
=
1.6 103

or
or

i.e.,

180 kg
=9s
20 kgs 1

v = 9.8 9 + 1.6 103 ln

200
20

= 3.6 kms 1
2. Mass at time t, m = m0 t

dm
=
dt
ma = thrust force mg

= 1.225 kgs 1
(i) Rate of consumption of fuel = 2 kgs 1
Time required for the consumption of fuel
180 kg
t=
= 90 s
2 kgs 1
Ultimate speed gained by rocket
m
v = u gt + v log e 0
m

t=

(i)

Substituting u = 0, v = 1.6 kms 1,


m0 = (20 + 180) kg and m = 20 kg in Eq. (i).
200
v = 9.8 90 + 1.6 103 ln

20
1
= 2.8 kms
(iii) Rate of consumption of fuel = 20 kgs 1

dm
ma = v
mg
dt

or
or

ma = v mg
d2 x
or ( m0 t) 2 = u ( m0 t) g (Qv = u)
dt
m
3. v = u gt + v ln 0
m
m0
= 0 gt + u ln
t

m0 1

3
3
(at t = 1 s)
= g 1 + u ln
2
3
= u ln g
2

Introductory Exercise 8.5

1. u (at t = 0 s) = ( 10 3 i + 10 j ) ms 1
20 m/s
60

At t = 1 s
Horizontal velocity = 10 3 ^i ms 1
Vertical velocity = (10 g 1) ^j ms 1
= 0 ^j ms 1

Change in velocity, v = v u
= ( 10 3 ) ^i (10 3 ^i + 10 ^j)
= 10 ^j ms 1
i.e.,

= Change in momentum in the time


interval
t = 0 s to t = 2 s
= m [(Velocity at t = 2 s)
(Velocity at t = 0 s)]
^
^
1
= 2 kg [( 4 i + 4 j) ms ( 4 ^j ) ms 1 ]
= 8 ^i Ns

(Q 1 kg ms 1 = 1 Ns)

3. Spring will become taut when the ball

would go down by 2 m.

v = 10 3 i ms

2. Impulse ( J ) imparted

v = 10 ms 1, downwards.

v2 = u2 + 2 g 2
or
v = 2 10 ms 1
p = m v
= 1(2 10 0) kg ms 1
= 2 10 kg ms 1
Impulse imparted = 2 10 Ns.

2m

166 | Mechanics-1
Introductory Exercise 8.6
1. At maximum extension of the spring both

blocks will move with same velocity v.


+ ive
1.0 ms1

2.0 ms1

3 kg

of

Rest

m m

or
or

1 2 K
K = m
+ EPE max
4 m
K
EPE max =
2

v1'

m2

m1

m2

m1

K (Transfer of KE of particle of mass


m2 )
1
1
= m2 v22 m2 v 22
2
2
2

From above we conclude that K will be


maximum when
m1 m2 = 0
m2
i.e.,
=1
m1

During collision the EPE of the system


would be at its maximum value when both
the particles move with same velocity V
given by the relation
mv + m 0 = ( m + m) V
(Law of conservation of momentum)
v

V =
2
1 2K
=
2 m
Applying
law
of
conservation
mechanical energy
1
K + 0 = ( m + m) V 2 + EPE max
2
or
K = mV 2 + EPE max

v2'

m m2
1
1
m2 v22 m2 1
v2
2
2
m1 + m2
2

m m2
1
= m2 v22 1 1

2
m1 + m2

1
mv2 = K
2
2K
v=
m

v1 = 0

2. Kinetic energy of particle = K

m m2
2 m2
v2 = 1
v2 +
v1
m1 + m2
m1 + m2
m m2
(as v1 = 0)
= 1
v2
m1 + m2

6 kg

( 1.0) 3 + ( + 2.0) 6 = (3 + 6) v

v = + 1 m/s
Applying
law
of
conservation
mechanical energy
1
1
1
1
3 ( 1)2 + 6 (2)2 = 9 12 + x2
2
2
2
k
1
Substituting k = 200 Nm
x = 0.3 m = 30 cm

3. Let us consider the following case

of

4. Continuing from the previous answer

K
=1
K
=

m1 m2

m1 + m2

( m1 + m2 )2 ( m1 m2 )2
( m1 + m2 )2
4 m1m2
=
( m1 + m2 )2

5. Substituting m2 = m and m1 = 2m in the

result of the previous question no. 4.


u

Rest

2m

K 4 (2m) ( m)
=
K
(2m + m)2
8
=
9

Centre of Mass, Conservation of Linear Momentum, Impulse and Collision |

Total number collisions between the


balls = 2.
m m1
2m1
7. v 2 = 2
v2 +
v1
m1 + m2
m1 + m2

6. As the collision is elastic


u

Rest

v1

v2

m1

m2

m1

m2

v1 =
and

m1 m2
u
m1 + m2

v2
m2

v2 = v1 + u
m m2
= 1
u+u
m1 + m2
2 m1
=
u
( m1 + m2 )
B

4m

4m

4m
Rest

8. In elastic collision of two bodies of equal

masses, the velocities get interchanged.


Before collision of A with B
velocity of A = + v
velocity of B = zero
Therefore, after collision of A with B
velocity of A = zero
velocity of B = + v
After collision of B with wall

2v
5

velocity of B = v
Before collision of B with A
velocity of A = zero

m 4m 3
v
m + 4m 5
9
=
v
25
9
3
6
=
v + v =
v
5
25
25

vB =

velocity of B = v

vA

Therefore, after collision of B with A


velocity of A = v
velocity of B = zero

After second collision : of B with A


A
6v
25

4m

m1

Ans. No.

4m

4m

After

v2 = v1 and v 1 = v2

3v
5

m2

However, if m1 = m2

After collision of B with C


m 4m
vB =
v
1
m + 4m
3
= v
5
3
2
and
vC = v + v = v
1
5
5
Before second collision of B with A
B

m1

v1'

We see that v 2 v1 and v 1 v2 .

C
v

v2'

Elastic collision

Rest

Before

v1

m m2
2m2
v 1 = 1
v1 +
v2
m1 + m2
m1 + m2

Before first collision of B with C


A

167

B
4m

C
9v
25

4m

2v
5

As velocity of C is greater than that of B,


no further collision will take place.

Now, no more collision will take place as A


will move towards left with speed v
leaving B at rest to its right side.
Thus, we see that speeds of the balls
remains unchanged after all the possible
collisions have taken place.

168 | Mechanics-1
Introductory Exercise 8.7
1. Applying

law

of

conservation

of

momentum
v

Rest

v'

2v
3

vmin = 2 m / s
vmax = 2 2 m / s
3. A ( m) hitting B (2m)
B

2v
mv = mv + m
3
v
v =
3
velocity of separation
e=
velocity of approach
2v
2v v
v

3
3 =1
=
= 3
v0
v
3

i.e.,

1 + e
1 e
v and v 2 =
v
2
2

2. v 1 =
v

Rest

v2'

v1'

A
B
Before collision

A
B
After collision

KE after collision = 0.2 J


(given)
1
1
2
2

mv2 + mv 1 = 0.2
2
2
or
(Q m = 0.1 kg)
v 22 + v 21 = 4
2
2
1 e 2 1 + e 2
or

v +
v =4
2
2
or

v2
(2 + 2e2 ) = 4
4

Extreme cases
If collision is perfectly in elastic i.e., e = 0
v2
(2) = 4
4
i.e.,
v = 2 2 m/s
If collision is perfectly elastic i.e., e = 1
v2
(2 + 2) = 4
4
i.e.,
v = 2 m/s

2m
m

v
A

A collision is elastic
2 ( m)
2
vB =
v= v
2m + m
3
m 2m
v
and
vA =
v=
2m + m
3
Velocity of B w.r.t. A = vB v A
2
v
= v = v
3
3
Next collision between the balls will
take place after time
2 r
t=
v
4. As collision is elastic
v2'

v'1

A
B
Before collision

p
m

Rest

After collision

1 e p
v 2 =

2 m
Now, Momentum of A before impact
Impulse given by B on A
= Momentum of A after impact.
p J = m v 2
1 e p
or
p J = m

2 m
2J
or
e=
1
p

Centre of Mass, Conservation of Linear Momentum, Impulse and Collision |

169

Introductory Exercise 8.8


V 0
v0
V
=
v
m
=
M

1. v0 sin = v cos ( + )
v0

v0

co

sin

v0 s
in (
+

s v0

v co

4.

s (

+ )

Rest
PRX = 30
QRX = 30

P
R

v0 cos = sin ( + ) (Impact being elastic)

tan = cot( + )

or
cot = cot( + )
2

or
= +
2

or
= 2
2
2. Speed after n impacts

Speed before first impact = u ( = 2 gh)


Speed before one impact = e u
Speed after 2 impacts = e ( eu) = e2u
........ ......... ........ ........ ........ .........
Speed after n impacts = enu
Height ( H ) upto which the ball rebounds
after nth rebound
( enu)2
H=
2g
2n

Rest
30
X Angle OA

30
C
B

Applying
law
momentum

conservation

of

mu = Resultant momentum of A and B


balls along the axis of X.
or

A
B
Before collision

B
After collision

m
v
M
Velocity of separation
e=
Velocity of approach

of

Rest

V =

As the balls are of same size, the centres


of the balls P, Q and R will be at vertices
of an equilateral triangle when ball C just
strikes balls A and B symmetrically and
as such the balls A and B will follow the
path as shown below

Rest

u
2g

3. mv + 0 = 0 + MV
m

Rest
Q

= e2 n h

or
or

mu = 2 mv cos 30
3
u = 2v
2
u=v 3

As the ball C will strike ball A (and as well


as ball B) with velocity ucos 30

170 | Mechanics-1
Velocity of approach of ball C towards ball A
= ucos 30 0
= ucos 30
=u 3
6.
Velocity of separation of ball A away from
ball C = v
u sin
v

e=
3
u

A
2
v
=
3
(v 3 )
2
2
=
3

after impact = 2 ^j
Velocity after impact = ^i + 2 ^j
At A, u|| = u cos
u = u sin
v

e u sin
H1
T1 B

u cos

u sin

R1

T2

H2

R2

At B
u|| = u cos
u = e u sin
2u sin
T1
1
g
=
=
T2 2 e u sin e
g

5. x-component of velocity

45
45

2u
Using T = g

2 ^j

45 v = 2i^ + 2j^
X

2 ^i

before impact = 2 ^i
after impact = e 2 ^i
1
= 2 ^i
2
= ^i
y-component of velocity
before impact = 2 ^j

2 (u sin ) (u cos )
2 u u||

R1
2
=
Using R =

R2 2 ( eu sin ) u cos
g
g
1
=
e
(u sin )2

H1
u2
2g
=
Using H =

2
H2 ( e u sin )
2g

2g
1
= 2
e

AIEEE Corner
Subjective Questions (Level 1)
1.
D 4kg

1m

C
3kg

P
(CM)
1m

yCM

1m

B 2kg
A 1kg

10 + 21 + 3 1 + 4 0
1+2+3 + 4
5
m
=
10
10 + 20 + 3 1 + 4 1
=
1+2+3 + 4
7
m
=
10

xCM =

1m

Centre of Mass, Conservation of Linear Momentum, Impulse and Collision |


2

5
7
AP2 = x2CM + y2CM = +
10
10
2. xCM

4
V 0 + a 3 b
3

xCM =
4
3
V a
3
4
a 3b
3
=
4
4
R 3 a 3
3
3
a3
= 3
b
3
R a

4.

= 0.74 m 2
A x + A2 x2
= 1 1
A1 + A2
A1 = a2
A2 =

171

a2
4

a
2

By symmetry yCM = 0
a

5. In Fig. 1, C is CM.

a2
( a)
4
=
2
a
a2 +
4

=
a
4+
a2 0 +

Fig. 2

m1

m2

(a4 , b4 )
X

m2

C'

(i)

m1a = m2 ( l a)
In Fig. 2, C is CM.
m1( a + b l1) = m2 ( l a + l2 b) (ii)
Substituting Eq. (i) in Eq. (ii),
m1( b l1) = m2 ( l2 b)
or
m1b m1l1 = m2 l2 m2 b
or
( m1 + m2 ) b = m1l1 + m2 l2
or

b=

A a
A 0 +
4 4
xCM =
A
A +
4
A 4 a
=

4 3A 4
a
=
12
A b
A 0 +
4 4
yCM =
A
A +
4
b
=
12
b
a
Centre of mass
,
12
12

l2

b
m1

l1

3. Let A = area of rectangle

Fig. 1

6. xCM =

m1l1 + m2 l2
m1 + m2

x dm
dm
dx

where, dm = mass of element of length dx.


x dx A
=
dx A
x2
x 0 2 dx
l
=
l
x2
0 0 l2 ax
l

x 3dx

x2 dx

(Q = 0

x2
)
l2

172 | Mechanics-1
l

7. xCM

3 x4
= 3
4 0
x
3
= l
4
1 10 + 2 12
=
1+2

New position of CM

10 m 10 m

6ms1 1 kg 2 kg 4ms1

+ ive

34
m
3
1 ( 6) + 2 ( + 4)
2
=
= + ms 1
1+2
3

=
vCM

1 kg
2 ms

1 ms

70 ^i + 105 ^j
m
3
0.10 kg

m = 0.3 kg
(b) Momentum of system = Momentum of CM
= ( m + 0. 1) kg 6 ^j ms 1
= 2.4 ^j kg ms 1

v=u+at
(1)(0) + (2)(10$i + 10 $j)
=
10 $j
3
20
10 $
= $i
j m/s
3
3

New position vector ( r1) of particle A

1
( r1) = s 0 + u t + a t2
2

1
= (10 ^i + 20 ^j) + 0 + ( 10 ^j) 12
2
= 10 ^i + 15 ^j m
New position vector ( r2 ) of particle B

1
r2 = (20 ^i + 40 ^j) + (10 ^i + 10 ^j) 1 + ( 10 ^j) 12
2
^
^
= 30 i + 45 j

v (0.3) + 0 (0.10)
(c) v CM = m
0.3 + 0.1

4
v m = v CM
4
4
= 6 j^ ms 1
3

+ ive

m 0 + 0 1 12
m + 0. 1

(a) 3 =

9. Acceleration ( a ) = 10 j ms 2

10 ^i + 15 ^j + 60 ^i + 90 ^j
3

12 m

= 0 ms 2
Displacement of CM in 1 s = 0 m.

6 ^j ms1
m kg
3 m CM
O

2 kg
1

r1 (1) + r2 (2)
1+2

10.

x CM (new position of CM)


34
2
=
m + 2 s ms 1
3

3
38
m = 12.67 m
=
3
1 2 ( + 2) + 2 ( 1)
8. vCM =
1+2
1

R=

= 8 ^j m s 1
m

11.

A t = 0s
2m

B ( t = 100 ms)
Position of 1st particle ( A) at t = 300 ms
1
s1 = 10 (300 103)2
2
= 0.45 m
Position of 2nd particle ( B) at t = 300 ms
(B is at the position of A at t = 100 ms)
1
s2 = 100 (200 103)2
2
= 0.20 m
2 m 0.2 + m 0.45
Position of CM =
2m + m
= 28.3 cm

Centre of Mass, Conservation of Linear Momentum, Impulse and Collision |


Velocity of 1st particle ( A) at t = 300 ms
v1 = 10 300 103
= 3 ms 1
Velocity of 2nd particle at t = 300 ms
v2 = 10 200 103 ms 1
= 2 ms 1
2m 2 + m 3

vCM =
2m + m
7
= = 2.33 ms 1
3
m A 0 + mB 80
12. 24 =
m A + mB
or
24 ( m A + 0.6) = 80 0.6
or
m A = 1.4 kg
Total mass of system = 1.4 kg + 0.6 kg
= 2.0 kg

14. Let x = displacement of wedge (30 kg)

towards right.
Displacement of block A towards right
(along x-axis) when it arrives at the
bottom of the wedge
A

30

5 kg

B (50 kg)
R

= QR x
= 0.5 x
Now, as net force on the system (wedge +
block) along x-axis is zero, the position of
CM of the system, along x-axis, will not
change

vCM = 6.0 t2 ^j

aCM = 12 t ^j ms 2

Net force acting on system (at t = 3 s)


= Total mass of system (aCOM at t = 3 s)
= 2.0 kg 36 ^j ms 2

5 (0.5 x) = 30x
0.5
m
x=
7
= 71.4 mm

i.e.,

= 72 N j
13.
L
O

173

15. As no external force acts on the system,

dx

xCM =

x dm
dm
L
0 x ( Ax dx)
L

o
L

x2 dx
x dx

B
FA

i.e.,

Ax dx

L3 / 3
L2 / 2
2
= L
3

the velocity of CM will be zero.

i.e.,

FB
B

m Av A + mBvB
=0
m A + mB
vA
m
2
= B =
vB
mA
1

(a) Ratio of speeds = 2


p
m v
m m
(b) A = A A = A B = 1
pB
mBvB
mB m A
(c)

p2 / 2 m A
KA
p2
m
2
= 2A
= 2A B =
KB
pB / 2 mB
pB m A 1

174 | Mechanics-1
16. While man travels from P to Q

or

Negative sign shows that the balloon will


move downwards.

CM
P

vCM

m
v
( M + m)

V =

3 v

m
v + mv
3v
2

=
=
m+m
4

L
.
2
Time taken by man to reach point Q
starting from point P
L /2 2L
tPQ =
=
3v/ 4 3v

+ ive

Displacement of CM (along horizontal) =

While man travels from Q to P


3v
m
v + mv
2

vCM =
m+m
3v
=
4
1
Displacement of CM =
2
Time taken by man to reach point P
starting from point Q
L / 2 2 L
t QP =
=
3v / 4 3 v
Total time = tPQ + t QP
2L 2L
+
3v 3v
4L
=
3v
=

4L
v
3v
4L
=
3

Net displacement of trolley =

17. (a) While the man climbs in the rope, no

extra external net force acts on the system


(balloon along with rope + man). Force
applied by man to gain velocity to climb
up is an internal force and as such the
velocity of the CM of the system will
remain stationary.
mv + ( M + m) V = 0

(b) If the man slops climbing i.e., v = 0,


then according to the above relation
the value of v will also be zero. Thus,
the balloon will also slop.
18.
m

vi

v2

v3

(i)

mv1 = Mv1 + mv3


m = 4 103 kg
M = 1 kg
v3 = 100 m/s
v1 = 500 m/s
Find v2
2
(a) 0 = v2 2as = v22 2(g) s
v2

= 2
2 gs

Here
s = 0.30 m
(b) Decrease in kinetic energy of bullet
1
= m(v12 v23 )
2
1
(c) KE of block = mv22
2
19. m

v1

m1

v2

= 0.5

m2

v3

( m + m1)v2 = mv1

Centre of Mass, Conservation of Linear Momentum, Impulse and Collision |


mv1

v2 =
m + m1
mv1
Common velocity v3 =
m + m1 + m2

(i)
(ii)

V2 =

or

2 m2 g l (1 cos )
M ( M + m)

2 m2 g l 2 sin2
2
2
V =
M ( M + m)

or
or

20. Let v = Velocity of pendulum bob when the

pendulum is vertical

PE = mgh
h

KE = 1 mv
2
v

V = 2 m sin

gl
M ( M + m)

21. Let the track shift by x (to the right) when


the cylinder reaches the bottom of the
track.

x + x = R r
or
x = ( R r) x
(x = horizontal displacement of cylinder
w.r.t. ground)
As no force would be acting along
horizontal direction, for no shift in CM
along horizontal. We would have
mx Mx = 0
or
m [( R r) x ] Mx = 0
or
( M + m) x = m ( R r)
m ( R r)

x=
M+m
Now, as the PE of the cylinder would
change into the kinetic energies of the

KE = 1
2

(R r)
m

V = Velocity of wagon when pendulum is


vertical

mv = MV
M
or
v=
V
m
KE of wagon + KE of bob = PE of bob
1
1
MV 2 + mv2 = mgh
2
2
2
1
1 M
or
MV 2 + m
V = mgh
2
2 m
1
M

or
MV 2 1 +
= mgh
2
m

1
or
MV 2 ( M + m) = m2 gh
2

2 m gh
M ( M + m)

V2 =

or

For m1 : v23 = v22 2a1s1 and Find s1


For m2 : v23 = 2a2 s2 and Find s2
m1 g
Here : a1 =
= g
m1
m1 g
and
a2 =
m2

175

x'

cylinder and the track we have,


1
1
(i)
mg ( R r) = mv2 + MV 2
2
2
where, v = velocity of cylinder
and V = velocity of track.
(when cylinder just reaches bottom of the
track)

176 | Mechanics-1
Applying
law
of
conservation
of
momentum
mv + MV = 0
M
or
v=
V
m
Thus, Eq. (i) becomes
2
1 M
1
mg ( R r) = m
V + MV 2
2 m
2
1
M

or
mg( R r) = MV 2
+ 1
2
m

2
2
m
g
(
R

r
)
or
V2 =
M ( M + m)
or

V =m

2 g ( R r)
M ( M + m)

22. No change in momentum of ball and also

that of wall along horizontal direction.


2 m/s

2 m/s

2 m/s

45

+ ive

2 m/s

2 m/s

50 g

45 45

2 m/s

Wall

Along perpendicular direction :


Momentum of ball after reflection

p f = 50 103 2 kg ms 1

|p f |= 50 103 2 kg ms 1

Change in the magnitude of the

momentum of the wall = |p f | |p i |


=0
23. m0 = 40 kg

m = ( 40 + 160) kg
= 200 kg
vi = 2 km/s = 2 103 kg 1
Rate of consumption of fuel = 4 kgs 1
Time (t) required to completely brunt out
160 kg
of the fuel =
= 40 s.
4 kgs 1
Ultimate vertical speed gained by the
rocket
m
= g t + vi ln
m0
200
= 10 40 + 2 103 ln
40
= 400 + 3218 = 2818 m/s
= 2.82 kms 1
24. When y length of rope has fallen on table

top
A
y

( p f ) = 50 103 2 kg ms 1
Momentum of ball before reflection

A v = 2gy

( p i ) = + 50 103 2 kg ms 1

p = p f p i

= 100 103 2 kg ms 1
= 0.14 kg ms 1
i.e.,

|p|= 0.14 kg ms 1

Momentum of wall (when ball strikes


wall)

p i = 50 103 2 kg ms 1

|p i|= 50 103 2 kg ms 1

Momentum of wall (when ball rebounds


from the wall)

dm
dt
dm dy
=v

dy dt
dm
M
= v2
= v2
dy
L
M
W =
g
L
Fnet = W + Fth
Fth = vrel

Centre of Mass, Conservation of Linear Momentum, Impulse and Collision |


M
M
Fnet =
yg + v2
L
L
M
M
=
yg + 2 gy
L
L
M
=3
yg
L
= Weight due to 3 y length of the rope.
25. Horizontal velocity of incoming sand
y

Fth

v = 2 ms1

v=0

Horizontal velocity of the conveyer belt

= ( v) (say) = v i

u rel = v1 v2

= 0 (v i)
= v ^i

dm
dt
dm ^
= v
i

dt

Fth = v rel

dm
> 0, the falling sand particles exert
dt
thrust force which decelerates the
conveyer belt.
As

Force required to keep the belt moving

F = F th
dm ^
=v
i
dt
= (2 ms 1 5 kgs 1) ^i
= 10N ^i
Power delivered by motor to drive belt at
2 m/s

= |F||v|= 10 N 2 ms 1
= 20 W

26.

177

ma = Fth mg
or
ma = Fth
(neglecting mg as compare to Fth )
dm
or
ma = u

dt
dm
a
or
= dt
m
u
dm
a
or
m = u dt
a
or
log e m = t + K
u
Now, as t = 0, m = m0
(given)
log e m0 = K
a
Thus,
log e m = t + log e m0
u
m
a
or
log e
= t
m0
u
or

m = m0 e

a
t
u

27. u = 100 ms 1, v = 0 ms 1, s = 6 cm

v2 = u2 + 2as
u2

a=
2s
(a) Now, v = u + at
u
u
2s

t= = 2 =
a
u
u
2s
2 6 102
=
= 1.2 ms
100
(b) Impulse on log = Change in
momentum of bullet
= m (0 u)
= mu
= 5 103 100
= 0.5 Ns
(c) Average force experienced by the log
Impulse
0.5
=
=
time
1.2 103
= 416.67 N
28. Let us consider right direction as positive.

Impulse of the spring on the block


= Change in momentum of block
= m (v u)

178 | Mechanics-1
32. m1 = 1 kg and m2 = 1 kg

= 3 [( + 40) ( 50)]
= + 270 Ns

v2 = + 4ms1 v1 = 6ms1

(to the right)


Impulse
Average force on block =
t
270
=
0.02
= 270 Ns

m2 1 kg

A
B
Before collision

(to the right)

= 13.5 kN

dp
= 4 ^i
dt

or dI = 4 ^i dt

or

(where, I = Impulse)

dI =

4 ^i dt

I = 4 ^i (2 0)

or

30. m v = F t

or
or

m v = Area under F-t graph


16 + 8
m v =
20000
2
m v = 240000
240000
v =
= 200 m/s
1200

33. v1 =

v u = 200

v2

v = 200 ms 1 as u = 0

31. v1 = v2 = 2 ms 1

v2 = v1 = + 3 ms
v2

v1

m2

m1

+ ive

B
After collision

1 2
2 2
=
( +4) +
( 6)
2 + 1
2 + 1
4
= 8
3
28
28
=
ms 1 =
ms 1
3
3
(in ive x direction).

= 8 ^i kg-ms 1
or

v1'

2 1
2 1
=
( 6) +
( 4)
2
+
1

2 + 1
8
= 2+
3
2
= + ms 1
3
2
= ms 1 (in + ive x direction)
3
m m1
2m1
v2 = 2
v2 +
v1
m
+
m
1
m1 + m2
2

p = m v = 2 (2t i 4 j)

29.

v2'

m m2
2m2
v1 = 1
v1 +
v2
m1 + m2
m1 + m2

= 13500 N

m1 2 kg

(given)

1+ e
1 e
v2 and v2 =
v2
2
2
v2'

v1 = 0
m 2

v1' = 2v2'
m

Rest

v2' = 2ms1 v1' = + 3ms1

+ ive

(When two bodies of equal masses collide


elastically
they
interchange
their
velocities)

or
or
or

v1 1 + e
=
v2 1 e
1+ e
2=
1 e
2 2e = 1 + e
1
e=
3

(Qv1 = 2 v2 )

Centre of Mass, Conservation of Linear Momentum, Impulse and Collision |


34. mv = mv1 + mv2
v
m

Rest
Before collision

Also,

or
or
or
or

or

or
or

or

v1

v2

v22 + v12 2 v1v2


v2
(v + v2 )2 4 v1v2
e2 = 1
v2
v2
v2
2
e2 =
v2

e2 =

1
2
1
or
e=
2
2m2
35. v1 =
v2
m1 + m2
or

After collision

v = v1 + v2
1
1
mv12 + mv22
3
2
2
=
1
2
4
mv
2
3
(v12 + v22 ) = v2
4
3
2
(v1 + v2 ) 2v1v2 = v2
4
3 2
2
v 2 v1v2 = v
4
1 2
2 v1v2 = v
4
Velocity of separation
e=
Velocity of approach
v2 v1
e=
v0
v2 v1
e=
v
2
v v1
e2 = 2

179

v2 = 2 ms1

e2 =

v1 = 0 ms1

v2'

v1'

A
m2 = 3 kg

m1 = 2 kg

(i)

23
2
2+3

12
= 2.4 ms 1
5
m m1
v2 = 2
v2
m1 + m2
=

and

32
2
2+3

2
= 0.4 ms 1
5

Distance between blocks when they stop


sliding
v 21
v 22
=

2 k g 2 k g
=

1
(v 21 v 22 )
2 k g

1
[(2.4)2 (0.4)2 ]
2 0.3 10

2.8
= 0.933 m
3

180 | Mechanics-1
Objective Questions (Level 1)
Single Correct Option
1. Momentum remains conserved. Decrease

in momentum of the ball is transferred to


sand while KE does not remain conserved
as it gets used up in doing work against
friction.
net
2. Fext
= M aCM
net
If Fext
= 0,

aCM = 0
d
i.e.,
v
=0
dt CM
or
vCM = constant
Option (a) is correct.
3. The forces acting on the blocks would be
equal and opposite as per Newton's 3rd
law of motion. Acceleration of the blocks
will depend upon their masses as per
Newtons 2nd law of motion. Accelerations
being different velocities will be unequal.
M1

M2

Option (c) is correct.


4. While colliding the balls will apply equal

and opposite impulsive force on each


other. Impulsive forces will change the
momentum of the balls but the total
momentum of the system of 2 balls will
remain conserved impulsive forces being
internal ones. Change in momentum of
the system will definitely be due to
external gravitational forces on the balls
but as the time of impact shall be very less
the impulsive force will over shadow the
weak gravitational force.
5. External force acting on the cannon shell

before explosion is the gravitational force.


Now, as no extra net external force would
be act on the shell during collision the
momentum of the system shall remain
conserved and the CM of the system (now
broken in pieces) will also keep on
following the path which the shell would
have followed had the explosion not taken
place. Further, as the explosion would
never be super-elastic, the KE of the
system cant increase after explosion.
Option (d) is correct.

6.

Velocity of separation
=e
Velocity of approach

As in an elastic collision e < 1


Velocity of separation < velocity of
approach
(when e = 0, the velocity of separation in
zero and the colliding bodies do not
separate from each other.)
Further, whether the collision in elastic or
inelastic the law of conservation of
momentum always hold gord.
Option (d) is correct.

7. p = M v CM , Option (a) is correct.

p = p1 + p2 + p3 + ....,

Option (b) is correct.


Further, we define momentum for every
type of motion.
Option (d) is correct.
8. Let us consider a system of two masses as

shown in figure.

v1

vCM

m1

CM

v2
m2

Momentum of system about CM

= m1 ( v1 v CM ) + m2 ( v2 v CM )

= m1 v1 + m2 v2 ( m1 + m2 ) v CM

= ( m1 + m2 ) v CM ( m1 + m2 ) v CM

=0

Option (c) is correct.


9. Option (a) If collision is inelastic.
A

v
m
B

v
C

Option (b) If collision is perfectly inelastic


Option (c) If the dimensions of the
particles 0
Option (d) would be the answer.

Centre of Mass, Conservation of Linear Momentum, Impulse and Collision |


10.

14. Horizontal velocity of the leaving coal :

F t = m v
m v
F=
t
5 (65 15)
=
2
= 125 N

system

v1 = + v ^i

d
C

Horizontal velocity of the system

mo
O

CM
d1

v2

mo
d
mo + mc
8
=
d
8+6
4
= d
7
4
= (1.2 1010 m)
7
= 0.64 1010 m

d1 =

Rest

Mm

9 kg

3 kg

6 kg

second
n [change in momentum per second] F
40
n
(1200 0) 144
100

144
or
or n 3
n
48
Option (a) is correct.
i.e.,

m
v
( m m)
3
=
16
93

16. Change in momentum along x-axis


v

v sin

P
+

= 8 ms 1
1
KE of 6 kg mass = 6 ( 8)2
2
= 192 J
2 m2
13. v1 =
v2
m1 + m2
v1 = 0

v2'

v1'

m2

m1

m2

m1

After collision

(as m1 < < m2 )

v cos
y

v2

= 2 v2
Option (b) is correct.

15. If n be the number of bullet shots per

V =

Before collision

= + v ^i

U rel = v1 v2 = 01

dm

Fth = U rel
=0
dt
As, the leaving coal does not exert any
thrust force on the wagon, the speed of the
wagon wont change.
Option (a) is correct.

12. M 0 = mv + ( M m) V

^i

11. mc d1 = mo ( d d1)
mc

181

v cos
x

v cos

= m (v cos v cos ) = 0
Net change in momentum
= Change in momentum along y-axis
= m[( + v sin ) ( v sin )]
= 2 mv sin
(as = 45)
= mv 2
Option (a) is correct.
17. Velocity of ball before first impact i.e.,

when it reaches point Q of the horizontal


plane

182 | Mechanics-1
50 kg

P u=0
450 kg
O
50 kg

+ eg
Q

1st

450 kg

2nd

v = 0 + ( g) 1 = g
Velocity of ball after 1st impact
= ev = eg
time elapsed between 1st and 2nd impact
with the horizontal plane
( + e g) = ( + e g) + ( g) t

t = 2e
4
2
(as, e = L)
= s
3
3
Option (c) is correct.

x dm = x m dx
dm m dx

CM (boat)

50 10 + 450 5 50 ( x) + 450 ( x + 5)
=
50 + 450
50 + 450
(Initially)

x = 1m
Option (b) is correct.
20. As discussed in the answer to previous

question no. 19.


M
M/3

Smooth

Smooth

(where m = mass per unit length)


Ax2
x L dx
Ax2
=
Q m =

2
L
Ax

L dx
L

x 3dx

x2 dx

M
3

(Initially)

(Finally)

x M +
= ML
3

3
x= L
4

i.e., the distance that plank moves =

L4
3
3
4
L
3
= L
4
Option (a) is correct.

The distance that the man moves


3
=L L
4
L
=
4
Option (b) is correct.

19. As there is no net external force acting on

the system in the horizontal direction, the


CM of the system shall not shift along
x-axis.

L
M L

x + ML +

2
3 2
=
M
M
M+
M+
3
3

Mx +

M/3
L

(Finally)

dx
0

eg

18. xCM =

CM (boat)
10 m

21.
v

Rest

Rest

3m

Before collision

3m

After collision

mv + 3 m 0 = m 0 + 3 mu

3
L
4

Centre of Mass, Conservation of Linear Momentum, Impulse and Collision |


v

u=
3
Velocity of separation
e=
Velocity of approach
u0 u
=
=
v0 v
1
=
3
Option (d) is correct.

Impulse on ball
= Change in momentum of ball
9
4
=
mv0 + mv0
20
5

5
= mv0
4
25. If a ball dropped from height h rebounds

to a height h, then speed of ball just


before 1st impact, u = 2 gh
Just after 1st impact u = 2 gh

22. Change in momentum of A or B = mu


u
m

Rest

Rest

Before collision

Option (b) is correct.


F'

F'

m
B

Option (c) is correct.

h = e2 h

Height attained after nth impact


= e2 n h
= (0.8)2 n 1

(as h = 1 m)

2n

= (0.8)

or

475 v = 500 ( ^i k)

20 ^ ^
( i k)
19
Option (c) is correct.
or

v=

27. While force is increasing with time

F = kt (where k is + ive constant)


or
X

53
Y

i.e.,

500 1 ^i = (500 25) v + 25 (20 k)

v0 cos 37 = v0 4
5
v0

3 v cos 53
4 0
= 9 v0
20
3v
4 0

= 0.8

before
throwing block
= Momentum of car after throwing block
+ Momentum of block

F
m

F = ma0
Acceleration of block B :
F F
aB =
m
F ma0 F
=
=
a0
m
m

37

h
64
=
h
100

26. Momentum of car (+ block)

a0 =

24.

u
=
u

Option (d) is correct.

23. Acceleration of block A


A

e=

After collision

(As collision is elastic)


Impulse = Change in momentum
Ft0
= mu
2
2 mu

F=
t0

183

37
4

53

or

or

ma = kt
dv k
=
t
dt m
k t2
v=
+C
m 2
k t2
v=

m 2
(If at

t = 0, v = 0)

184 | Mechanics-1
Thus, graph between v and t would be

30. As no external force is acting along the

horizontal direction on the system (wedge


+ block). The CM of the system shall not
change along horizontal when the block
moves over the wedge but would change
along vertical as net force (= gravitational
force) is acting on the block.
Further, as no non-conservative force is
acting on the system, its total energy will
not change.
Option (d) is correct.

v0
Graph would be
parabolic in
nature

t0

While force is decreasing with time.


F = kt (where k is + ive constant)
k t2

v=
+ C
m 2
At t = 0, v = v0
k t20

C = v0 +
m 2
k 2
Thus,
v = v0 +
( t0 t2 )
2m
Thus, graph between v and t would be

Parabola

v
v0

31.

v'
m

M
v

Rest
Before collision

Just after
collision

or 8 R2 x1 = 2R2 R or 8 x1 = 2 R
R
or
x1 =
4
Option (c) is correct.
A x + A2 x2 + A3x3
29. xCM = 1 1
A1 + A2 + A3
0 = [ ( 4 R)2 R2 R2 ] x1
+ [ R2 ] 3 R + [ R2 ] 0
or
14 x1 = 3 R
3
or
x1 =
R
14
Option (d) is correct.

Final position

mv = ( M + m) v
From final position, v = 2 gh
From Eq. (i),
m
or
v = 2 gh
M+m

(i)

v = 1 +
2 gh

Option (a) is correct.


A x + A2 x2
28. xCM = 1 1
A1 + A2
[ (3 R)2 R2 ] x1 + [ R2 ] 2R
0=
(3 R)2 R2

Rest
h

or
t0

Option (c) is correct.


32. As no net extra external force is acting on

the system the CM of the gun and the


bullet system remains at rest. The force
exerted by the trigger of the gun on the
bullet is an interval one.
33. m1 g T = m1a

T
T

a
m2g

m1g

Centre of Mass, Conservation of Linear Momentum, Impulse and Collision |


T m2 g = m2 a
( m1 m2 ) g = ( m1 + m2 ) a
m m2
or
a= 1
g
m1 + m2
m a + m2 ( a)
aCM = 1
m1 + m2
m1 m2
=
a
m1 + m2
2

m m2
= 1
g
m1 + m2
Option (b) is correct.
m
34. v = u gt + vi ln 0
m
m0
m
(neglecting gravity as given)
m
or
v = v2 ln 0 (Taking u = 0)
m
m0
or
(as vi = v)
v = v ln
m
m

log e 0 = 1
m
m0
1

= e = 2.718
m
Option (a) is correct.
or

v = u + v2 ln

36. Velocity of separation would be zero as the

collision is completely inelastic.


A

v cos
2
v

v sin
2

v sin
2

/2

2m

/2
v
v cos
2

Velocity of approach = Velocity of A w.r.t.


B or velocity of B w.r.t. A

= + v sin v sin

2
2

= 2 v sin
2
Common velocity ( V ) after collision

mv cos + mv cos = 2 mV
2
2

V = v cos
2
Option (d) is correct.
37. v sin 53 = u sin 37

35. Velocity of water after striking the plate

would be almost zero as then it flows


parallel to the plate.

185

u sin 37
u cos 37

v cos 53

v
v sin 53

53

37

Floor

Jet of water
Water flow

Force exerted on plate


= [Rate of change of momentum of
water]
m 0 mu
=

mu m
=
=
u
t
t
kg
= 0.5
1ms 1
s
= 0.5 N
Option (c) is correct.

4
3
=u
5
5
3
i.e.,
v= u
4
Impulse exerted on floor
= [Change in momentum of ball]
= [( mv cos 53 ) ( + mu cos 37 )]
= m [v cos 53 + u cos 37 ]
v

3
3
4
= m u +u
5
5
4
5
= mu
4
5
= 1 5 = 6.25 Ns
4

186 | Mechanics-1
x = Displacement of boat
x 40 + (1 + x) 15 = 2 40 + 1 15
i.e.,
x = 1.46 m
(The frictional forces on the boat by the
boy and that by the boy on the boat are
internal forces).

38. As the CM will not change along x-axis,

for no net force acting on system (wedge +


block) along x-axis.
m

M
x'

a = cot
h

41.

Mg
M

m
X
Mg
a

mx + M ( x + x ) = Ma + Mx
m
mx + Mx = ma or x =
a
M+m
m h cot
=
M+m

2 cm

D
M

2 cm

2 cm
2 cm

M
A

M
falls very close to
5
mortar, its velocity after explosion must
be u.
As piece of mass

Shell A
u (say)
M
or
ta
r

Y
M

v=

42. The shell explodes at A, the highest point.

Option (b) is correct.


2(4M ) + 3 ( M ) + 3 (+ M )
39. xCM =
= 2 cm
4M + ( M ) + (+ M )
Y

m v1 + m v2 + m v 3
m+m+m

1
= [ v1 + v2 + v 3 ]
3
1
^
= [v0 ^i + ( 3v0 ^j ) + (5v0 k)]
3
v
^
= 0 [ ^i 3 ^j + 5 k ]
3
Option (d) is correct.

R/2
After explosion
path of M/5 part

2M
D+C

R/2

Shell
4M
5

(Velocity of 4M/5 man part


of shell after explosion)
3/2 u
After explosion path of
4M/5 part

B
R/2

2 cm

2 cm

2 cm

2 cm

3R/4

yCM

2(4M ) + 1( M ) + 3 (+ M )
=
= 2.5 cms
4M + ( M ) + (+ M )

Option (c) is correct.


40. As explained in the answer to question no.

20 and 19.

Thus, Mu =
i.e.,

M
4M
( u) +
(v)
5
5
3
v= u
2

As the velocity has increased


40 kg
15 kg
2m
15 kg

2m

range of this part will be


3
R
3R
of i.e.,
2
2
4
R 3 R 5R

D=
+
=
2
4
4
Option (c) is correct.

3
times the
2

Centre of Mass, Conservation of Linear Momentum, Impulse and Collision |


m m1
43. v2 = 2
v2
m2 + m1
v2

v1 = 0

A m

1
1
mv22 m v2

2
3
2

=
1
2
mv2
2
1 1

= 2 18
1
2
8
=
9

v2'
m

B 2m

m2 = m

187

2m

m1 = 2m

m 2m
v2
m + 2m

1
v2
3

Fraction of KE lost by colliding particle A


KE lost
=
Initial KE

JEE Corner
Assertion and Reason
1. To answer this question, let us find the

centre of mass of an L-shaped rigid body


(as shown in figure) of uniform thickness.

The centre of mass of a uniform plate lies


at its centre. The CM of a uniform sphere
is at its centre.

Y
L

4L A2

xCM

L
P

A1

2L
O

2L

4L

A x + A2 x2
= 1 1
A1 + A2
=

(6L 2L) 3 L + ( 4 L 2L) L


(6L 2L) + ( 4 L 2L)

36L3 + 8 L3
12L2 + 8 L2

44 L3
20L2

These examples show that assertion is


false.
It is correct that centre of mass and centre
of gravity of a body coincide if the body is
placed in a uniform gravitational field.
Option (d) is correct.
F F1
F
2. a1 = 1 , a2 =
2m
m

Thus, we see that CM is lying outside.


Centre of mass of the rigid body of
uniform thickness as shown in figure
would be at point P which is neither
outside nor inside.

a1

a2

2m
F1

= 2.2 L
Similarly yCM = 2.2 L

ma1 + 2ma2
m + 2m
F1 + ( F F1)
=
3m
F
=
3m

aCM =

188 | Mechanics-1
As F is constant, the value of aCM will
remain constant (Reason).
As CM is accelerated the velocity of CM
will obviously increase (Assertion).
Option (a) is correct.
3. As per assertion if force is applied on a

system the linear momentum of the


system must not remain conserved. But, it
will not be true if we apply two equal and
opposite external forces on the system as
then net external force on the system will
be zero and the linear momentum will
remain conserved as given under Reason
which is correct.
Option (d) is correct.
4. A rocket moves forward due to the thrust

force produced on it as per Newtons 3rd


law of motion (as given under Reason)
when gas inside (not the surrounding air
as given under assertion) it is pushed
backwards.
Option (d) is correct.
5. Linear momentum of a system remains

conserved when no net external force acts


on the system i.e., only a net external force
on a system can change its linear
momentum. Inside a system internal
forces are always in pairs and as such
can't change linear momentum.
If two blocks connected by a spring placed
on a smooth surface are stretched apart
the internal restoring forces acting on
blocks will definitely increase the KE of
the system but this argument under
Reason has nothing to do with the nothing
under Assertion that internal forces can't
change linear momentum.
Option (b) is correct.
p2
6. KE =
2m
1
i.e., KE
(if momentum p is constant).
m
Nothing under reason is correct.
When bullet is fired and comes out of the
gun.
Linear momentum of gun
= Linear momentum of bullet = p

(In magnitude)
(KE) gun
Mass of bullet

=
(KE) bullet
Mass of gun
(as given under Reason)
This is what is given in Assertion.
Thus, both Assertion and Reason are true
and also reason is the correct explanation
of the assertion.
Option (a) is correct.
7. As no net external force is acting there on

the system along horizontal direction the


momentum of the system remains
conserved along horizontal direction but
as gravitational force (a net external force)
acts on block in the vertical direction
(downwards) the momentum of the system
does not remain constant along vertical
direction. As overall momentum of the
system does not remain constant,
Assertion is true.
As wedge is at rest, Reason is false.
Option (c) is correct.
8. In any collision, there is not change in the

momentum of the system as given in


reason, which is true.

i.e.,

p1 + p2 = 0

p1 = p2

(i)

Assertion is false.
Option (d) is correct.
9. Reason is true as explained in the answer

to question no. 6 and also in Eq. (i) in the


answer to question no. 8.
As KE is inversely proportional to mass,
K
the KE of the block of man 2m will be
2
when the KE of the block of mass m is K .
Assertion is true.
Further, as Reason is the correct
explanation of the Assertion. Option
would be (a).
10. Assertion is false as for example heat
energy can be given to a system without
any increase in momentum of the system
while KE given to a system increases its
momentum.

Centre of Mass, Conservation of Linear Momentum, Impulse and Collision |


2

p
KE =
as given is Reason which is
2m

true.
Option (d) is correct.
11. As no external force would be acting on

the system of electron and proton along


the line joining electron and proton the
CM of electron and proton will remain at
rest. Therefore, Assertion is false.
Further, as proton is heavier than
electron the reason is true.
Option (d) is correct.
m m2
2m2
12. v1 = 1
v1 +
v2
m1 + m2
m1 + m2
v2
m2

v1

m1
A
B
Before collision

and v2 =

v2'

v1'

m2

m1
A
B
After collision

m2 m1
2m1
v2 +
v1
m1 + m2
m1 + m2

v2 v1
m m1 2m2
2m1 m1 + m2
= 2
v2 +
v1
m1 + m2
m1 + m2
= v2 + v1
= (v2 v1)
i.e., v 21 = v21
i.e., relative velocity of A w.r.t. B after
collision
= (relative velocity of A w.r.t. B before
collision)
Reason is true and Assertion is false.
Option (d) is correct.
13. As explained in the answer to question no.

11, the CM of the objects will remain at


rest. Therefore, assertion is false.

x1

189

x2

m1

CM

m2

m1x1 = m2 x2
x2 m1
=
x1 m2

x2 > x1 as m2 > m1
Reason is true.
Option (d) is correct.

14. F =

dp
dt

(Newton's second law of motion)

a=

F
m

(outcome of the above)

Reason is true.

First equation tells that if same force F is


applied on different masses the rate of

p
change of momentum i.e., d of each mass
dt
will be same. Second equation tells that if

same force F is applied on different

masses the a produced in each will be


different.
Assertion is true.
Further, as Reason is the correct
explanation of the Assertion.
Option (a) is correct.
15. Assertion is false as explained in the

answer to question no. 12.


In every type of collision the linear
momentum of the system remains
conserved. Therefore, Reason is true.
Option (d) is correct.

190 | Mechanics-1
Objective Questions (Level 2)
Single Correct Option
1. m1v1 + m 0 = m 5 g l + m1

v1
3

= cos 1

or

3
4

Option (c) is correct.

3. Here |v2 |= |v1|= v say

dp

v2

v1

dm

m1

m
Rest

v1

u = 5gl

v1/3

2 v1
= m 5g l
3
3 m
v1 =
5g l
2 m1

Net momentum of the two elements as

shown in figure = dp = |dp|

2m
H
m
v

u
2
2
0 = v2 + 2 ( g) H
v2
H=
2g
v=

or
or
or

= 2v dm sin
M
= 2v
( R d) sin
R
2M v
=
sin d

/2 2Mv
p=
sin d
0

2M v
=
[cos ] 0 / 2

2M v
2Mv
=
[0 1] =

Option (b) is correct.


1
1
4. (2 m) v2 = kx20
2
2

(u / 2)2
2g

u2
8g
2g l
H=
8g
=

l
l (1 cos ) =
4
1
1 cos =
4

v1
dl

2. mu = 2mv

u = 2gl Rest

Option (b) is correct.

m1

dm

kx0

kx0

or
or
or
or

v=

k
x0
2m

2mv = 2 mk x0
Fav t = 2 mk x0
Fav =

Option (b) is correct.

2 mk
t

x0

Centre of Mass, Conservation of Linear Momentum, Impulse and Collision |

4 p = 1(1 p)
1

p= m
5
Displacement ( x) of bar when pendulum
becomes vertical
x
= sin
p

5. As the collisions of the striker and the

walls of the carrom are perfectly elastic the


striker will follow the path OPQROP ...
Q

45

x = p sin
1
1
m = 0.1 m
= sin 30=
5
10
When the ball reaches the other extreme
end the bar will further shift to the left by
distance x and as such the net
displacement of the bar will be 2x i.e.,
0.2 m.
Option (b) is correct.

45
A

Change in KE = work done against friction


1
mv2 0 = mgs
2
(m = mass of striker, s = displacement of
the striker)
v2

s=
2 g
(2)2
= 1m
=
2 0.2 10
1
1
PQ = OP = OA 2 =
2= m
2 2
2
1 1
OP + PQ = + = 1 m
2 2
Striker will stop at point Q where
1
1
co-ordinates are
,
.
2 2 2
Option (a) is correct.
6. As no force is acting on the system along

horizontal, the CM of the system will not


shift horizontally.
4 kg
p

7.
p

ep

Floor During
first
collision

ep

e2p

During
Second
collision

e 2p

e 3p

During
third
collision

Floor

Momentum imparted to the floor in


1st collision = p ( ep) = p (1 + e)
2nd collision = ep ( e2 p) = ep (1 + e)
3rd collision = e2 p ( e3 p) = e2 p (1 + e)
As theoretically there will be infinite
collision, total momentum imparted to
floor
= p (1 + e) + ep (1 + e) + e2 p (1 + e) +....
= p (1 + e) [1 + e + e2 + K ]
1 + e
1
= p
= p (1 + e)

1 e
1 e
Option (d) is correct.
8. Let F be the frictional force applied by

CM

191

1 kg
x

x
4 kg

CM

plate when bullet enters into it


1
(i)

mu2 = Fh
2
If plate was free to move
mu + 0 = ( M + m) u
m

u =
u
M+m
1
1
New KE of bullet = mu2 ( M + m) u2
2
2
(entering plate)

192 | Mechanics-1
2

1
1
mu2 ( M + m)
u
2
2
M + m

1
m 1
M
2
= mu2 1

= mu
2
M
+
m
2
M
+ m

(ii)
= Fh
Dividing Eq. (ii) by Eq. (i),
h
M
=
h M+m
M
i.e.,
h =
h
M + m

3 m1 = m2
m1 1
=
m2 3

or

12. As the resultant of the velocities of 1st

and 2nd are just opposite to that of 3rd,


the 4th particle will travel in the line in
which 3rd is travelling.
v
2nd

Option (a) is correct.


9. v2 = 16 ms1

45

v1 = 2gs = 10 ms1

v2'
Rest

Now, v1 =

1+ e
v2
2

Let the velocity of 4th particle is u as


shown in figure.

u cos 45 + v cos 45 = v
i.e.,
u = v ( 2 1)
Total energy released
1
1
1
1
= mv2 + mv2 + mv2 + m [v( 2 1)] 2
2
2
2
2
1
2
2
= mv [3 + ( 2 1) ] = mv2 [3 2 ]
2
Option (a) is correct.

(entering plase)

10 =

13.

b l2 bL l L2 b
yCM = l

+
+

2
2
6
according
l=

bL
bl +
to question
2
mg
L l2 L l L2

or
l l + =
+
+
2 2
2
6

l2 L2
mg
or
=
2
6
A
L
or
l=
3
11.

m1 v m1
1
A
B v2

3rd

1+ e
16
2
1
i.e.,
e=
4
Option (b) is correct.
l
1
L

( l b) + L b l +

2
2
3
10. yCM =
1

( l b) + L b
2

1st

4th

mg

v2

m1v1 = m2v2 m1v2


e=1
2 v2 = v1
m1(2 v2 ) = m2v2 m1v2

(i)

mg

2mg

2mg

2mg

2mg
C

mg
B

C
A

mg
C

mg

mu = 2mv1 C will increase


u the tension in
i.e., v1 =
2 the string.
mu = 2mv 2
u
i.e., v 2 =
2

2 mg

mg

2 mg

C will also increase the


weight of B when collision
takes
place.
Thus,
mu = (2m + m ) v 3
u
i.e., v 3 =
3

Centre of Mass, Conservation of Linear Momentum, Impulse and Collision |


v1 : v2 : v3

193

m A

u u u
= : : = 3 :3 :2
2 2 3
Option (b) is correct.

14.

Place of
collision

time = t (say)

CM

3m
w

h/2

h/2

2m

30

v cos 30

Motion of B :
h
h 1 h
=v
g
2
g
2 g

vCM = v cos 30
3
=v
2
Option (a) is correct.

15. r CM = i

i.e.,

h=v

v=

vB =

r1 (position vector of lighter piece)


^

= 3 ^i + 2 ^j 4 k

r CM =

m1 r1

r2 =

+
m1 + m2

( m1 + m2 ) r CM
m2

m1 r1

= ^j + 2 k
The heavier part will be at (0, 1, 2).
Option (d) is correct.
16. Motion of A :

i.e.,

t=

h
g

h
=0
g

gh g

gh

Rest

2m

m + 2m

m gh = 3 m
gh

=
3
Velocity of the combined mass when it
reach ground
v 2 = 2 + 2 gh
gh
=
+ 2 gh
g
19 gh
i.e.,
v =
3
Option (d) is correct.

2
^
2 i (3 ^i + 2 ^j 4 k)
3
=
4
3
1 ^
^
= [6 i 2 (3 ^i + 2 ^j 4 k)]
4
1
^
= [ 4 ^j + 8 k ]
4
^

h 1 2
= gt , v A (at time t) = g t =
2 2

gh

Collision of A and B at time t :

m2 r2

h
g

gh

17. u = velocity of man w.r.t. cart

Let v = velocity of cart w.r.t. ground


Velocity of man w.r.t. ground = u + v
m (u + v) + 2 mv = 3 m 0
u

v=
3
Work done = KE gained by man and cart
1
1
= m (u + v)2 + 2 mv2
2
2

194 | Mechanics-1
2

1
u
1
u
m u + 2 m
3
2
3
2
1
4u2 1
u2
= m
+ 2m
2
3
2
9
2mu2 mu2
=
+
3
9
7
= mu2
9
Option (d) is correct.
30 m + 50 m
18. vCM =
m+m

1
mv2
2

= 2 mv2

3
mv2
2
3 p2
=
2m
Option (c) is correct.
=

20. According to question


4m

4m
CM

= 40 m/s upwards.
30 m/s

4m
CM

x
a

a
2 = ( 4 m) x + ( m) a
4m + 4m
4m + m

( 4 m) x + ( 4 m)
Initial position of CM

40 m
20 m

x a 4x + a
+ =
2 4
5
a
i.e.,
x=
6
Option (b) is correct.
or

50 m/s

If the velocity of CM becomes zero at


displacements
02 = 402 + 2 ( 10) s

s = 80 m
Maximum height attained by CM
= 20 m + 80 m
= 100 m
Option (c) is correct.

21. xCM of Fig. 1 will as that of Fig. 2.


y
a

40

20

10

a
a
Fig. 1

19. As the masses are equal and the collision

is elastic, the particles will exchange their


velocities as shown in figure.
p

50
O

A
B
Before collision
2v

2v

A
B
After collision

Gain in KE of 1st particle


1
1
= m ( 2 v)2 mu2
2
2

a
x

a
Fig. 2

a
3a
+ 50
2
2
=
40 + 50
20a + 75a
=
90
95a 19 a
=
=
90
18
40

xCM

In terms of velocity
A
B
Before collision

40

2p

In terms of momentum

y
a

20

m
R

Centre of Mass, Conservation of Linear Momentum, Impulse and Collision |


yCM

3a
a
40
+ 50
2
2
=
40 + 50

There is no

need to find

the value of yCM

17 a
18
Option (a) is correct.

195

24. Velocity of 2nd ball when 1st with velocity

v strikes 2nd at rest

22. Conservation of momentum along y-axis

v
m0 0 sin 45 = 2 m0 v sin
2
v
i.e.,
(i)
2 v sin = 0
2
Conservation of momentum along x-axis.
v0/2
Rest

m/22

m/2n 1

1st

2nd

3rd

nth

2m0

2m0
v

v
m0 0 cos 45 + 2 m0 v cos = m0v0
2
v
i.e.,
(ii)
2 v cos = 0
2
Squaring and adding Eqs. (i) and (ii),
v
2v = 0 2
2
v

v= 0
2 2
Option (b) is correct.
1
1
23. kx20 = m2v22
2
2
k
i.e.,
v2 = x0
m2
m 0 + m2v2
vCM = 1
m1 + m2

v2 = v

Rest

v2'

v1'

m2 = m

m1 = m/2

45
x

m0

m/2

v1 =

m0
v0

2m2
2m
4
v2 =
v= v
m
m1 + m2
3
+m
2

Velocity of 3rd ball when 2nd with velocity


4
v strikes 3rd at rest
3
v2 = 4 v
3
2
m2 = m
2

Rest

v2'

3
m1 = m
4

m
2
2m2
2 4v
v1 =
v2 =
m m 3
m1 + m2
+
4
2
4 4
= v
3 3
2
4
= v
3
31
4
Velocity of 3rd ball =
v
3
m2
4
As in every collision
=
m1 + m2 3
n1
4
The velocity of nth ball =
v
3

(When wall just breaks off the velocity of


mass m1 would be zero)
m2
=
v2
m1 + m2
m2
k
=
x0
m1 + m2
m2
x0
=
k m2
m1 + m2

Now, this must be equal to 5 gr


for it to complete the circle
n1
4

v = 5 gr

3
n1
3
i.e.,
v =
5 gr
4

Option (b) is correct.

Option (a) is correct.

v1'
3

196 | Mechanics-1
25. Impulse given to the block will also

release it from abstraction besides giving


and then imparting the restoring force on
it due to velocity to it 5cm of the spring
expansion will accelerate it.
Impulse = 4 kg ms 1
4 kg ms 1
Initial velocity (u) =
= 2 ms 1
2 kg

5cm
6.7 cm
F

F
6.7 cm
3 m/s
6.7 cm

Average Acceleration ( a)
kx
=
=
2m

5
4000

100
22

KE of the block moves blocks ahead and


the spring stretches by 6.7 cm but the
block does not return due to same region.
Block is now at rest for the second time.
Distance travelled by block when it
comes to rest for the second time
= (5 + 6.7 + 6.7 + 6.7) cm
= 25 cm approx.
Option (b) is correct.

= 50 m/s2

5
m
100
v2 = u2 + 2as

Displacement ( s) = x =

= 22 + 2 50

5
100

= 4+5
=9

v = 3 ms 1
Option (b) is correct.
26. Compression in spring Velocity gained

by block when the spring is at its natural


length will compress the spring.
1 2 1
kx = mv2
2
2
m
x=v
k
2
=3
4000
29 5
=
4000 5
45
10000
6.7
m
=
100
= 6.7 cm

Spring at its natural length.


Block at rest first time due to some
reason.
Restoring force brings the spring to its
natural length and block attains a velocity
of 3 ms 1.

M1= 8 m
2v

27.

(M + s)
48 m

M2= 16 m
2v
B

6L
12 L

8 m AP = 16m (12L AP)


AP = 24 L 2 AP
3 AP = 24 L
AP = 8 L
As the CM of M and S does not change,
the CM of the bar shall also not change
i.e., the displacement of bar will be zero.
Let x be the displacement of rod.
x 8 m + ( x + 6 L) 48 m + ( x + 12 L) 16m
= (6L 48 m) + (8 L 24 m)
i.e., x = 0 m
i.e., no displacement of bar.
Option (d) is correct.
8 m (2 v) + 16 m ( v)
28. vCM of M and S =
=0
8 m + 16 m
There CM of M and S will not change
while they move i.e.,the point P (where
they meet) is at the edge of the table
supporting the end B.
Option (b) is correct.

Centre of Mass, Conservation of Linear Momentum, Impulse and Collision |


29. When the spider eats up the moth and

v
relative
2

travels towards A with velocity


to rod.

197

20L 20L
=
=
= 20T
v
L/ T
= 20 4 = 80 s
31. Form CM not to shift

v
24 m + vR + 48 m vR = 0
2

8L
x'

[vR = velocity (absolute) of rod]


v

vR =
6
Option (c) is correct.
30. Time taken by spider to reach point A

starting from point B


4L 8L
=
+
v
v/2

M+S
48 m

48 m

( x + 8 L) 24 m + ( x + 6L) 48 m
= 64 48 m
8L
i.e., x =
3
Option (a) is correct.

More than One Correct Options

v
2

1. Along vertical : 2 mV sin = m sin 45

i.e.,

2 V sin =

v
2 2

m
v
m

Rest
2m

(i)
v/2

45

2m
L

Along horizontal :
v
mv = m cos 45 + 2mV cos
2
1
i.e.,
(ii)
2 V cos = v 1

2 2
Squaring and adding Eq. (i)and (ii),
2
2
v
v

8 V 2 =
+ v

2 2

2 2
2
v2
v
v
=
+ v2 +
2v
8
7
2 2
5v2 v2
=

4
2
Dividing Eq. (ii) by Eq. (i)
1
2
2
tan =
2 21
2 2

1
< 1
2 21

< 45
Thus, the divergence angle between the

particles will be less than


.
2
Option (b) is correct.
1
Initial KE = mv2
2
2
1 v
1
Final KE = m + 2 mV 2
2 2
2
1
1
5
1
= mv2 + 2
2

2
32
8 2
4
As Final KE < Initial KE
Collision is inelastic.
Option (d) is correct.
m m1
2. v2 = 2
v2
m1 + m2
v2

v1 = 0

m2 = m

m1 = 5m

v2'

m 5m
v2
5m + m
2
= v2
3
2
=
2g l
3
=

v1'

198 | Mechanics-1
mv 22
= mg
l
mv 2
or
T=
+ mg
l
m8g
=
+ mg
9
17 mg
=
9
Option (a) is correct.
Velocity of block
2m2
v1 =
v2
m1 + m2
2m
=
2g l
5m + m
1
=
2g l
3
Option (c) is correct.
Maximum height attained by pendulum
bob
v 2 8 g l / 9 4 l
= 2 =
=
2g
2g
g

u
= mv sin e +
v

= mu sin (1 + e)
Option (d) is correct.
u 1 (1 e2 ) sin2
= u 1 sin2 + e2 sin2
= u cos2 + e2 sin2
= u cos2 +

= v2 cos2 + v2 sin2
=v
Option (c) is correct.
cos2 + e2 sin2
tan2
= cos2 +
sin2
tan2
= cos2 (1 + tan2 )
= cos2 sec2
cos2
=
cos2

v sin
v
v cos

Option (d) is correct.

4. u = (3 i + 2 j) ms 1

u sin

v sin
u sin
cos sin
=

cos sin
tan
=
tan
Option (b) is correct.
Change in momentum of particle
= ( mv sin ) ( + mu sin )
Impulse delivered by floor to the particle
= mv sin + mu sin
sin u
= mv sin
+
sin v
and

Final KE
v2
=
2
Initial KE
u

v2
sin2
u2

= u2 cos2 + v2 sin2

Option (d) is correct.


v cos
3. v cos = u cos =
u cos

u cos

u
v

e=

u
mM

v = (2 ^i + ^j) m/s
m

Impulse received by particle of mass m

= mu + mv
= m (3 ^i + 2 ^j) + m( 2 ^i + ^j)
= m (5 ^i + ^j) unit
Option (b) is correct.
Impulse received by particle of mass M
= (impulse received by particle of
mass m)
= m (5 ^i + ^j)
Option (d) is correct.

Centre of Mass, Conservation of Linear Momentum, Impulse and Collision |


5. T = m1a

and

m2 g T = m2 a
m2
Solving,
a=
g
m1 + m2
a
m1

T
T

m2
y
m2g

m1a + m2 0
m1 + m2
m1
=
a
m1 + m2
m1m2
=
g
( m1 + m2 )2

( aCM ) x =

Option (b) is correct.


m 0 + m2 a
( aCM ) y = 1
( m1 + m2 )
m2
=
a
m1 + m2

3
mv2
16
Option (c) is correct.
=

m
=
g
m1 + m2

8. As the mass of the system keeps on

Option (c) is correct.


6. As the block comes down, the CM of the

system will also come down i.e., it does not


remain stationary.
mg
aCM =
g
m+M
aCM is downwards and also aCM < g.
Option (d) is correct.
As no force acts along horizontal direction,
the momentum of the system will remain
conserved along horizontal direction.
Option (c) is correct.
7. Velocity of B after collision :
v2

v1 = 0

v2'

v1'

m2

m1

m1

m2

A
B
Before collision

199

1 + e
v1 =
v
2 2
3
1
= v [as e = and v2 = v (given)]
4
2
v

2
Impulse given by A to B
= change in momentum of B
3
= m v m 0
4
3
= mv
4
Option (b) is correct.
Velocity of A after collision
1 e
v2 =
v
2 2
v
=
4
Loss of KE during collision
1
1
= mv22 m(v 21 + v 22 )
2
2
2
2

1
v
3v
= m v2
4
4
2

A
B
After collision

decreasing momentum of the system does


not remain constant.
Thrust force is developed on the rocket
due to Newtons 3rd law of motion.
Option (b) is correct.
dv vi dm
As,
a=
=

g
dt m dt
The value of a will remain constant if vi
dm
and
are constant.
dt
Option (c) is correct.
Fnet = Ft (Thrust force due to gas ejection)
W (weight of rocket)
F
a = net
m
Thus, Newtons 2nd law is applied.
Option (d) is correct.

200 | Mechanics-1
Match the Columns
1. If x0 is the compression made in the

spring, the restoring force on B will


decrease from kx0 to zero as the spring
regains its original length. Thus, the
acceleration of B will also decrease from
kx0
to zero.
mB

2. Initial aCM =

m ( + g) + m ( + g)
m+m
u=0
2nd particle

180 m
+

mA

mB

20 ms1
1st particle

B
m
x0

kx0

kx0

So, the aCM will also decrease from


kx0
to zero.
m A + mB
(a) (r)
When spring is released after compressing
it, the restoring on B will accelerate it
towards right while the reaction force on
A will apply a force on the wall which in
turn will apply equal and opposite force on
A and consequently A will travel towards
right. As both travel towards right the
velocity of CM will be maximum in the
beginning.
After this A will start compressing the
spring and at a certain instant when the
spring is compressed to maximum value
both the blocks will travel towards right
with a constant velocity and then the
velocity of CM will become constant.
(b) (q)
As the blocks will never move along
y-axis, the y-component of the CM of the
two blocks will not change.
(d) (p)
As the two blocks will keep on moving
towards right (surface below being
smooth) the x-coordinate of the CM of the
blocks will keep on increasing.
(c) (s)

=+g
= + 10 SI unit
(a) (q)
Initial vCM =

m ( 20) + m 0
m+m

= 10

|vCM|= 10 SI unit
(b) (q)
For the time taken by the first particle to
return to ground
1
s = ut + at2
2
0 = ( 20) t + 5t2

t=4s
Now, as the collision of the first particle
with the ground is perfectly inelastic, the
first particle will remain on ground at
rest.
Now, let us find the position of 2nd
particle at t = 5 s
1
s = (0) 5 + (10) 52
2
= 125 m
The particle (2nd) will still be in space
moving downwards.
m 0 + m g
aCM =
m+m
g
(SI unit)
= =5
2
(c) (p)
Velocity of 2nd particle at t = 5 s
v = 0 + 10 5
= 50 ms 1

Centre of Mass, Conservation of Linear Momentum, Impulse and Collision |


At t = 5 s
vCM

m 0 + m 50
=
m+m
(SI unit)

= 25
(d) (s)
3. Initial KE of block B = 4 J
A

m = 0.5 kg

0.5 u2 = 4
2

u = 4 ms 1
Initial momentum of B = 0.5 4
= 2 kg ms 1
(a) (r)
Initial momentum
pCM = pA + pB
=0+2
= 2 kg ms 1
(b) (r)
Velocity given to block B will compress the
spring and this will gradually increase the
velocity of A. When the spring gets
compressed to its maximum both the
blocks will have the same velocities i.e.,
same momentum as both have same mass.
pA = pB
(at maximum compression of the spring)
But, pA + pB = initial momentum of B.

pA + pA = 2
i.e.,
pA = 1 kgms 1
(c) (q)
After the maximum compression in the
spring, the spring will gradually expand
but now the velocity of block A will
increase and that of B will decrease and
when the spring attains maximum
expansion the velocity of B will be zero
and so will be its momentum.
(d) (p)
4. If collision is elastic, the two blocks will

interchange there velocities (mass of both


balls being equal).
Thus, velocity of A after collision = v

201

(a) (r)
If collision is perfectly inelastic, the two
balls will move together (with velocities V).

mv = ( m + m) V
v

V =
2
(b) (s)
1
If collision is inelastic with e = ,
2
1+ e
v1 =
v2
2
1
1+
2 v [Qv = v (given)]
=
2
2
3
= v
4
(c) (p)
1
If collision is inelastic with e = ,
4
1
1 +

4 v = 5 v
v1 =
2
8
(d) (q).
5. If A moves x towards right
A

B
50 kg
C 30 kg

60 kg

Smooth

Let plank (along with B) move by x to the


right.
x 30 + x (60 + 30)

=0
30 + (60 + 30)
x
i.e.,
x =
3
x
= , towards left.
3
(a) (r)
If B moves x towards left
Let plank (along with A) move x to the
left
x 60 + x (30 + 30)

=0
60 + (30 + 30)
i.e.,

x = x
= x, towards right

202 | Mechanics-1
(b) (p)
If A moves x towards right and B moves x
towards left.
Let plank moves x towards right
30 x + 60 ( x) + 30 ( x )

=0
30 + (60) + (30)
i.e., x = x
= x, towards right
(c) (p)
If A and B both move x towards right.
Let plank moves x towards right
(30 + 60) x + 30x

=0
(30 + 60) + 30
i.e.,

x = 3 x
= 3x, towards right

(d) (s)
6. For man to be in equilibrium

1
T1

T1

T1
N

T1

T2
T2

m1
N

T2
T2
m2g

m1g

(i)
N + T1 = W
For the block of mass m1 to be in
quilibrium
(ii)
T2 = N + m1 g
For the block of mass m2 to be in
quilibrium
(iii)
T2 = m2 g
For the equilibrium of pulley 2
(iv)
T1 = 2T2
Solving Eqs. (i), (ii), (iii) and (iv)
W = 3 m2 g m1 g
= (3 m2 m1) g

= (60 10) 10
= 500 N
(a) (r)
For the equilibrium of man
N = W T1
= W 2T2
= W 2 ( N + m1 g)
i.e.,
3 N = W 2m1 g
N = 100 N
(d) (s)
Force exerted by man on string to
accelerate the centre of mass of the system
upwards
Centre of mass of the system will move
upwards if man move upward.
i.e., when
T1 > W N
> 500 10
> 400 N
Options are 500 N and 600 N.
(b) (r) and (s).

Force ( T1) exerted by man on string to


accelerate the centre of mass of the
system.
Centre of mass of the system will move
downward if man moves downward.
i.e., when
T1 < W N
< 400 N
Options are 100 N and 150 N.
(c) (q).
23 +06
7. vCM =
3+6
2ms1

3 kg

6 kg

2
ms 1
3
When both the blocks move with same
velocity (say v) deformation in the spring
will be maximum
2 3 = 3v + 6 v
2
i.e.,
v = ms 1
3
= velocity of A
= velocity of B
=

Centre of Mass, Conservation of Linear Momentum, Impulse and Collision |


vCM

2
2
3 +6
3
3
=
3+6
2
= ms 1
3

When both the blocks move with same


velocities, each will be at rest w.r.t. the
other
(a) (p), (r), (s)
(b) (p), (r), (s)
Minimum speed of 3 kg block will be
2
ms 1 and at that moment velocity of CM
3
2
will be
ms 1.
3
(c) (p)
Initial velocity of 6 kg block is zero.
When the spring is compressed to
maximum value the velocity of 6 kg will
be maximum and the velocity of CM will
2
be ms 1 (as explained above).
3
(c) (p).

8. vCM =

203

2 5 + 1 ( 10)
2+1

= 0 ms 1
+ ive
Rough
1 kg
5 ms1

10 ms1

2 kg
Smooth

(a) (r)
Momentum of CM = 2 ( + 5) + 1 ( 10)
= 0 kg ms 1
(b) (r)

Velocity and so the momentum of 1 kg


block will decrease to zero as the surface
below is rough.
(c) (q)
Velocity and so the KE of 2 kg block will
decrease to zero when the velocity of 1 kg
block becomes zero (according to law of
conservation of momentum).
(d) (q).

Potrebbero piacerti anche